0% found this document useful (0 votes)
23 views71 pages

1000 crt-1

The document presents a series of critical reasoning questions designed for management entrance exams, focusing on logical deductions and conclusions based on given scenarios. It includes various passages and questions that challenge the reader's comprehension and analytical skills. The material is proprietary to Pagalguys.com Education Pvt. Ltd., and reproduction without consent is prohibited.

Uploaded by

9d25vr859b
Copyright
© © All Rights Reserved
We take content rights seriously. If you suspect this is your content, claim it here.
Available Formats
Download as PDF, TXT or read online on Scribd
0% found this document useful (0 votes)
23 views71 pages

1000 crt-1

The document presents a series of critical reasoning questions designed for management entrance exams, focusing on logical deductions and conclusions based on given scenarios. It includes various passages and questions that challenge the reader's comprehension and analytical skills. The material is proprietary to Pagalguys.com Education Pvt. Ltd., and reproduction without consent is prohibited.

Uploaded by

9d25vr859b
Copyright
© © All Rights Reserved
We take content rights seriously. If you suspect this is your content, claim it here.
Available Formats
Download as PDF, TXT or read online on Scribd
You are on page 1/ 71

1

COMPREHENSION & VERBAL ABILITY


Critical Reasoning - 1000

CAT GYAN - 01

Passion turns Gray

Absolute Solution for Management Enterance Exams.

Ó All copyrights to this material vests with Pagalguys .com Education Pvt. Ltd.,
No part of this material either in part or as a whole shall be copied , printed, electronically reproduced sold or
distributed with out the written consent of Pagalguys .com Education Pvt. Ltd. And any such violation
would entail initiation of suitable legal proceedings.
4. In Millington, a city of 50,000 people, Mercedes
TEST 1
Pedrosa, a realtor, calculated that a family with
30 Minutes 20 Questions
Millington’s median family income, $28,000 a year, could
1. Nearly one in three subscribers to Financial afford to buy Millington’s median-priced $77,000 house.
Forecaster is a millionaire, and over half are in top This calculation was based on an 11.2 percent mortgage
management. Shouldn’t you subscribe to Financial interest rate and on the realtor’s assumption that a family
Forecaster now? could only afford to pay up to 25 percent of its income
for housing.
A reader who is neither a millionaire nor in top
management would be most likely to act in accordance Which of the following corrections of a figure appearing
with the advertisement’s suggestion if he or she drew in the passage above, if it were the only correction that
which of the following questionable conclusions invited needed to be made, would yield a new calculation
by the advertisement? showing that even incomes below the median family
(A) Among finance-related periodicals. Financial income would enable families in Millington to afford
Forecaster provides the most detailed financial Millington’s median-priced house?
information. (A) Millington’s total population was 45,000 people.
(B) Top managers cannot do their jobs properly (B) Millington’s median annual family income was
without reading Financial Forecaster. $27,000.
(C) The advertisement is placed where those who will (C) Millington’s median-priced house cost $80,000.
be likely to read it are millionaires. (D) The rate at which people in Millington had to pay
(D) The subscribers mentioned were helped to become mortgage interest was only 10 percent.
millionaires or join top management by reading (E) Families in Millington could only afford to pay up
Financial Forecaster. to 22 percent of their annual income for housing.
(E) Only those who will in fact become millionaires, or
at least top managers, will read the advertisement. 5. Psychological research indicates that college hockey
and football players are more quickly moved to hostility
Questions 2-3 are based on the following. and aggression than are college athletes in noncontact
sports such as swimming. But the researchers’
Contrary to the charges made by some of its opponents, the conclusion—that contact sports encourage and teach
provisions of the new deficit-reduction law for indiscriminate participants to be hostile and aggressive—is untenable.
cuts in the federal budget are justified. Opponents should The football and hockey players were probably more
remember that the New Deal pulled this country out of great hostile and aggressive to start with than the swimmers.
economic troubles even though some of its programs were
Which of the following, if true, would most strengthen
later found to be unconstitutional.
the conclusion drawn by the psychological researchers?
2. The author’s method of attacking the charges of (A) The football and hockey players became more
certain opponents of the new deficit-reduction law is to hostile and aggressive during the season and
(A) attack the character of the opponents rather than remained so during the off-season, whereas there
their claim was no increase in aggressiveness among the
(B) imply an analogy between the law and some New swimmers.
Deal programs (B) The football and hockey players, but not the
(C) point out that the opponents’ claims imply a swimmers, were aware at the start of the
dilemma experiment that they were being tested for
aggressiveness.
(D) show that the opponents’ reasoning leads to an
absurd conclusion (C) The same psychological research indicated that the
football and hockey players had a great respect for
(E) show that the New Deal also called for
cooperation and team play, whereas the swimmers
indiscriminate cuts in the federal budget
were most concerned with excelling as individual
3. The opponents could effectively defend their competitors.
position against the author’s strategy by pointing out (D) The research studies were designed to include no
that college athletes who participated in both contact
(A) the expertise of those opposing the law is and noncontact sports.
outstanding (E) Throughout the United States, more incidents of fan
(B) the lack of justification for the new law does not violence occur at baseball games than occur at
imply that those who drew it up were either inept hockey or football games.
or immoral
6. Ross: The profitability of Company X, restored to
(C) the practical application of the new law will not private ownership five years ago, is clear evidence that
entail indiscriminate budget cuts businesses will always fare better under private than
(D) economic troubles present at the time of the New under public ownership.
Deal were equal in severity to those that have led
Julia: Wrong. A close look at the records shows that X
to the present law
has been profitable since the appointment of a first-
(E) the fact that certain flawed programs or laws have class manager, which happened while X was still in the
improved the economy does not prove that every pubic sector.
such program can do so
Which of the following best describes the weak point in
Ross’s claim on which Julia’s response focuses?

(1)
PaGaLGuYS .com Education
TM
(A) The evidence Ross cites comes from only a single (D) James weighs more than Mark.
observed case, that of Company X. (E) Kelly weighs less than Mark.
(B) The profitability of Company X might be only
temporary. Questions 10-11 are based on the following.
(C) Ross’s statement leaves open the possibility that Partly because of bad weather, but also partly because some
the cause he cites came after the effect he attributes major pepper growers have switched to high-priced cocoa,
to it. world production of pepper has been running well below
(D) No mention is made of companies that are partly worldwide sales for three years. Pepper is consequently in
government owned and partly privately owned. relatively short supply. The price of pepper has soared in
(E) No exact figures are given for the current profits of response: it now equals that of cocoa.
Company X.
10. Which of the following can be inferred from the
7. Stronger patent laws are needed to protect passage?
inventions from being pirated. With that protection, (A) Pepper is a profitable crop only if it is grown on a
manufacturers would be encouraged to invest in the large scale.
development of new products and technologies. Such (B) World consumption of pepper has been unusually
investment frequently results in an increase in a high for three years.
manufacturer’s productivity.
(C) World production of pepper will return to previous
Which of the following conclusions can most properly levels once normal weather returns.
be drawn from the information above? (D) Surplus stocks of pepper have been reduced in the
(A) Stronger patent laws tend to benefit financial past three years.
institutions as well as manufacturers. (E) The profits that the growers of pepper have made in
(B) Increased productivity in manufacturing is likely to the past three years have been unprecedented.
be accompanied by the creation of more
manufacturing jobs. 11. Some observers have concluded that the rise in the
(C) Manufacturers will decrease investment in the price of pepper means that the switch by some growers
development of new products and technologies from pepper to cocoa left those growers no better off
unless there are stronger patent laws. than if none of them had switched; this conclusion,
however, is unwarranted because it can be inferred to be
(D) The weakness of current patent laws has been a
likely that
cause of economic recession.
(A) those growers could not have foreseen how high the
(E) Stronger patent laws would stimulate improvements
price of pepper would go
in productivity for many manufacturers.
(B) the initial cost involved in switching from pepper to
8. Which of the following best completes the passage cocoa is substantial
below? (C) supplies of pepper would not be as low as they are
At large amusement parks, live shows are used very if those growers had not switched crops
deliberately to influence crowd movements. Lunchtime (D) cocoa crops are as susceptible to being reduced by
performances relieve the pressure on a park’s bad weather as are pepper crops
restaurants. Evening performances have a rather (E) as more growers turn to growing cocoa, cocoa
different purpose: to encourage visitors to stay for supplies will increase and the price of cocoa will
supper. Behind this surface divergence in immediate fall precipitously
purpose there is the unified underlying goal of______
(A) keeping the lines at the various rides short by 12. Using computer techniques, researchers analyze
drawing off part of the crowd layers of paint that lie buried beneath the surface layers
of old paintings. They claim, for example, that additional
(B) enhancing revenue by attracting people who come
mountainous scenery once appeared in Leonardo da
only for the live shows and then leave the park
Vinci’s Mona Lisa, which was later painted over. Skeptics
(C) avoiding as far as possible traffic jams caused by reply to these claims, however, that X-ray examinations
visitors entering or leaving the park of the Mona Lisa do not show hidden mountains.
(D) encouraging as many people as possible to come to
Which of the following, if true, would tend most to
the park in order to eat at the restaurants
weaken the force of the skeptics’ objections?
(E) utilizing the restaurants at optimal levels for as
(A) There is no written or anecdotal record that
much of the day as possible
Leonardo da Vinci ever painted over major areas of
9. James weighs more than Kelly. his Mona Lisa.
(B) Painters of da Vinci’s time commonly created images
Luis weighs more than Mark.
of mountainous scenery in the backgrounds of
Mark weighs less than Ned. portraits like the Mona Lisa.
Kelly and Ned are exactly the same weight. (C) No one knows for certain what parts of the Mona
If the information above is true, which of the following Lisa may have been painted by da Vinci’s
must also be true? assistants rather than by da Vinci himself.
(A) Luis weighs more than Ned. (D) Infrared photography of the Mona Lisa has
revealed no trace of hidden mountainous scenery.
(B) Luis weighs more than James.
(E) Analysis relying on X-rays only has the capacity to
(C) Kelly weighs less than Luis.
detect lead-based white pigments in layers of paint
(2)
PaGaLGuYS .com Education
TM
beneath a painting’s surface layers. on public referenda, many of them must learn more
about technology.
13. While Governor Verdant has been in office, the
(B) Thorough studies of technological issues and
state’s budget has increased by an average of 6 percent
innovations should be made a required part of the
each year. While the previous governor was in office, the
public and private school curriculum.
state’s budget increased by an average of 11.5 percent
each year. Obviously, the austere budgets during (C) It should be suggested that prospective voters
Governor Verdant’s term have caused the slowdown in attend applied science courses in order to acquire a
the growth in state spending. minimal competency in technical matters.
(D) If young people are not to be overly influenced by
Which of the following, if true, would most seriously
famous technocrats, they must increase their
weaken the conclusion drawn above?
knowledge of pure science.
(A) The rate of inflation in the state averaged 10 percent
(E) On public referenda issues, young people tend to
each year during the previous governor’s term in
confuse real or probable technologies with
office and 3 percent each year during Verdant’s
impossible ideals.
term.
(B) Both federal and state income tax rates have been 16. In a political system with only two major parties,
lowered considerably during Verdant’s term in the entrance of a third-party candidate into an election
office. race damages the chances of only one of the two major
(C) In each year of Verdant’s term in office, the state’s candidates. The third-party candidate always attracts
budget has shown some increase in spending over some of the voters who might otherwise have voted for
the previous year. one of the two major candidates, but not voters who
(D) During Verdant’s term in office, the state has either support the other candidate. Since a third-party
discontinued or begun to charge private citizens for candidacy affects the two major candidates unequally, for
numerous services that the state offered free to reasons neither of them has any control over, the practice
citizens during the previous governor’s term. is unfair and should not be allowed.
(E) During the previous governor’s term in office, the If the factual information in the passage above is true,
state introduced several so-called “austerity” which of the following can be most reliably inferred
budgets intended to reduce the growth in state from it?
spending. (A) If the political platform of the third party is a
compromise position between that of the two
14. Federal agricultural programs aimed at benefiting major parties, the third party will draw its voters
one group whose livelihood depends on farming often end equally from the two major parties.
up harming another such group.
(B) If, before the emergence of a third party, voters
Which of the following statements provides support for were divided equally between the two major
the claim above? parties, neither of the major parties is likely to
I. An effort to help feed-grain producers resulted in capture much more than one-half of the vote.
higher prices for their crops, but the higher prices (C) A third-party candidate will not capture the votes of
decreased the profits of livestock producers. new voters who have never voted for candidates of
either of the two major parties.
II. In order to reduce crop surpluses and increase
prices, growers of certain crops were paid to leave (D) The political stance of a third party will be more
a portion of their land idle, but the reduction was radical than that of either of the two major parties.
not achieved because improvements in efficiency (E) The founders of a third party are likely to be a
resulted in higher production on the land in use. coalition consisting of former leaders of the two
major parties.
III. Many farm workers were put out of work when a
program meant to raise the price of grain provided 17. Companies considering new cost-cutting
grain growers with an incentive to reduce manufacturing processes often compare the projected
production by giving them surplus grain from results of making the investment against the alternative of
government reserves. not making the investment with costs, selling prices, and
(A) I, but not II and not III share of market remaining constant.
(B) II, but not I and not III Which of the following, assuming that each is a realistic
(C) I and III, but not II possibility, constitutes the most serious disadvantage
(D) II and III, but not I for companies of using the method above for evaluating
(E) I, II and III the financial benefit of new manufacturing processes?
(A) The costs of materials required by the new process
15. Technological education is worsening. People might not be known with certainty.
between eighteen and twenty-four, who are just emerging (B) In several years interest rates might go down,
from their formal education, are more likely to be reducing the interest costs of borrowing money to
technologically illiterate than somewhat older adults. And pay for the investment.
yet, issues for public referenda will increasingly involve
(C) Some cost-cutting processes might require such
aspects of technology.
expensive investments that there would be no net
Which of the following conclusions can be properly gain for many years, until the investment was paid
drawn from the statements above? for by savings in the manufacturing process.
(A) If all young people are to make informed decisions (D) Competitors that do invest in a new process might

(3)
PaGaLGuYS .com Education
TM
reduce their selling prices and thus take market siege were found in the lower level of excavation.
share away from companies that do not.
TEST 2
(E) The period of year chosen for averaging out the cost
30 Minutes 20 Questions
of the investment might be somewhat longer or
shorter, thus affecting the result. 1. After the national speed limit of 55 miles per hour
was imposed in 1974, the number of deaths per mile
18. There are far fewer children available for adoption driven on a highway fell abruptly as a result. Since then,
than there are people who want to adopt. Two million however, the average speed of vehicles on highways has
couples are currently waiting to adopt, but in 1982, the risen, but the number of deaths per mile driven on a
last year for which figures exist, there were only some highway has continued to fall.
50,000 adoptions.
Which of the following conclusions can be properly
Which of the following statements, if true, most drawn from the statements above?
strengthens the author’s claim that there are far fewer
(A) The speed limit alone is probably not responsible
children available for adoption than there are people
for the continued reduction in highway deaths in
who want to adopt?
the years after 1974.
(A) The number of couples waiting to adopt has
(B) People have been driving less since 1974.
increased significantly in the last decade.
(C) Driver-education courses have been more effective
(B) The number of adoptions in the current year is
since 1974 in teaching drivers to drive safely.
greater than the number of adoptions in any
preceding year. (D) In recent years highway patrols have been less
effective in catching drivers who speed.
(C) The number of adoptions in a year is approximately
equal to the number of children available for (E) The change in the speed limit cannot be responsible
adoption in that period. for the abrupt decline in highway deaths in 1974.
(D) People who seek to adopt children often go through 2. Neighboring landholders: Air pollution from the
a long process of interviews and investigation by giant aluminum refinery that has been built next to our
adoption agencies. land is killing our plants.
(E) People who seek to adopt children generally make
Company spokesperson: The refinery is not to blame,
very good parents.
since our study shows that the damage is due to insects
Questions 19-20 are based on the following. and fungi.
Which of the following, if true, most seriously weakens
Archaeologists seeking the location of a legendary siege and the conclusion drawn by the company spokesperson?
destruction of a city are excavating in several possible places,
(A) The study did not measure the quantity of
including a middle and a lower layer of a large mound. The
pollutants emitted into the surrounding air by the
bottom of the middle layer contains some pieces of pottery
aluminum refinery.
of type 3, known to be from a later period than the time of
the destruction of the city, but the lower layer does not. (B) The neighboring landholders have made no change in
the way they take care of their plants.
19. Which of the following hypotheses is best (C) Air pollution from the refinery has changed the
supported by the evidence above? chemical balance in the plants’ environment,
(A) The lower layer contains the remains of the city allowing the harmful insects and fungi to thrive.
where the siege took place. (D) Pollutants that are invisible and odorless are emitted
(B) The legend confuses stories from two different into the surrounding air by the refinery.
historical periods. (E) The various species of insects and fungi mentioned
(C) The middle layer does not represent the period of in the study have been occasionally found in the
the siege. locality during the past hundred years.
(D) The siege lasted for a long time before the city was
3. Sales taxes tend to be regressive, affecting poor
destroyed.
people more severely than wealthy people. When all
(E) The pottery of type 3 was imported to the city by purchases of consumer goods are taxed at a fixed
traders. percentage of the purchase price, poor people pay a
20. The force of the evidence cited above is most larger proportion of their income in sales taxes than
seriously weakened if which of the following is true? wealthy people do.
(A) Gerbils, small animals long native to the area, dig It can be correctly inferred on the basis of the
large burrows into which objects can fall when the statements above that which of the following is true?
burrows collapse. (A) Poor people constitute a larger proportion of the
(B) Pottery of types 1 and 2, found in the lower level, taxpaying population than wealthy people do.
was used in the cities from which, according to the (B) Poor people spend a larger proportion of their
legend, the besieging forces came. income on purchases of consumer goods than
(C) Several pieces of stone from a lower-layer wall have wealthy people do.
been found incorporated into the remains of a (C) Wealthy people pay, on average, a larger amount of
building in the middle layer. sales taxes than poor people do.
(D) Both the middle and the lower layer show evidence (D) The total amount spent by all poor people on
of large-scale destruction of habitations by fire. purchases of consumer goods exceeds the total
(E) Bronze ax heads of a type used at the time of the amount spent by all wealthy people on consumer

(4)
PaGaLGuYS .com Education
TM
goods. $50 would not be persuaded to spend more by any
(E) The average purchase price of consumer goods discount program.
bought by wealthy people is higher than that of (C) Most people who received discounts on home
consumer goods bought by poor people. appliances through Red Label’s program will shop
at Red Label after the program ends.
4. Reviewing historical data, medical researchers in
(D) Since the beginning of the discount program, most
California found that counties with the largest number of
of the people who spend $50 or more at Red Label
television sets per capita have had the lowest incidence of
are people who have never before shopped there
a serious brain disease, mosquito-borne encephalitis. The
and whose average grocery bill has always been
researchers have concluded that people in these counties
higher than $50.
stay indoors more and thus avoid exposure to the disease.
(E) Almost all of the people who have begun spending
The researchers’ conclusion would be most strengthened $50 or more at Red Label since the discount
if which of the following were true? program began are longtime customers who have
(A) Programs designed to control the size of disease- increased the average amount of their shopping
bearing mosquito populations have not affected the bills by making fewer trips.
incidence of mosquito borne encephalitis.
(B) The occupations of county residents affect their 7. Throughout the 1950’s, there were increases in the
risk of exposure to mosquito-borne encephalitis numbers of dead birds found in agricultural areas after
more than does television-watching. pesticide sprayings. Pesticide manufacturers claimed that
the publicity given to bird deaths stimulated volunteers
(C) The incidence of mosquito-borne encephalitis in
to look for dead birds, and that the increase in numbers
counties with the largest number of television sets
reported was attributable to the increase in the number of
per capita is likely to decrease even further.
people looking.
(D) The more time people in a county spend outdoors,
the greater their awareness of the dangers of Which of the following statements, if true, would help
mosquito-borne encephalitis. to refute the claim of the pesticide manufacturers?
(E) The more television sets there are per capita in a (A) The publicity given to bird deaths was largely
county, the more time the average county resident regional and never reached national proportions.
spends watching television. (B) Pesticide sprayings were timed to coincide with
various phases of the life cycles of the insects they
5. The city’s public transportation system should be destroyed.
removed from the jurisdiction of the municipal (C) No provision was made to ensure that a dead bird
government, which finds it politically impossible either would not be reported by more than one observer.
to raise fares or to institute cost-saving reductions in
(D) Initial increases in bird deaths had been noticed by
service. If public transportation were handled by a
agricultural workers long before any publicity had
private firm, profits would be vigorously pursued,
been given to the matter.
thereby eliminating the necessity for covering operating
costs with government funds. (E) Dead birds of the same species as those found in
agricultural areas had been found along coastal
The statements above best support the conclusion that areas where no farming took place.
(A) the private firms that would handle public
transportation would have experience in the 8. Teenagers are often priced out of the labor market
transportation industry by the government-mandated minimum-wage level
(B) political considerations would not prevent private because employers cannot afford to pay that much for
firms from ensuring that revenues cover operating extra help. Therefore, if Congress institutes a
costs subminimum wage, a new lower legal wage for teenagers,
the teenage unemployment rate, which has been rising
(C) private firms would receive government funding if it
since 1960, will no longer increase.
were needed to cover operating costs
(D) the public would approve the cost-cutting actions Which of the following, if true, would most weaken the
taken by the private firm argument above?
(E) the municipal government would not be resigned to (A) Since 1960 the teenage unemployment rate has risen
accumulating merely enough income to cover costs when the minimum wage has risen.
(B) Since 1960 the teenage unemployment rate has risen
6. To entice customers away from competitors, Red even when the minimum wage remained constant.
Label supermarkets have begun offering discounts on (C) Employers often hire extra help during holiday and
home appliances to customers who spend $50 or more on warm weather seasons.
any shopping trip to Red Label. Red Label executives
(D) The teenage unemployment rate rose more quickly
claim that the discount program has been a huge success,
in the 1970’s than it did in the 1960’s.
since cash register receipts of $50 or more are up thirty
percent since the beginning of the program. (E) The teenage unemployment rate has occasionally
declined in the years since 1960.
Which of the following, if true, most seriously weakens
the claim of the Red Label executives? 9. Which of the following best completes the passage
(A) Most people who switched to Red Label after the below?
program began spend more than $50 each time The computer industry’s estimate that it loses millions
they shop at Red Label. of dollars when users illegally copy programs without
(B) Most people whose average grocery bill is less than paying for them is greatly exaggerated. Most of the

(5)
PaGaLGuYS .com Education
TM
illegal copying is done by people with no serious company’s own business
interest in the programs. Thus, the loss to the industry (D) fails to give a reason why the proposal of the
is much smaller than estimated because______ mining company should not be put into effect to
(A) many users who illegally copy programs never find alleviate the concern of the mining company for
any use for them staying in business
(B) most of the illegally copied programs would not be (E) establishes that even the mining company’s business
purchased even if purchasing them were the only will prosper if the mining company’s proposal is
way to obtain them rejected
(C) even if the computer industry received all the
12. Y has been believed to cause Z. A new report,
revenue it claims to be losing, it would still be
noting that Y and Z are often observed to be preceded by
experiencing financial difficulties
X, suggests that X, not Y, may be the cause of Z.
(D) the total market value of all illegal copies is low in
comparison to the total revenue of the computer Which of the following further observations would best
industry support the new report’s suggestion?
(E) the number of programs that are frequently copied (A) In cases where X occurs but Y does not, X is
illegally is low in comparison to the number of usually followed by Z.
programs available for sale (B) In cases where X occurs, followed by Y, Y is
usually followed by Z.
10. This year the New Hampshire Division of (C) In cases where Y occurs but X does not, Y is
Company X set a new record for annual sales by that usually followed by Z.
division. This record is especially surprising since the
(D) In cases where Y occurs but Z does not, Y is
New Hampshire Division has the smallest potential
usually preceded by X.
market and the lowest sales of any of Company X’s
divisions. (E) In cases where Z occurs, it is usually preceded by X
and Y.
Which of the following identifies a flaw in the logical
coherence of the statement above? 13. Mr. Primm: If hospitals were private enterprises,
(A) If overall sales for Company X were sharply dependent on profits for their survival, there would be no
reduced, the New Hampshire Division’s new sales teaching hospitals, because of the intrinsically high cost
record is irrelevant to the company’s prosperity. of running such hospitals.
(B) Since the division is competing against its own Ms. Nakai: I disagree. The medical challenges provided
record, the comparison of its sales record with that by teaching hospitals attract the very best physicians.
of other divisions is irrelevant. This, in turn, enables those hospitals to concentrate on
(C) If this is the first year that the New Hampshire nonroutine cases.
Division has been last in sales among Company Which of the following, if true, would most strengthen
X’s divisions, the new record is not surprising at Ms. Nakai’s attempt to refute Mr. Primm’s claim?
all.
(A) Doctors at teaching hospitals command high
(D) If overall sales for Company X were greater than salaries.
usual, it is not surprising that the New Hampshire
(B) Sophisticated, nonroutine medical care commands a
Division was last in sales.
high price.
(E) Since the New Hampshire Division has the smallest
(C) Existing teaching hospitals derive some revenue
potential market, it is not surprising that it had the
from public subsidies.
lowest sales.
(D) The patient mortality rate at teaching hospitals is
11. Statement of a United States copper mining high.
company: Import quotas should be imposed on the less (E) The modern trend among physicians is to become
expensive copper mined outside the country to maintain highly specialized.
the price of copper in this country; otherwise, our
companies will not be able to stay in business. 14. A recent survey of all auto accident victims in Dole
County found that, of the severely injured drivers and
Response of a United States copper wire manufacturer:
front-seat passengers, 80 percent were not wearing seat
United States wire and cable manufacturers purchase
belts at the time of their accidents. This indicates that, by
about 70 percent of the copper mined in the United
wearing seat belts, drivers and front-seat passengers can
States. If the copper prices we pay are not at the
greatly reduce their risk of being severely injured if they
international level, our sales will drop, and then the
are in an auto accident.
demand for United States copper will go down.
The conclusion above is not properly drawn unless
If the factual information presented by both companies
which of the following is true?
is accurate, the best assessment of the logical
relationship between the two arguments is that the wire (A) Of all the drivers and front-seat passengers in the
manufacturer’s argument survey, more than 20 percent were wearing seat
belts at the time of their accidents.
(A) is self-serving and irrelevant to the proposal of the
mining company (B) Considerably more than 20 percent of drivers and
front-seat passengers in Dole County always wear
(B) is circular, presupposing what it seeks to prove
seat belts when traveling by car.
about the proposal of the mining company
(C) More drivers and front-seat passengers in the
(C) shows that the proposal of the mining company
survey than rear-seat passengers were very
would have a negative effect on the mining
(6)
PaGaLGuYS .com Education
TM
severely injured. Which of the following, if true, would most strengthen
(D) More than half of the drivers and front-seat the conclusion above?
passengers in the survey were not wearing seat (A) An employee’s high stress level can be a cause of
belts at the time of their accidents. unhappiness and poor adjustment for his or her
(E) Most of the auto accidents reported to police in family.
Dole County do not involve any serious injury. (B) People who have responsibility for small children
and who work outside the home have higher stress
15. Six months or so after getting a video recorder, levels than those who do not.
many early buyers apparently lost interest in obtaining
(C) The goal of a national family policy is to lower the
videos to watch on it. The trade of businesses selling and
stress levels of parents.
renting videos is still buoyant, because the number of
homes with video recorders is still growing. But clearly, (D) Any national family policy that is adopted would
once the market for video recorders is saturated, include legislation requiring employers to provide
businesses distributing videos face hard times. paid parental leave and establishing government-
sponsored day care.
Which of the following, if true, would most seriously
(E) Most children who have been cared for in daycare
weaken the conclusion above?
centers are happy and well adjusted.
(A) The market for video recorders would not be
considered saturated until there was one in 80 18. Lark Manufacturing Company initiated a voluntary
percent of homes. Quality Circles program for machine operators.
(B) Among the items handled by video distributors are Independent surveys of employee attitudes indicated that
many films specifically produced as video features. the machine operators participating in the program were
(C) Few of the early buyers of video recorders raised less satisfied with their work situations after two years
any complaints about performance aspects of the of the program’s existence than they were at the
new product. program’s start. Obviously, any workers who participate
in a Quality Circles program will, as a result, become less
(D) The early buyers of a novel product are always
satisfied with their jobs.
people who are quick to acquire novelties, but also
often as quick to tire of them. Each of the following, if true, would weaken the
(E) In a shrinking market, competition always conclusion drawn above EXCEPT:
intensifies and marginal businesses fail. (A) The second survey occurred during a period of
recession when rumors of cutbacks and layoffs at
16. Advertiser: The revenue that newspapers and Lark Manufacturing were plentiful.
magazines earn by publishing advertisements allows (B) The surveys also showed that those Lark machine
publishers to keep the prices per copy of their operators who neither participated in Quality
publications much lower than would otherwise be Circles nor knew anyone who did so reported the
possible. Therefore, consumers benefit economically from same degree of lessened satisfaction with their
advertising. work situations as did the Lark machine operators
Consumer: But who pays for the advertising that pays who participated in Quality Circles.
for low-priced newspapers and magazines? We (C) While participating in Quality Circles at Lark
consumers do, because advertisers pass along Manufacturing, machine operators exhibited two of
advertising costs to us through the higher prices they the primary indicators of improved job
charge for their products. satisfaction: increased productivity and decreased
Which of the following best describes how the absenteeism.
consumer counters the advertiser’s argument? (D) Several workers at Lark Manufacturing who had
(A) By alleging something that, if true, would weaken participated in Quality Circles while employed at
the plausibility of the advertiser’s conclusion other companies reported that, while participating
in Quality Circles in their previous companies,
(B) By questioning the truth of the purportedly factual
their work satisfaction had increased.
statement on which the advertiser’s conclusion is
based (E) The machine operators who participated in Quality
Circles reported that, when the program started,
(C) By offering an interpretation of the advertiser’s
they felt that participation might improve their
opening statement that, if accurate, shows that
work situations.
there is an implicit contradiction in it
(D) By pointing out that the advertiser’s point of view Questions 19-20 are based on the following.
is biased
(E) By arguing that the advertiser too narrowly restricts Blood banks will shortly start to screen all donors for NANB
the discussion to the effects of advertising that are hepatitis. Although the new screening tests are estimated to
economic disqualify up to 5 percent of all prospective blood donors,
they will still miss two-thirds of donors carrying NANB
17. Mr. Lawson: We should adopt a national family hepatitis. Therefore, about 10 percent of actual donors will
policy that includes legislation requiring employers to still supply NANB-contaminated blood.
provide paid parental leave and establishing government-
sponsored day care. Such laws would decrease the stress 19. The argument above depends on which of the
levels of employees who have responsibility for small following assumptions?
children. Thus, such laws would lead to happier, better- (A) Donors carrying NANB hepatitis do not, in a large
adjusted families. percentage of cases, carry other infections for
which reliable screening tests are routinely
(7)
PaGaLGuYS .com Education
TM
performed. (C) How do deep-sea fish survive in total darkness?
(B) Donors carrying NANB hepatitis do not, in a large (D) What are the inherited illnesses to which hamsters
percentage of cases, develop the disease are subject?
themselves at any point. (E) Are there plants that require specific periods of
(C) The estimate of the number of donors who would darkness in order to bloom?
be disqualified by tests for NANB hepatitis is an
underestimate. 3. Millions of identical copies of a plant can be
produced using new tissue-culture and cloning techniques.
(D) The incidence of NANB hepatitis is lower among
the potential blood donors than it is in the If plant propagation by such methods in laboratories
population at large. proves economical, each of the following, if true,
(E) The donors who will still supply NANB- represents a benefit of the new techniques to farmers
contaminated blood will donate blood at the EXCEPT:
average frequency for all donors. (A) The techniques allow the development of superior
strains to take place more rapidly, requiring fewer
20. Which of the following inferences about the generations of plants grown to maturity.
consequences of instituting the new tests is best (B) It is less difficult to care for plants that will grow at
supported by the passage above? rates that do not vary widely.
(A) The incidence of new cases of NANB hepatitis is (C) Plant diseases and pests, once they take hold,
likely to go up by 10 percent. spread more rapidly among genetically uniform
(B) Donations made by patients specifically for their plants than among those with genetic variations.
own use are likely to become less frequent. (D) Mechanical harvesting of crops is less difficult if
(C) The demand for blood from blood banks is likely to plants are more uniform in size.
fluctuate more strongly. (E) Special genetic traits can more easily be introduced
(D) The blood supplies available from blood banks are into plant strains with the use of the new
likely to go down. techniques.
(E) The number of prospective first-time donors is
likely to go up by 5 percent. 4. Which of the following best completes the passage
below?
TEST 3 Sales campaigns aimed at the faltering personal
30 Minutes 20 Questions computer market have strongly emphasized ease of use,
1. Child’s World, a chain of toy stores, has relied on a called user-friendliness. This emphasis is oddly
“supermarket concept” of computerized inventory premature and irrelevant in the eyes of most potential
control and customer self-service to eliminate the buyers, who are trying to address the logically prior
category of sales clerks from its force of employees. It issue of whether______
now plans to employ the same concept in selling (A) user-friendliness also implies that owners can
children’s clothes. service their own computers
The plan of Child’s World assumes that (B) personal computers cost more the more user-
(A) supermarkets will not also be selling children’s friendly they are
clothes in the same manner (C) currently available models are user-friendly enough
(B) personal service by sales personnel is not required to suit them
for selling children’s clothes successfully (D) the people promoting personal computers use them
(C) the same kind of computers will be used in in their own homes
inventory control for both clothes and toys at (E) they have enough sensible uses for a personal
Child’s World computer to justify the expense of buying one
(D) a self-service plan cannot be employed without 5. A weapons-smuggling incident recently took place
computerized inventory control in country Y. We all know that Y is a closed society. So
(E) sales clerks are the only employees of Child’s World Y’s government must have known about the weapons.
who could be assigned tasks related to inventory
Which of the following is an assumption that would
control
make the conclusion above logically correct?
2. Continuous indoor fluorescent light benefits the (A) If a government knows about a particular weapons-
health of hamsters with inherited heart disease. A group smuggling incident, it must have intended to use
of them exposed to continuous fluorescent light survived the weapons for its own purposes.
twenty-five percent longer than a similar group exposed (B) If a government claims that it knew nothing about a
instead to equal periods of indoor fluorescent light and of particular weapons-smuggling incident, it must
darkness. have known everything about it.
The method of the research described above is most (C) If a government does not permit weapons to enter a
likely to be applicable in addressing which of the country, it is a closed society.
following questions? (D) If a country is a closed society, its government has a
(A) Can industrial workers who need to see their work large contingent of armed guards patrolling its
do so better by sunlight or by fluorescent light? borders.
(B) Can hospital lighting be improved to promote the (E) If a country is a closed society, its government has
recovery of patients? knowledge about everything that occurs in the

(8)
PaGaLGuYS .com Education
TM
country. (D) The less standardized a household’s diet is, the
more canned and prepackaged foods the household
6. Banning cigarette advertisements in the mass media discards as waste.
will not reduce the number of young people who smoke.
(E) The more fresh produce a household buys, the more
They know that cigarettes exist and they know how to
fresh produce it throws away.
get them. They do not need the advertisements to supply
that information. Questions 9-10 are based on the following.
The above argument would be most weakened if which
In the past, teachers, bank tellers, and secretaries were
of the following were true?
predominantly men; these occupations slipped in pay and
(A) Seeing or hearing an advertisement for a product status when they became largely occupied by women.
tends to increase people’s desire for that product. Therefore, if women become the majority in currently male-
(B) Banning cigarette advertisements in the mass media dominated professions like accounting, law, and medicine, the
will cause an increase in advertisements in places income and prestige of these professions will also drop.
where cigarettes are sold.
(C) Advertisements in the mass media have been an 9. The argument above is based on
exceedingly large part of the expenditures of the (A) another argument that contains circular reasoning
tobacco companies. (B) an attempt to refute a generalization by means of an
(D) Those who oppose cigarette use have advertised exceptional case
against it in the mass media ever since cigarettes (C) an analogy between the past and the future
were found to be harmful. (D) an appeal to popular beliefs and values
(E) Older people tend to be less influenced by mass- (E) an attack on the character of the opposition
media advertisements than younger people tend to
be. 10. Which of the following, if true, would most likely
be part of the evidence used to refute the conclusion
7. People tend to estimate the likelihood of an event’s above?
occurrence according to its salience; that is, according to
(A) Accountants, lawyers, and physicians attained their
how strongly and how often it comes to their attention.
current relatively high levels of income and prestige
By placement and headlines, newspapers emphasize at about the same time that the pay and status of
stories about local crime over stories about crime teachers, bank tellers, and secretaries slipped.
elsewhere and about many other major events. (B) When large numbers of men join a female-dominated
It can be concluded on the basis of the statements above occupation, such as airline flight attendant, the
that, if they are true, which of the following is most status and pay of the occupation tend to increase.
probably also true? (C) The demand for teachers and secretaries has
(A) The language used in newspaper headlines about increased significantly in recent years, while the
local crime is inflammatory and fails to respect the demand for bank tellers has remained relatively
rights of suspects. stable.
(B) The coverage of international events in newspapers (D) If present trends in the awarding of law degrees to
is neglected in favor of the coverage of local events. women continue, it will be at least two decades
(C) Readers of local news in newspapers tend to before the majority of lawyers are women.
overestimate the amount of crime in their own (E) The pay and status of female accountants, lawyers,
localities relative to the amount of crime in other and physicians today are governed by significantly
places. different economic and sociological forces than
(D) None of the events concerning other people that are were the pay and status of female teachers, bank
reported in newspapers is so salient in people’s tellers, and secretaries in the past.
minds as their own personal experiences.
11. An electric-power company gained greater profits
(E) The press is the news medium that focuses people’s and provided electricity to consumers at lower rates per
attention most strongly on local crimes. unit of electricity by building larger-capacity more
efficient plants and by stimulating greater use of
8. By analyzing the garbage of a large number of
electricity within its area. To continue these financial
average-sized households, a group of modern urban
trends, the company planned to replace an old plant by a
anthropologists has found that a household discards less
plant with triple the capacity of its largest plant.
food the more standardized—made up of canned and
prepackaged foods—its diet is. The more standardized a The company’s plan as described above assumed each
household’s diet is, however, the greater the quantities of of the following EXCEPT:
fresh produce the household throws away. (A) Demand for electricity within the company’s area
Which of the following can be properly inferred from of service would increase in the future.
the passage? (B) Expenses would not rise beyond the level that could
(A) An increasing number of households rely on a be compensated for by efficiency or volume of
highly standardized diet. operation, or both.
(B) The less standardized a household’s diet is, the (C) The planned plant would be sufficiently reliable in
more nonfood waste the household discards. service to contribute a net financial benefit to the
company as a whole.
(C) The less standardized a household’s diet is, the
smaller is the proportion of fresh produce in the (D) Safety measures to be instituted for the new plant
household’s food waste. would be the same as those for the plant it would

(9)
PaGaLGuYS .com Education
TM
replace. (B) Many countries are less concerned with exports
(E) The tripling of capacity would not result in alone than with he balance between exports and
insuperable technological obstacles to efficiency. imports.
(C) With respect to the mix of products each exports,
Questions 12-13 are based on the following. the four countries are very different from each
other.
Meteorologists say that if only they could design an accurate
mathematical model of the atmosphere with all its (D) Of the four countries, two had a much larger, and
complexities, they could forecast the weather with real two had a much smaller, share of total world
precision. But this is an idle boast, immune to any evaluation, exports in 1970 than in 1984.
for any inadequate weather forecast would obviously be (E) The exports of the four countries range from 15
blamed on imperfections in the model. percent to 75 percent of the total national output.

12. Which of the following, if true, could best be used Questions 15-16 are based on the following.
as a basis for arguing against the author’s position that
the meteorologists’ claim cannot be evaluated? In the United States, the Postal Service has a monopoly on
first-class mail, but much of what is sent first class could be
(A) Certain unusual configurations of data can serve as
transmitted electronically. Electronic transmittal operators
the basis for precise weather forecasts even though
argue that if the Postal Service were to offer electronic
the exact causal mechanisms are not understood.
transmission, it would have an unfair advantage, since its
(B) Most significant gains in the accuracy of the electronic transmission service could be subsidized from the
relevant mathematical models are accompanied by profits of the monopoly.
clear gains in the precision of weather forecasts.
(C) Mathematical models of the meteorological 15. Which of the following, if each is true, would allay
aftermath of such catastrophic events as volcanic the electronic transmittal operators’ fears of unfair
eruptions are beginning to be constructed. competition?
(D) Modern weather forecasts for as much as a full day (A) If the Postal Service were to offer electronic
ahead are broadly correct about 80 percent of the transmission, it could not make a profit on first-
time. class mail.
(E) Meteorologists readily concede that the accurate (B) If the Postal Service were to offer electronic
mathematical model they are talking about is not transmission, it would have a monopoly on that
now in their power to construct. kind of service.
(C) Much of the material that is now sent by first-class
13. Which of the following, if true, would cast the most mail could be delivered much faster by special
serious doubt on the meteorologists’ boast, aside from the package couriers, but is not sent that way because
doubt expressed in the passage above? of cost.
(A) The amount of energy that the Earth receives from (D) There is no economy of scale in electronic
the Sun is monitored closely and is known not to transmission—that is, the cost per transaction
be constant. does not go down as more pieces of information
(B) Volcanic eruptions, the combustion of fossil fuels, are transmitted.
and several other processes that also cannot be (E) Electronic transmission will never be cost-effective
quantified with any accuracy are known to have a for material not sent by first-class mail such as
significant and continuing impact on the newspapers and bulk mail.
constitution of the atmosphere.
(C) As current models of the atmosphere are improved, 16. Which of the following questions can be answered
even small increments in complexity will mean on the basis of the information in the passage above?
large increases in the number of computers required (A) Is the Postal Service as efficient as privately owned
for the representation of the models. electric transmission services?
(D) Frequent and accurate data about the atmosphere (B) If private operators were allowed to operate first-
collected at a large number of points both on and class mail services, would they choose to do so?
above the ground are a prerequisite for the (C) Do the electronic transmittal operators believe that
construction of a good model of the atmosphere. the Postal Service makes a profit on first-class
(E) With existing models of the atmosphere, large scale mail?
weather patterns can be predicted with greater (D) Is the Postal Service prohibited from offering
accuracy than can relatively local weather patterns. electronic transmission services?
14. Of the countries that were the world’s twenty (E) Is the Postal Service expected to have a monopoly
largest exporters in 1953, four had the same share of total on electronic transmission?
world exports in 1984 as in 1953. Theses countries can 17. Lists of hospitals have been compiled showing
therefore serve as models for those countries that wish to which hospitals have patient death rates exceeding the
keep their share of the global export trade stable over the national average. The data have been adjusted to allow for
years. differences in the ages of patients.
Which of the following, if true, casts the most serious Each of the following, if true, provides a good logical
doubt on the suitability of those four countries as ground for hospitals to object to interpreting rank on
models in the sense described? these lists as one of the indices of the quality of
(A) Many countries wish to increase their share of hospital care EXCEPT:
world export trade, not just keep it stable.
( 10 )
PaGaLGuYS .com Education
TM
(A) Rank order might indicate insignificant differences, throughout their history, have been situated far
rather than large differences, in numbers of patient inland from sources of sea salt and far south of
deaths. Saharan salt mines.
(B) Hospitals that keep patients longer are likely to (E) No significant differences in salt metabolism have
have higher death rates than those that discharge been found between those people who have had
patients earlier but do not record deaths of patients salt available throughout their history and those
at home after discharge. who have not.
(C) Patients who are very old on admission to a
20. The following proposal to amend the bylaws of an
hospital are less likely than younger patients to
organization was circulated to its members for comment.
survive the same types of illnesses or surgical
procedures. When more than one nominee is to be named for an
(D) Some hospitals serve a larger proportion of low- office, prospective nominees must consent to
income patients, who tend to be more seriously ill nomination and before giving such consent must be told
when admitted to a hospital. who the other nominees will be.
(E) For-profit hospitals sometimes do not provide Which of the following comments concerning the logic
intensive-care units and other expensive services of the proposal is accurate if it cannot be known who
for very sick patients but refer or transfer such the actual nominees are until prospective nominees have
patients to other hospitals. given their consent to be nominated?
(A) The proposal would make it possible for each of
18. Teresa: Manned spaceflight does not have a future, several nominees for an office to be aware of who
since it cannot compete economically with other means of all of the other nominees are.
accomplishing the objectives of spaceflight.
(B) The proposal would widen the choice available to
Edward: No mode of human transportation has a better those choosing among the nominees.
record of reliability: two accidents in twenty-five years. (C) If there are several prospective nominees, the
Thus manned spaceflight definitely has a positive proposal would deny the last nominee equal
future. treatment with the first.
Which of the following is the best logical evaluation of (D) The proposal would enable a prospective nominee
Edward’s argument as a response to Teresa’s argument? to withdraw from competition with a specific
(A) It cites evidence that, if true, tends to disprove the person without making that withdrawal known.
evidence cited by Teresa in drawing her conclusion. (E) If there is more than one prospective nominee, the
(B) It indicates a logical gap in the support that Teresa proposal would make it impossible for anyone to
offers for her conclusion. become a nominee.
(C) It raises a consideration that outweighs the
TEST 4
argument Teresa makes.
30 Minutes 20 Questions
(D) It does not meet Teresa’s point because it assumes
that there is no serious impediment to transporting 1. Which of the following best completes the passage
people into space, but this was the issue raised by below?
Teresa. In a survey of job applicants, two-fifths admitted to
(E) It fails to respond to Teresa’s argument because it being at least a little dishonest. However, the survey
does not address the fundamental issue of whether may underestimate the proportion of job applicants
space activities should have priority over other who are dishonest, because______
claims on the national budget. (A) some dishonest people taking the survey might
have claimed on the survey to be honest
19. Black Americans are, on the whole, about twice as
likely as White Americans to develop high blood (B) some generally honest people taking the survey
pressure. This likelihood also holds for westernized Black might have claimed on the survey to be dishonest
Africans when compared to White Africans. (C) some people who claimed on the survey to be at
least a little dishonest may be very dishonest
Researchers have hypothesized that this predisposition
in westernized Blacks may reflect an interaction (D) some people who claimed on the survey to be
between western high-salt diets and genes that adapted dishonest may have been answering honestly
to an environmental scarcity of salt. (E) some people who are not job applicants are
probably at least a little dishonest
Which of the following statements about present-day,
westernized Black Africans, if true, would most tend to Questions 2-3 are based on the following.
confirm the researchers’ hypothesis?
(A) The blood pressures of those descended from The average life expectancy for the United States population
peoples situated throughout their history in as a whole is 73.9 years, but children born in Hawaii will live
Senegal and Gambia, where salt was always an average of 77 years, and those born in Louisiana, 71.7
available, are low. years. If a newlywed couple from Louisiana were to begin
(B) The unusually high salt consumption in certain their family in Hawaii, therefore, their children would be
areas of Africa represents a serious health problem. expected to live longer than would be the case if the family
remained in Louisiana.
(C) Because of their blood pressure levels, most White
Africans have markedly decreased their salt 2. Which of the following, if true, would most
consumption. seriously weaken the conclusion drawn in the passage?
(D) Blood pressures are low among the Yoruba, who,
( 11 )
PaGaLGuYS .com Education
TM
(A) Insurance company statisticians do not believe that (B) The amount of money accumulated by putting the
moving to Hawaii will significantly lengthen the prepayment funds in an interest-bearing account
average Louisianian’s life. today will be greater than the total cost of tuition
(B) The governor of Louisiana has falsely alleged that for any of the pubic colleges when the child
statistics for his state are inaccurate. enrolls.
(C) The longevity ascribed to Hawaii’s current (C) The annual cost of tuition at the state’s pubic
population is attributable mostly to genetically colleges is expected to increase at a faster rate than
determined factors. the annual increase in the cost of living.
(D) Thirty percent of all Louisianians can expect to live (D) Some of the state’s public colleges are
longer than 77 years. contemplating large increases in tuition next year.
(E) Most of the Hawaiian Islands have levels of air (E) The prepayment plan would not cover the cost of
pollution well below the national average for the room and board at any of the state’s public
United States. colleges.

3. Which of the following statements, if true, would 6. Company Alpha buys free-travel coupons from
most significantly strengthen the conclusion drawn in the people who are awarded the coupons by Bravo Airlines
passage? for flying frequently on Bravo airplanes. The coupons are
(A) As population density increases in Hawaii, life sold to people who pay less for the coupons than they
expectancy figures for that state are likely to be would pay by purchasing tickets from Bravo. This
revised downward. marketing of coupons results in lost revenue for Bravo.
(B) Environmental factors tending to favor longevity are To discourage the buying and selling of free-travel
abundant in Hawaii and less numerous in coupons, it would be best for Bravo Airlines to restrict
Louisiana. the
(C) Twenty-five percent of all Louisianians who move (A) number of coupons that a person can be awarded in
to Hawaii live longer than 77 years. a particular year
(D) Over the last decade, average life expectancy has (B) use of the coupons to those who were awarded the
risen at a higher rate for Louisianians than for coupons and members of their immediate families
Hawaiians. (C) days that the coupons can be used to Monday
(E) Studies show that the average life expectancy for through Friday
Hawaiians who move permanently to Louisiana is (D) amount of time that the coupons can be used after
roughly equal to that of Hawaiians who remain in they are issued
Hawaii. (E) number of routes on which travelers can use the
coupons
4. Insurance Company X is considering issuing a new
policy to cover services required by elderly people who 7. The ice on the front windshield of the car had
suffer from diseases that afflict the elderly. Premiums for formed when moisture condensed during the night. The
the policy must be low enough to attract customers. ice melted quickly after the car was warmed up the next
Therefore, Company X is concerned that the income from morning because the defrosting vent, which blows only
the policies would not be sufficient to pay for the claims on the front windshield, was turned on full force.
that would be made.
Which of the following, if true, most seriously
Which of the following strategies would be most likely jeopardizes the validity of the explanation for the speed
to minimize Company X’s losses on the policies? with which the ice melted?
(A) Attracting middle-aged customers unlikely to (A) The side windows had no ice condensation on them.
submit claims for benefits for many years (B) Even though no attempt was made to defrost the
(B) Insuring only those individuals who did not suffer back window, the ice there melted at the same rate
any serious diseases as children as did the ice on the front windshield.
(C) Including a greater number of services in the policy (C) The speed at which ice on a window melts increases
than are included in other policies of lower cost as the temperature of the air blown on the window
(D) Insuring only those individuals who were rejected increases.
by other companies for similar policies (D) The warm air from the defrosting vent for the front
(E) Insuring only those individuals who are wealthy windshield cools rapidly as it dissipates
enough to pay for the medical services throughout the rest of the car.
(E) The defrosting vent operates efficiently even when
5. A program instituted in a particular state allows
the heater, which blows warm air toward the feet
parents to prepay their children’s future college tuition at
or faces of the driver and passengers, is on.
current rates. The program then pays the tuition annually
for the child at any of the state’s public colleges in which 8. To prevent some conflicts of interest, Congress
the child enrolls. Parents should participate in the could prohibit high-level government officials from
program as a means of decreasing the cost for their accepting positions as lobbyists for three years after such
children’s college education. officials leave government service. One such official
Which of the following, if true, is the most appropriate concluded, however, that such a prohibition would be
reason for parents not to participate in the program? unfortunate because it would prevent high-level
(A) The parents are unsure about which pubic college in government officials from earning a livelihood for three
the state the child will attend. years.

( 12 )
PaGaLGuYS .com Education
TM
The official’s conclusion logically depends on which of designers did not plan for such a contingency.
the following assumptions?
Questions 11-12 are based on the following.
(A) Laws should not restrict the behavior of former
government officials. The fewer restrictions there are on the advertising of legal
(B) Lobbyists are typically people who have services, the more lawyers there are who advertise their
previously been high-level government officials. services, and the lawyers who advertise a specific service
(C) Low-level government officials do not often become usually charge less for that service than lawyers who do not
lobbyists when they leave government service. advertise. Therefore, if the state removes any of its current
(D) High-level government officials who leave restrictions, such as the one against advertisements that do
government service are capable of earning a not specify fee arrangements, overall consumer legal costs
livelihood only as lobbyists. will be lower than if the state retains its current restrictions.
(E) High-level government officials who leave 11. If the statements above are true, which of the
government service are currently permitted to act following must be true?
as lobbyists for only three years.
(A) Some lawyers who now advertise will charge more
9. A conservation group in the United States is trying for specific services if they do not have to specify
to change the long-standing image of bats as frightening fee arrangements in the advertisements.
creatures. The group contends that bats are feared and (B) More consumers will use legal services if there are
persecuted solely because they are shy animals that are fewer restrictions on the advertising of legal
active only at night. services.
Which of the following, if true, would cast the most (C) If the restriction against advertisements that do not
serious doubt on the accuracy of the group’s specify fee arrangements is removed, more lawyers
contention? will advertise their services.
(A) Bats are steadily losing natural roosting places such (D) If more lawyers advertise lower prices for specific
as caves and hollow trees and are thus turning to services, some lawyers who do not advertise will
more developed areas for roosting. also charge less than they currently charge for
those services.
(B) Bats are the chief consumers of nocturnal insects
and thus can help make their hunting territory (E) If the only restrictions on the advertising of legal
more pleasant for humans. services were those that apply to every type of
advertising, most lawyers would advertise their
(C) Bats are regarded as frightening creatures not only in
services.
the United States but also in Europe, Africa, and
South America. 12. Which of the following, if true, would most
(D) Raccoons and owls are shy and active only at night; seriously weaken the argument concerning overall
yet they are not generally feared and persecuted. consumer legal costs?
(E) People know more about the behavior of other (A) The state has recently removed some other
greatly feared animal species, such as lions, restrictions that had limited the advertising of legal
alligators, and snakes, than they do about the services.
behavior of bats. (B) The state is unlikely to remove all of the
restrictions that apply solely to the advertising of
10. Meteorite explosions in the Earth’s atmosphere as
legal services.
large as the one that destroyed forests in Siberia, with
approximately the force of a twelve-megaton nuclear (C) Lawyers who do not advertise generally provide
blast, occur about once a century. legal services of the same quality as those provided
by lawyers who do advertise.
The response of highly automated systems controlled
(D) Most lawyers who now specify fee arrangements in
by complex computer programs to unexpected
their advertisements would continue to do so even
circumstances is unpredictable.
if the specification were not required.
Which of the following conclusions can most properly (E) Most lawyers who advertise specific services do
be drawn, if the statements above are true, about a not lower their fees for those services when they
highly automated nuclear-missile defense system begin to advertise.
controlled by a complex computer program?
(A) Within a century after its construction, the system 13. Defense Department analysts worry that the ability
would react inappropriately and might accidentally of the United States to wage a prolonged war would be
start a nuclear war. seriously endangered if the machine-tool manufacturing
(B) The system would be destroyed if an explosion of a base shrinks further. Before the Defense Department
large meteorite occurred in the Earth’s atmosphere. publicly connected this security issue with the import
quota issue, however, the machine-tool industry raised
(C) It would be impossible for the system to distinguish
the national security issue in its petition for import
the explosion of a large meteorite from the
quotas.
explosion of a nuclear weapon.
(D) Whether the system would respond Which of the following, if true, contributes most to an
inappropriately to the explosion of a large explanation of the machine-tool industry’s raising the
meteorite would depend on the location of the issue above regarding national security?
blast. (A) When the aircraft industries retooled, they provided
(E) It is not certain what the system’s response to the a large amount of work for tool builders.
explosion of a large meteorite would be, if its (B) The Defense Department is only marginally
( 13 )
PaGaLGuYS .com Education
TM
concerned with the effects of foreign competition home as a civilian.
on the machine-tool industry. Which of the following would reveal most clearly the
(C) The machine-tool industry encountered difficulty in absurdity of the conclusion drawn above?
obtaining governmental protection against imports (A) Counting deaths among members of the armed
on grounds other than defense. forces who served in the United States in addition
(D) A few weapons important for defense consist of to deaths among members of the armed forces
parts that do not require extensive machining. serving overseas
(E) Several federal government programs have been (B) Expressing the difference between the numbers of
designed which will enable domestic machine-tool deaths among civilians and members of the armed
manufacturing firms to compete successfully with forces as a percentage of the total number of deaths
foreign toolmakers. (C) Separating deaths caused by accidents during service
14. Opponents of laws that require automobile drivers in the armed forces from deaths caused by combat
and passengers to wear seat belts argue that in a free injuries
society people have the right to take risks as long as the (D) Comparing death rates per thousand members of
people do not harm others as a result of taking the risks. each group rather than comparing total numbers of
As a result, they conclude that it should be each person’s deaths
decision whether or not to wear a seat belt. (E) Comparing deaths caused by accidents in the United
Which of the following, if true, most seriously weakens States to deaths caused by combat in the armed
the conclusion drawn above? forces
(A) Many new cars are built with seat belts that 17. One state adds a 7 percent sales tax to the price of
automatically fasten when someone sits in the most products purchased within its jurisdiction. This tax,
front seat. therefore, if viewed as tax on income, has the reverse
(B) Automobile insurance rates for all automobile effect of the federal income tax: the lower the income, the
owners are higher because of the need to pay for higher the annual percentage rate at which the income is
the increased injuries or deaths of people not taxed.
wearing seat belts. The conclusion above would be properly drawn if
(C) Passengers in airplanes are required to wear seat which of the following were assumed as a premise?
belts during takeoffs and landings. (A) The amount of money citizens spend on products
(D) The rate of automobile fatalities in states that do subject to the state tax tends to be equal across
not have mandatory seat-belt laws is greater than income levels.
the rate of fatalities in states that do have such (B) The federal income tax favors citizens with high
laws. incomes, whereas the state sales tax favors citizens
(E) In automobile accidents, a greater number of with low incomes.
passengers who do not wear seat belts are injured (C) Citizens with low annual incomes can afford to pay
than are passengers who do wear seat belts. a relatively higher percentage of their incomes in
15. The cost of producing radios in Country Q is ten state sales tax, since their federal income tax is
percent less than the cost of producing radios in Country relatively low.
Y. Even after transportation fees and tariff charges are (D) The lower a state’s sales tax, the more it will tend
added, it is still cheaper for a company to import radios to redistribute income from the more affluent
from Country Q to Country Y than to produce radios in citizens to the rest of society.
Country Y. (E) Citizens who fail to earn federally taxable income
The statements above, if true, best support which of are also exempt from the state sales tax.
the following assertions? 18. The average age of chief executive officers (CEO’s)
(A) Labor costs in Country Q are ten percent below in a large sample of companies is 57. The average age of
those in Country Y. CEO’s in those same companies 20 years ago was
(B) Importing radios from Country Q to Country Y will approximately eight years younger. On the basis of those
eliminate ten percent of the manufacturing jobs in data, it can be concluded that CEO’s in general tend to be
Country Y. older now.
(C) The tariff on a radio imported from Country Q to Which of the following casts the most doubt on the
Country Y is less than ten percent of the cost of conclusion drawn above?
manufacturing the radio in Country Y. (A) The dates when the CEO’s assumed their current
(D) The fee for transporting a radio from Country Q to positions have not been specified.
Country Y is more than ten percent of the cost of (B) No information is given concerning the average
manufacturing the radio in Country Q. number of years that CEO’s remain in office.
(E) It takes ten percent less time to manufacture a radio (C) The information is based only on companies that
in Country Q than it does in Country Y. have been operating for at least 20 years.
16. During the Second World War, about 375,000 (D) Only approximate information is given concerning
civilians died in the United States and about 408,000 the average age of the CEO’s 20 years ago.
members of the United States armed forces died overseas. (E) Information concerning the exact number of
On the basis of those figures, it can be concluded that it companies in the sample has not been given.
was not much more dangerous to be overseas in the armed
forces during the Second World War than it was to stay at Questions 19-20 are based on the following.
( 14 )
PaGaLGuYS .com Education
TM
Surveys show that every year only 10 percent of cigarette 2. A proposed ordinance requires the installation in
smokers switch brands. Yet the manufacturers have been new homes of sprinklers automatically triggered by the
spending an amount equal to 10 percent of their gross presence of a fire. However, a home builder argued that
receipts on cigarette promotion in magazines. It follows from because more than ninety percent of residential fires are
these figures that inducing cigarette smokers to switch brands extinguished by a household member, residential
did not pay, and that cigarette companies would have been no sprinklers would only marginally decrease property
worse off economically if they had dropped their advertising. damage caused by residential fires.

19. Of the following, the best criticism of the Which of the following, if true, would most seriously
conclusion that inducing cigarette smokers to switch weaken the home builder’s argument?
brands did not pay is that the conclusion is based on (A) Most individuals have no formal training in how to
(A) computing advertising costs as a percentage of gross extinguish fires.
receipts, not of overall costs (B) Since new homes are only a tiny percentage of
(B) past patterns of smoking and may not carry over to available housing in the city, the new ordinance
the future would be extremely narrow in scope.
(C) the assumption that each smoker is loyal to a single (C) The installation of smoke detectors in new
brand of cigarettes at any one time residences costs significantly less than the
installation of sprinklers.
(D) the assumption that each manufacturer produces
only one brand of cigarettes (D) In the city where the ordinance was proposed, the
average time required by the fire department to
(E) figures for the cigarette industry as a whole and may
respond to a fire was less than the national average.
not hold for a particular company
(E) The largest proportion of property damage that
20. Which of the following, if true, most seriously results from residential fires is caused by fires that
weakens the conclusion that cigarette companies could start when no household member is present.
have dropped advertising without suffering economically?
3. Even though most universities retain the royalties
(A) Cigarette advertisements provide a major
from faculty members’ inventions, the faculty members
proportion of total advertising revenue for
retain the royalties from books and articles they write.
numerous magazines.
Therefore, faculty members should retain the royalties
(B) Cigarette promotion serves to attract first-time from the educational computer software they develop.
smokers to replace those people who have
stopped smoking. The conclusion above would be more reasonably drawn
if which of the following were inserted into the
(C) There exists no research conclusively demonstrating
argument as an additional premise?
that increases in cigarette advertising are related to
increases in smoking. (A) Royalties from inventions are higher than royalties
from educational software programs.
(D) Advertising is so firmly established as a major
business activity of cigarette manufacturers that (B) Faculty members are more likely to produce
they would be unlikely to drop it. educational software programs than inventions.
(E) Brand loyalty is typically not very strong among (C) Inventions bring more prestige to universities than
those who smoke inexpensive cigarettes. do books and articles.
(D) In the experience of most universities, educational
TEST 5 software programs are more marketable than are
30 Minutes 20 Questions books and articles.
1. Toughened hiring standards have not been the (E) In terms of the criteria used to award royalties,
primary cause of the present staffing shortage in public educational software programs are more nearly
schools. The shortage of teachers is primarily caused by comparable to books and articles than to
the fact that in recent years teachers have not experienced inventions.
any improvements in working conditions and their
4. Increases in the level of high-density lipoprotein
salaries have not kept pace with salaries in other
(HDL) in the human bloodstream lower bloodstream-
professions.
cholesterol levels by increasing the body’s capacity to rid
Which of the following, if true, would most support the itself of excess cholesterol. Levels of HDL in the
claims above? bloodstream of some individuals are significantly
(A) Many teachers already in the profession would not increased by a program of regular exercise and weight
have been hired under the new hiring standards. reduction.
(B) Today more teachers are entering the profession Which of the following can be correctly inferred from
with a higher educational level than in the past. the statements above?
(C) Some teachers have cited higher standards for hiring (A) Individuals who are underweight do not run any
as a reason for the current staffing shortage. risk of developing high levels of cholesterol in the
(D) Many teachers have cited low pay and lack of bloodstream.
professional freedom as reasons for their leaving (B) Individuals who do not exercise regularly have a
the profession. high risk of developing high levels of cholesterol in
(E) Many prospective teachers have cited the new the bloodstream late in life.
hiring standards as a reason for not entering the (C) Exercise and weight reduction are the most effective
profession. methods of lowering bloodstream cholesterol levels
in humans.
( 15 )
PaGaLGuYS .com Education
TM
(D) A program of regular exercise and weight reduction airports are flying to destinations in cities over 600
lowers cholesterol levels in the bloodstream of miles away.
some individuals. (D) Many new airports are being built in areas that are
(E) Only regular exercise is necessary to decrease presently served by high-speed ground-
cholesterol levels in the bloodstream of individuals transportation systems.
of average weight. (E) A large proportion of air travelers are vacationers
who are taking long-distance flights.
5. When limitations were in effect on nuclear-arms
testing, people tended to save more of their money, but Questions 8-9 are based on the following.
when nuclear-arms testing increased, people tended to
spend more of their money. The perceived threat of If there is an oil-supply disruption resulting in higher
nuclear catastrophe, therefore, decreases the willingness international oil prices, domestic oil prices in open-market
of people to postpone consumption for the sake of countries such as the United States will rise as well, whether
saving money. such countries import all or none of their oil.
The argument above assumes that 8. If the statement above concerning oil-supply
(A) the perceived threat of nuclear catastrophe has disruptions is true, which of the following policies in an
increased over the years open-market nation is most likely to reduce the long-term
(B) most people supported the development of nuclear economic impact on that nation of sharp and unexpected
arms increases in international oil prices?
(C) people’s perception of the threat of nuclear (A) Maintaining the quantity of oil imported at
catastrophe depends on the amount of nuclear- constant yearly levels
arms testing being done (B) Increasing the number of oil tankers in its fleet
(D) the people who saved the most money when (C) Suspending diplomatic relations with major oil-
nuclear-arms testing was limited were the ones producing nations
who supported such limitations (D) Decreasing oil consumption through conservation
(E) there are more consumer goods available when (E) Decreasing domestic production of oil
nuclear-arms testing increases
9. Which of the following conclusions is best
6. Which of the following best completes the passage supported by the statement above?
below?
(A) Domestic producers of oil in open-market countries
People buy prestige when they buy a premium product. are excluded from the international oil market when
They want to be associated with something special. there is a disruption in the international oil supply.
Mass-marketing techniques and price-reduction (B) International oil-supply disruptions have little, if
strategies should not be used because______ any, effect on the price of domestic oil as long as
(A) affluent purchasers currently represent a shrinking an open-market country has domestic supplies
portion of the population of all purchasers capable of meeting domestic demand.
(B) continued sales depend directly on the maintenance (C) The oil market in an open-market country is
of an aura of exclusivity actually part of the international oil market, even if
(C) purchasers of premium products are concerned with most of that country’s domestic oil is usually sold
the quality as well as with the price of the to consumers within its borders.
products (D) Open-market countries that export little or none of
(D) expansion of the market niche to include a broader their oil can maintain stable domestic oil prices
spectrum of consumers will increase profits even when international oil prices rise sharply.
(E) manufacturing a premium brand is not necessarily (E) If international oil prices rise, domestic distributors
more costly than manufacturing a standard brand of oil in open-market countries will begin to import
of the same product more oil than they export.

7. A cost-effective solution to the problem of airport 10. The average normal infant born in the United States
congestion is to provide high-speed ground transportation weighs between twelve and fourteen pounds at the age of
between major cities lying 200 to 500 miles apart. The three months. Therefore, if a three-month-old child
successful implementation of this plan would cost far less weighs only ten pounds, its weight gain has been below
than expanding existing airports and would also reduce the United States average.
the number of airplanes clogging both airports and Which of the following indicates a flaw in the reasoning
airways. above?
Which of the following, if true, could proponents of the (A) Weight is only one measure of normal infant
plan above most appropriately cite as a piece of development.
evidence for the soundness of their plan? (B) Some three-month-old children weigh as much as
(A) An effective high-speed ground-transportation seventeen pounds.
system would require major repairs to many (C) It is possible for a normal child to weigh ten pounds
highways and mass-transit improvements. at birth.
(B) One-half of all departing flights in the nation’s (D) The phrase “below average” does not necessarily
busiest airport head for a destination in a major mean insufficient.
city 225 miles away.
(E) Average weight gain is not the same as average
(C) The majority of travelers departing from rural weight.
( 16 )
PaGaLGuYS .com Education
TM
11. Red blood cells in which the malarial-fever parasite disease.
resides are eliminated from a person’s body after 120 (C) Because of a new diagnostic technique, many
days. Because the parasite cannot travel to a new people who until this year would have been
generation of red blood cells, any fever that develops in a diagnosed as having the intestinal disease are now
person more than 120 days after that person has moved correctly diagnosed as suffering from intestinal
to a malaria-free region is not due to the malarial parasite. ulcers.
Which of the following, if true, most seriously weakens (D) Because of medical advances this year, far fewer
the conclusion above? people who contract the intestinal disease will
(A) The fever caused by the malarial parasite may develop severe cases of the disease.
resemble the fever caused by flu viruses. (E) The water in the rural county was brought up to the
(B) The anopheles mosquito, which is the principal sanitary standards of the water in neighboring
insect carrier of the malarial parasite, has been counties ten years ago.
eradicated in many parts of the world.
14. The price the government pays for standard
(C) Many malarial symptoms other than the fever, weapons purchased from military contractors is
which can be suppressed with antimalarial determined by a pricing method called “historical
medication, can reappear within 120 days after the costing.” Historical costing allows contractors to protect
medication is discontinued. their profits by adding a percentage increase, based on the
(D) In some cases, the parasite that causes malarial current rate of inflation, to the previous year’s
fever travels to cells of the spleen, which are less contractual price.
frequently eliminated from a person’s body than
Which of the following statements, if true, is the best
are red blood cells.
basis for a criticism of historical costing as an
(E) In any region infested with malaria-carrying economically sound pricing method for military
mosquitoes, there are individuals who appear to be contracts?
immune to malaria.
(A) The government might continue to pay for past
12. Fact 1: Television advertising is becoming less inefficient use of funds.
effective: the proportion of brand names promoted on (B) The rate of inflation has varied considerably over
television that viewers of the advertising can recall is the past twenty years.
slowly decreasing. (C) The contractual price will be greatly affected by the
Fact 2: Television viewers recall commercials aired first cost of materials used for the products.
or last in a cluster of consecutive commercials far better (D) Many taxpayers question the amount of money the
than they recall commercials aired somewhere in the government spends on military contracts.
middle. (E) The pricing method based on historical costing
Fact 2 would be most likely to contribute to an might not encourage the development of innovative
explanation of fact 1 if which of the following were also weapons.
true?
15. Some who favor putting governmental enterprises
(A) The average television viewer currently recalls into private hands suggest that conservation objectives
fewer than half the brand names promoted in would in general be better served if private environmental
commercials he or she saw. groups were put in charge of operating and financing the
(B) The total time allotted to the average cluster of national park system, which is now run by the
consecutive television commercials is decreasing. government.
(C) The average number of hours per day that people Which of the following, assuming that it is a realistic
spend watching television is decreasing. possibility, argues most strongly against the suggestion
(D) The average number of clusters of consecutive above?
commercials per hour of television is increasing. (A) Those seeking to abolish all restrictions on
(E) The average number of television commercials in a exploiting the natural resources of the parks might
cluster of consecutive commercials is increasing. join the private environmental groups as members
and eventually take over their leadership.
13. The number of people diagnosed as having a certain
intestinal disease has dropped significantly in a rural (B) Private environmental groups might not always
county this year, as compared to last year, Health agree on the best ways to achieve conservation
officials attribute this decrease entirely to improved objectives.
sanitary conditions at water-treatment plants, which (C) If they wished to extend the park system, the
made for cleaner water this year and thus reduced the private environmental groups might have to seek
incidence of the disease. contributions from major donors and the general
public.
Which of the following, if true, would most seriously
weaken the health officials’ explanation for the lower (D) There might be competition among private
incidence of the disease? environmental groups for control of certain park
areas.
(A) Many new water-treatment plants have been built
in the last five years in the rural county. (E) Some endangered species, such as the California
condor, might die out despite the best efforts of
(B) Bottled spring water has not been consumed in
the private environmental groups, even if those
significantly different quantities by people
groups are not hampered by insufficient resources.
diagnosed as having the intestinal disease, as
compared to people who did not contract the 16. A recent spate of launching and operating mishaps
( 17 )
PaGaLGuYS .com Education
TM
with television satellites led to a corresponding surge in (B) The signals received on the satellite are stronger
claims against companies underwriting satellite insurance. than those received by a terrestrial telescope.
As a result, insurance premiums shot up, making (C) The resolution of detail achieved by the satellite-
satellites more expensive to launch and operate. This, in Earth interferometer system is inferior to that
turn, has added to the pressure to squeeze more achieved by exclusively terrestrial systems.
performance out of currently operating satellites. (D) The computer programs required for making use of
Which of the following, if true, taken together with the the signals received by the satellite required a long
information above, best supports the conclusion that time for development.
the cost of television satellites will continue to increase? (E) The location of an orbiting satellite relative to
(A) Since the risk to insurers of satellites is spread over locations on Earth can be well enough known for
relatively few units, insurance premiums are interferometric purposes.
necessarily very high.
19. Recent estimates predict that between 1982 and
(B) When satellites reach orbit and then fail, the causes
1995 the greatest increase in the number of people
of failure are generally impossible to pinpoint with
employed will be in the category of low-paying service
confidence.
occupations. This category, however, will not increase its
(C) The greater the performance demands placed on share of total employment, whereas the category of high-
satellites, the more frequently those satellites break paying service occupations will increase its share.
down.
If the estimates above are accurate, which of the
(D) Most satellites are produced in such small numbers
following conclusions can be drawn?
that no economies of scale can be realized.
(A) In 1982 more people were working in low-paying
(E) Since many satellites are built by unwieldy
service occupations than were working in high-
international consortia, inefficiencies are inevitable.
paying service occupations.
17. Tocqueville, a nineteenth-century writer known for (B) In 1995 more people will be working in high-paying
his study of democracy in the United States, believed that service occupations than will be working in low-
a government that centralizes power in one individual or paying service occupations.
institution is dangerous to its citizens. Biographers claim (C) Nonservice occupations will account for the same
that Tocqueville disliked centralized government because share of total employment in 1995 as in 1982.
he blamed Napoleon’s rule for the poverty of his (D) Many of the people who were working in low-
childhood in Normandy. paying service occupations in 1982 will be working
Which of the following, if true, would cast the most in high-paying service occupations by 1995.
serious doubt on the biographers’ claim? (E) The rate of growth for low-paying service
(A) Although Napoleon was popularly blamed at the occupations will be greater than the overall rate of
time for the terrible living conditions in Normandy, employment growth between 1982 and 1995.
historians now know that bad harvests were really
to blame for the poor economic conditions. 20. For a local government to outlaw all strikes by its
workers is a costly mistake, because all its labor disputes
(B) Napoleon was notorious for refusing to share power
must then be settled by binding arbitration, without any
with any of his political associates.
negotiated public-sector labor settlements guiding the
(C) Tocqueville said he knew that if his father had not arbitrators. Strikes should be outlawed only for categories
suffered ill health, his family would have had a of public-sector workers for whose services no acceptable
steady income and a comfortable standard of living. substitute exists.
(D) Although Tocqueville asserted that United States
The statements above best support which of the
political life was democratic, the United States of
following conclusions?
the nineteenth century allowed political power to
be concentrated in a few institutions. (A) Where public-service workers are permitted to
strike, contract negotiations with those workers are
(E) Tocqueville once wrote in a letter that, although his
typically settled without a strike.
childhood was terribly impoverished, it was not
different from the experience of his friends and (B) Where strikes by all categories of pubic-sector
neighbors in Normandy. workers are outlawed, no acceptable substitutes
for the services provided by any of those workers
18. Radio interferometry is a technique for studying are available.
details of celestial objects that combines signals (C) Binding arbitration tends to be more advantageous
intercepted by widely spaced radio telescopes. This for public-service workers where it is the only
technique requires ultraprecise timing, exact knowledge of available means of settling labor disputes with such
the locations of the telescopes, and sophisticated workers.
computer programs. The successful interferometric (D) Most categories of public-sector workers have no
linking of an Earth-based radio telescope with a radio counterparts in the private sector.
telescope on an orbiting satellite was therefore a
(E) A strike by workers in a local government is unlikely
significant technological accomplishment.
to be settled without help from an arbitrator.
Which of the following can be correctly inferred from
the statements above? TEST 6
(A) Special care was taken in the launching of the 30 Minutes 20 Questions
satellite so that the calculations of its orbit would 1. Rural households have more purchasing power than
be facilitated. do urban or suburban households at the same income
( 18 )
PaGaLGuYS .com Education
TM
level, since some of the income urban and suburban a whole.
households use for food and shelter can be used by rural (D) The biggest growth in sales in the manufacturing
households for other needs. sector has come in industries that market the most
Which of the following inferences is best supported by innovative products.
the statement made above? (E) Recent improvements in management practices have
(A) The average rural household includes more people allowed many manufacturers to experience
than does the average urban or suburban enormous gains in worker productivity.
household.
Questions 4-5 refer to the following.
(B) Rural households have lower food and housing costs
than do either urban or suburban households. If the airspace around centrally located airports were
(C) Suburban households generally have more restricted to commercial airliners and only those private
purchasing power than do either rural or urban planes equipped with radar, most of the private-plane traffic
households. would be forced to use outlying airfields. Such a reduction in
(D) The median income of urban and suburban the amount of private-plane traffic would reduce the risk of
households is generally higher than that of rural midair collision around the centrally located airports.
households.
4. The conclusion drawn in the first sentence depends
(E) All three types of households spend more of their on which of the following assumptions?
income on food and housing than on all other
(A) Outlying airfields would be as convenient as
purchases combined.
centrally located airports for most pilots of private
2. In 1985 state border colleges in Texas lost the planes.
enrollment of more than half, on average, of the Mexican (B) Most outlying airfields are not equipped to handle
nationals they had previously served each year. Teaching commercial-airline traffic.
faculties have alleged that this extreme drop resulted from (C) Most private planes that use centrally located
a rise in tuition for international and out-of-state students airports are not equipped with radar.
from $40 to $120 per credit hour. (D) Commercial airliners are at greater risk of becoming
Which of the following, if feasible, offers the best involved in midair collisions than are private
prospects for alleviating the problem of the drop in planes.
enrollment of Mexican nationals as the teaching faculties (E) A reduction in the risk of midair collision would
assessed it? eventually lead to increases in commercial-airline
(A) Providing grants-in-aid to Mexican nationals to traffic.
study in Mexican universities
5. Which of the following, if true, would most
(B) Allowing Mexican nationals to study in Texas
strengthen the conclusion drawn in the second sentence?
border colleges and to pay in-state tuition rates,
which are the same as the previous international (A) Commercial airliners are already required by law to
rate be equipped with extremely sophisticated radar
systems.
(C) Reemphasizing the goals and mission of the Texas
state border colleges as serving both in-state (B) Centrally located airports are experiencing over-
students and Mexican nationals crowded airspace primarily because of sharp
increases in commercial-airline traffic.
(D) Increasing the financial resources of Texas colleges
by raising the tuition for in-state students (C) Many pilots of private planes would rather buy
attending state institutions. radar equipment than be excluded from centrally
located airports.
(E) Offering career counseling for those Mexican
nationals who graduate from state border colleges (D) The number of midair collisions that occur near
and intend to return to Mexico centrally located airports has decreased in recent
years.
3. Affirmative action is good business. So asserted the (E) Private planes not equipped with radar systems
National Association of Manufacturers while urging cause a disproportionately large number of midair
retention of an executive order requiring some federal collisions around centrally located airports.
contractors to set numerical goals for hiring minorities
and women. “Diversity in work force participation has 6. Which of the following best completes the passage
produced new ideas in management, product below?
development, and marketing,” the association claimed. Established companies concentrate on defending what
The association’s argument as it is presented in the they already have. Consequently, they tend not to be
passage above would be most strengthened if which of innovative themselves and tend to underestimate the
the following were true? effects of the innovations of others. The clearest
(A) The percentage of minority and women workers in example of this defensive strategy is the fact
business has increased more slowly than many that______
minority and women’s groups would prefer. (A) ballpoint pens and soft-tip markers have eliminated
(B) Those businesses with the highest percentages of the traditional market for fountain pens, clearing
minority and women workers are those that have the way for the marketing of fountain pens as
been the most innovative and profitable. luxury or prestige items
(C) Disposable income has been rising as fast among (B) a highly successful automobile was introduced by
minorities and women as among the population as the same company that had earlier introduced a

( 19 )
PaGaLGuYS .com Education
TM
model that had been a dismal failure decreased seven percent. During the same period, there
(C) a once-successful manufacturer of slide rules reacted has been an equivalent rise in the number of persons
to the introduction of electronic calculators by riding buses into the midtown area. Obviously, the
trying to make better slide rules mayor’s publicity campaign has convinced many people
(D) one of the first models of modern accounting to leave their cars at home and ride the bus to work.
machines, designed for use in the banking industry, Which of the following, if true, casts the most serious
was purchased by a public library as well as by doubt on the conclusion drawn above?
banks (A) Fares for all bus routes in Greenville have risen an
(E) the inventor of a commonly used anesthetic did not average of five percent during the past six months.
intend the product to be used by dentists, who (B) The mayor of Greenville rides the bus to City Hall
currently account for almost the entire market for in the city’s midtown area.
that drug (C) Road reconstruction has greatly reduced the number
7. Most archaeologists have held that people first of lanes available to commuters in major streets
reached the Americas less than 20,000 years ago by leading to the midtown area during the past six
crossing a land bridge into North America. But recent months.
discoveries of human shelters in South America dating (D) The number of buses entering the midtown area of
from 32,000 years ago have led researchers to speculate Greenville during the morning hours is exactly the
that people arrived in South America first, after voyaging same now as it was one year ago.
across the Pacific, and then spread northward. (E) Surveys show that longtime bus riders are no more
Which of the following, if it were discovered, would be satisfied with the Greenville bus service than they
pertinent evidence against the speculation above? were before the mayor’s publicity campaign began.
(A) A rock shelter near Pittsburgh, Pennsylvania, 10. In the aftermath of a worldwide stock-market crash,
contains evidence of use by human beings 19,000 Country T claimed that the severity of the stock-market
years ago. crash it experienced resulted from the accelerated process
(B) Some North American sites of human habitation of denationalization many of its industries underwent
predate any sites found in South America. shortly before the crash.
(C) The climate is warmer at the 32,000-year-old south Which of the following, if it could be carried out, would
American site than at the oldest known North be most useful in an evaluation of Country T’s
American site. assessment of the causes of the severity of its stock-
(D) The site in South America that was occupied 32,000 market crash?
years ago was continuously occupied until 6,000 (A) Calculating the average loss experienced by
years ago. individual traders in Country T during the crash
(E) The last Ice Age, between 11,500 and 20,000 years (B) Using economic theory to predict the most likely
ago, considerably lowered worldwide sea levels. date of the next crash in Country T
8. In Asia, where palm trees are non-native, the trees’ (C) Comparing the total number of shares sold during
flowers have traditionally been pollinated by hand, which the worst days of the crash in Country T to the
has kept palm fruit productivity unnaturally low. When total number of shares sold in Country T just prior
weevils known to be efficient pollinators of palm flowers to the crash
were introduced into Asia in 1980, palm fruit (D) Comparing the severity of the crash in Country T
productivity increased—by up to fifty percent in some to the severity of the crash in countries otherwise
areas—but then decreased sharply in 1984. economically similar to Country T that have not
experienced recent denationalization
Which of the following statements, if true, would best
explain the 1984 decrease in productivity? (E) Comparing the long-term effects of the crash on the
purchasing power of the currency of Country T to
(A) Prices for palm fruit fell between 1980 and 1984
the immediate, more severe short-term effects of
following the rise in production and a concurrent
the crash on the purchasing power of the currency
fall in demand.
of Country T
(B) Imported trees are often more productive than
native trees because the imported ones have left 11. With the emergence of biotechnology companies, it
behind their pests and diseases in their native was feared that they would impose silence about
lands. proprietary results on their in-house researchers and their
(C) Rapid increases in productivity tend to deplete academic consultants. This constraint, in turn, would
trees of nutrients needed for the development of slow the development of biological science and
the fruit-producing female flowers. engineering.
(D) The weevil population in Asia remained at Which of the following, if true, would tend to weaken
approximately the same level between 1980 and most seriously the prediction of scientific secrecy
1984. described above?
(E) Prior to 1980 another species of insect pollinated (A) Biotechnological research funded by industry has
the Asian palm trees, but not as efficiently as the reached some conclusions that are of major
species of weevil that was introduced in 1980. scientific importance.
(B) When the results of scientific research are kept
9. Since the mayor’s publicity campaign for
secret, independent researchers are unable to build
Greenville’s bus service began six months ago, morning
on those results.
automobile traffic into the midtown area of the city has
( 20 )
PaGaLGuYS .com Education
TM
(C) Since the research priorities of biotechnology Any of the following, if introduced into the argument as
companies are not the same as those of academic an additional premise, makes the argument above
institutions, the financial support of research by logically correct EXCEPT:
such companies distorts the research agenda. (A) Collard greens have more nutritional value than kale.
(D) To enhance the companies’ standing in the scientific (B) Spinach has more nutritional value than lettuce.
community, the biotechnology companies (C) Spinach has more nutritional value than collard
encourage employees to publish their results, greens.
especially results that are important.
(D) Spinach and collard greens have the same nutritional
(E) Biotechnology companies devote some of their value.
research resources to problems that are of
(E) Kale and collard greens have the same nutritional
fundamental scientific importance and that are not
value.
expected to produce immediate practical
applications. 15. On the basis of a decrease in the college-age
population, many colleges now anticipate increasingly
12. Some people have questioned the judge’s
smaller freshman classes each year. Surprised by a 40
objectivity in cases of sex discrimination against women.
percent increase in qualified applicants over the previous
But the record shows that in sixty percent of such cases,
year, however, administrators at Nice College now plan
the judge has decided in favor of the women. This record
to hire more faculty for courses taken by all freshmen.
demonstrates that the judge has not discriminated against
women in cases of sex discrimination against women. Which of the following statements about Nice College’s
current qualified applicants, if true, would strongly
The argument above is flawed in that it ignores the
suggest that the administrators’ plan is flawed?
possibility that
(A) A substantially higher percentage than usual plan to
(A) a large number of the judge’s cases arose out of
study for advanced degrees after graduation from
allegations of sex discrimination against women
college.
(B) many judges find it difficult to be objective in cases
(B) According to their applications, their level of
of sex discrimination against women
participation in extracurricular activities and
(C) the judge is biased against women defendants or varsity sports is unusually high.
plaintiffs in cases that do not involve sex
(C) According to their applications, none of them lives
discrimination
in a foreign country.
(D) the majority of the cases of sex discrimination
(D) A substantially lower percentage than usual rate
against women that have reached the judge’s court
Nice College as their first choice among the colleges
have been appealed from a lower court
to which they are applying.
(E) the evidence shows that the women should have
(E) A substantially lower percentage than usual list
won in more than sixty percent of the judge’s cases
mathematics as their intended major.
involving sex discrimination against women
Questions 16-17 are based on the following.
13. The tobacco industry is still profitable and
projections are that it will remain so. In the United States A researcher discovered that people who have low levels of
this year, the total amount of tobacco sold by tobacco- immune-system activity tend to score much lower on tests of
farmers has increased, even though the number of adults mental health than do people with normal or high immune-
who smoke has decreased. system activity. The researcher concluded from this
Each of the following, if true, could explain the experiment that the immune system protects against mental
simultaneous increase in tobacco sales and decrease in illness as well as against physical disease.
the number of adults who smoke EXCEPT:
16. The researcher’s conclusion depends on which of
(A) During this year, the number of women who have
the following assumptions?
begun to smoke is greater than the number of men
who have quit smoking. (A) High immune-system activity protects against
mental illness better than normal immune-system
(B) The number of teen-age children who have begun to
activity does.
smoke this year is greater than the number of
adults who have quit smoking during the same (B) Mental illness is similar to physical disease in its
period. effects on body system.
(C) During this year, the number of nonsmokers who (C) People with high immune-system activity cannot
have begun to use chewing tobacco or snuff is develop mental illness.
greater than the number of people who have quit (D) Mental illness does not cause people’s immune-
smoking. system activity to decrease.
(D) The people who have continued to smoke consume (E) Psychological treatment of mental illness is not as
more tobacco per person than they did in the past. effective as is medical treatment.
(E) More of the cigarettes made in the United States
17. The researcher’s conclusion would be most
this year were exported to other countries than
seriously weakened if it were true that
was the case last year.
(A) there was a one-year delay between the completion
14. Kale has more nutritional value than spinach. But of a pilot study for the experiment and the
since collard greens have more nutritional value than initiation of the experiment itself
lettuce, it follows that kale has more nutritional value (B) people’s levels of immune-system activity are not
than lettuce. affected by their use of medications
( 21 )
PaGaLGuYS .com Education
TM
(C) a few people with high immune-system activity had open the possibility that
scores on the test of mental health that were (A) the compliance date was subject to postponement,
similar to the scores of people who had normal according to the terms of the treaty
immune-system activity (B) one of the countries might not be required to make
(D) people who have low immune-system activity tend any changes or take any steps in order to comply
to contract more viral infections than do people with the treaty, whereas all the other countries are
with normal or high immune-system activity so required
(E) high levels of stress first cause mental illness and (C) each country might have a well-founded excuse,
then cause decreased immune-system activity in based on the provision, for its own lack of
normal individuals compliance
18. The value of a product is determined by the ratio of (D) the treaty specified that the signal for one of the
its quality to its price. The higher the value of a product, countries to initiate action was notification by the
the better will be its competitive position. Therefore, other countries that they had completed action
either increasing the quality or lowering the price of a (E) there was ambiguity with respect to the date after
given product will increase the likelihood that consumer which all actions contemplated in the treaty are to
will select that product rather than a competing one. be complete
Which of the following, if true, would most strengthen TEST 7
the conclusion drawn above? 30 Minutes 20 Questions
(A) It is possible to increase both the quality and the
1. A milepost on the towpath read “21” on the side
price of a product without changing its competitive
facing the hiker as she approached it and “23” on its
position.
back. She reasoned that the next milepost forward on the
(B) For certain segments of the population of path would indicate that she was halfway between one
consumers, higher-priced brands of some product end of the path and the other. However, the milepost one
lines are preferred to the lower-priced brands. mile further on read “20” facing her and “24” behind.
(C) Competing products often try to appeal to different
Which of the following, if true, would explain the
segments of the population of consumers.
discrepancy described above?
(D) The competitive position of a product can be
(A) The numbers on the next milepost had been
affected by such factors as advertising and brand
reversed.
loyalty.
(B) The numbers on the mileposts indicate kilometers,
(E) Consumers’ perceptions of the quality of a product
not miles.
are based on the actual quality of the product.
(C) The facing numbers indicate miles to the end of the
19. In January there was a large drop in the number of path, not miles from the beginning.
new houses sold, because interest rates for mortgages (D) A milepost was missing between the two the hiker
were falling and many consumers were waiting to see how encountered.
low the rates would go. This large sales drop was (E) The mileposts had originally been put in place for
accompanied by a sharp rise in the average price of new the use of mountain bikers, not for hikers.
houses sold.
Which of the following, if true, best explains the sharp 2. Airline: Newly developed collision-avoidance
rise in the average price of new houses? systems, although not fully tested to discover potential
malfunctions, must be installed immediately in passenger
(A) Sales of higher-priced houses were unaffected by
planes. Their mechanical warnings enable pilots to avoid
the sales drop because their purchasers have fewer
crashes.
constraints limiting the total amount they pay.
(B) Labor agreements of builders with construction Pilots: Pilots will not fly in planes with collision-
unions are not due to expire until the next January. avoidance systems that are not fully tested.
Malfunctioning systems could mislead pilots, causing
(C) The prices of new houses have been rising slowly
crashes.
over the past three years because there is an
increasing shortage of housing. The pilots’ objection is most strengthened if which of
(D) There was a greater amount of moderate-priced the following is true?
housing available for resale by owners during (A) It is always possible for mechanical devices to
January than in the preceding three months. malfunction.
(E) Interest rates for home mortgages are expected to (B) Jet engines, although not fully tested when first put
rise sharply later in the year if predictions of into use, have achieved exemplary performance and
increased business activity in general prove to be safety records.
accurate. (C) Although collision-avoidance systems will enable
pilots to avoid some crashes, the likely
20. Seven countries signed a treaty binding each of them malfunctions of the not-fully-tested systems will
to perform specified actions on a certain fixed date, with cause even more crashes.
the actions of each conditional on simultaneous action
(D) Many airline collisions are caused in part by the
taken by the other countries. Each country was also to
exhaustion of overworked pilots.
notify the six other countries when it had completed its
action. (E) Collision-avoidance systems, at this stage of
development, appear to have worked better in
The simultaneous-action provision of the treaty leaves passenger planes than in cargo planes during
( 22 )
PaGaLGuYS .com Education
TM
experimental flights made over a six-month period. river after the dam was built has been 43 degrees.
(D) Nonnative species of fish, introduced into the
3. Guitar strings often go “dead”—become less
Emerald River after the dam was built, have begun
responsive and bright in tone—after a few weeks of
competing with the declining native fish species for
intense use. A researcher whose son is a classical guitarist
food and space
hypothesized that dirt and oil, rather than changes in the
material properties of the string, were responsible. (E) Five of the fish species native to the Emerald River
are not native to any other river in North America.
Which of the following investigations is most likely to
yield significant information that would help to evaluate 6. It is true that it is against international law to sell
the researcher’s hypothesis? plutonium to countries that do not yet have nuclear
(A) Determining if a metal alloy is used to make the weapons. But if United States companies do not do so,
strings used by classical guitarists companies in other countries will.
(B) Determining whether classical guitarists make their Which of the following is most like the argument above
strings go dead faster than do folk guitarists in its logical structure?
(C) Determining whether identical lengths of string, of (A) It is true that it is against the police department’s
the same gauge, go dead at different rates when policy to negotiate with kidnappers. But if the
strung on various brands of guitars police want to prevent loss of life, they must
(D) Determining whether a dead string and a new string negotiate in some cases.
produce different qualities of sound (B) It is true that it is illegal to refuse to register for
(E) Determining whether smearing various substances military service. But there is a long tradition in the
on new guitar strings causes them to go dead United States of conscientious objection to serving
in the armed forces.
4. Most consumers do not get much use out of the (C) It is true that it is illegal for a government official to
sports equipment they purchase. For example, seventeen participate in a transaction in which there is an
percent of the adults in the United States own jogging apparent conflict of interest. But if the facts are
shoes, but only forty-five percent of the owners jog more examined carefully, it will clearly be seen that there
than once a year, and only seventeen percent jog more was no actual conflict of interest in the defendant’s
than once a week. case.
Which of the following, if true, casts most doubt on the (D) It is true that it is against the law to burglarize
claim that most consumers get little use out of the people’s homes. But someone else certainly would
sports equipment they purchase? have burglarized that house if the defendant had
(A) Joggers are most susceptible to sports injuries not done so first.
during the first six months in which they jog. (E) It is true that company policy forbids supervisors
(B) Joggers often exaggerate the frequency with which to fire employees without two written warnings.
they jog in surveys designed to elicit such But there have been many supervisors who have
information. disobeyed this policy.
(C) Many consumers purchase jogging shoes for use in
7. In recent years many cabinetmakers have been
activities other than jogging.
winning acclaim as artists. But since furniture must be
(D) Consumers who take up jogging often purchase an useful, cabinetmakers must exercise their craft with an
athletic shoe that can be used in other sports. eye to the practical utility of their product. For this
(E) Joggers who jog more than once a week are often reason, cabinetmaking is not art.
active participants in other sports as well.
Which of the following is an assumption that supports
5. Two decades after the Emerald River Dam was drawing the conclusion above from the reason given for
built, none of the eight fish species native to the Emerald that conclusion?
River was still reproducing adequately in the river below (A) Some furniture is made to be placed in museums,
the dam. Since the dam reduced the annual range of water where it will not be used by anyone.
temperature in the river below the dam from 50 degrees to (B) Some cabinetmakers are more concerned than others
6 degrees, scientists have hypothesized that sharply with the practical utility of the products they
rising water temperatures must be involved in signaling produce.
the native species to begin the reproductive cycle. (C) Cabinetmakers should be more concerned with the
Which of the following statements, if true, would most practical utility of their products than they
strengthen the scientists’ hypothesis? currently are.
(A) The native fish species were still able to reproduce (D) An object is not an art object if its maker pays
only in side streams of the river below the dam attention to the object’s practical utility.
where the annual temperature range remains (E) Artists are not concerned with the monetary value
approximately 50 degrees. of their products.
(B) Before the dam was built, the Emerald River
annually overflowed its banks, creating backwaters 8. Although custom prosthetic bone replacements
that were critical breeding areas for the native produced through a new computer-aided design process
species of fish. will cost more than twice as much as ordinary
replacements, custom replacements should still be cost-
(C) The lowest recorded temperature of the Emerald
effective. Not only will surgery and recovery time be
River before the dam was built was 34 degrees,
reduced, but custom replacements should last longer,
whereas the lowest recorded temperature of the
thereby reducing the need for further hospital stays.
( 23 )
PaGaLGuYS .com Education
TM
Which of the following must be studied in order to encircling the lungs’ airways to contract. This partially
evaluate the argument presented above? seals off the lungs. An asthma attack occurs when the
(A) The amount of time a patient spends in surgery messenger molecules are activated unnecessarily, in
versus the amount of time spent recovering from response to harmless things like pollen or household dust.
surgery Which of the following, if true, points to the most
(B) The amount by which the cost of producing custom serious flaw of a plan to develop a medication that
replacements has declined with the introduction of would prevent asthma attacks by blocking receipt of
the new technique for producing them any messages sent by the messenger molecules referred
(C) The degree to which the use of custom replacements to above?
is likely to reduce the need for repeat surgery when (A) Researchers do not yet know how the body
compared with the use of ordinary replacements produces the messenger molecules that trigger
(D) The degree to which custom replacements produced asthma attacks.
with the new technique are more carefully (B) Researchers do not yet know what makes one
manufactured than are ordinary replacements person’s messenger molecules more easily
(E) The amount by which custom replacements activated than another’s.
produced with the new technique will drop in cost (C) Such a medication would not become available for
as the production procedures become standardized several years, because of long lead times in both
and applicable on a larger scale development and manufacture.
(D) Such a medication would be unable to distinguish
9. Extinction is a process that can depend on a variety
between messages triggered by pollen and
of ecological, geographical, and physiological variables.
household dust and messages triggered by noxious
These variables affect different species of organisms in
air.
different ways, and should, therefore, yield a random
pattern of extinctions. However, the fossil record shows (E) Such a medication would be a preventative only and
that extinction occurs in a surprisingly definite pattern, would be unable to alleviate an asthma attack once
with many species vanishing at the same time. it had started.

Which of the following, if true, forms the best basis for 12. Since the routine use of antibiotics can give rise to
at least a partial explanation of the patterned extinctions resistant bacteria capable of surviving antibiotic
revealed by the fossil record? environments, the presence of resistant bacteria in people
(A) Major episodes of extinction can result from could be due to the human use of prescription antibiotics.
widespread environmental disturbances that affect Some scientists, however, believe that most resistant
numerous different species. bacteria in people derive from human consumption of
(B) Certain extinction episodes selectively affect bacterially infected meat.
organisms with particular sets of characteristics Which of the following statements, if true, would most
unique to their species. significantly strengthen the hypothesis of the
(C) Some species become extinct because of scientists?
accumulated gradual changes in their local (A) Antibiotics are routinely included in livestock feed
environments. so that livestock producers can increase the rate of
(D) In geologically recent times, for which there is no growth of their animals.
fossil record, human intervention has changed the (B) Most people who develop food poisoning from
pattern of extinctions. bacterially infected meat are treated with
(E) Species that are widely dispersed are the least likely prescription antibiotics.
to become extinct. (C) The incidence of resistant bacteria in people has
tended to be much higher in urban areas than in
10. Neither a rising standard of living nor balanced rural areas where meat is of comparable quality.
trade, by itself, establishes a country’s ability to compete
(D) People who have never taken prescription
in the international marketplace. Both are required
antibiotics are those least likely to develop
simultaneously since standards of living can rise because
resistant bacteria.
of growing trade deficits and trade can be balanced by
means of a decline in a country’s standard of living. (E) Livestock producers claim that resistant bacteria in
animals cannot be transmitted to people through
If the facts stated in the passage above are true, a infected meat.
proper test of a country’s ability to be competitive is
its ability to 13. The recent decline in the value of the dollar was
(A) balance its trade while its standard of living rises triggered by a prediction of slower economic growth in
(B) balance its trade while its standard of living falls the coming year. But that prediction would not have
adversely affected the dollar had it not been for the
(C) increase trade deficits while its standard of living
government’s huge budget deficit, which must therefore
rises
be decreased to prevent future currency declines.
(D) decrease trade deficits while its standard of living
falls Which of the following, if true, would most seriously
weaken the conclusion about how to prevent future
(E) keep its standard of living constant while trade
currency declines?
deficits rise
(A) The government has made little attempt to reduce
11. Certain messenger molecules fight damage to the the budget deficit.
lungs from noxious air by telling the muscle cells (B) The budget deficit has not caused a slowdown in

( 24 )
PaGaLGuYS .com Education
TM
economic growth. above?
(C) The value of the dollar declined several times in the (A) Quality rather than price is a major factor in
year prior to the recent prediction of slower determining the type of steel to be used for a
economic growth. particular application.
(D) Before there was a large budget deficit, predictions (B) Foreign steel mills have long produced grades of
of slower economic growth frequently caused steel comparable in quality to the steel produced
declines in the dollar’s value. by the big American mills.
(E) When there is a large budget deficit, other events in (C) American quotas on imported goods have often
addition to predictions of slower economic growth induced other countries to impose similar quotas
sometimes trigger declines in currency value. on American goods.
(D) Domestic “mini-mills” consistently produce better
14. Which of the following best completes the passage
grades of steel than do the big American mills.
below?
(E) Domestic “mini-mills” produce low-volume,
At a recent conference on environmental threats to the specialized types of steels that are not produced
North Sea, most participating countries favored uniform by the big American steel mills.
controls on the quality of effluents, whether or not
specific environmental damage could be attributed to a 17. Correctly measuring the productivity of service
particular source of effluent. What must, of course, be workers is complex. Consider, for example, postal
shown, in order to avoid excessively restrictive controls, workers: they are often said to be more productive if
is that______ more letters are delivered per postal worker. But is this
(A) any uniform controls that are adopted are likely to really true? What if more letters are lost or delayed per
be implemented without delay worker at the same time that more are delivered?
(B) any substance to be made subject to controls can The objection implied above to the productivity
actually cause environmental damage measure described is based on doubts about the truth of
(C) the countries favoring uniform controls are those which of the following statements?
generating the largest quantities of effluents (A) Postal workers are representative of service
(D) all of any given pollutant that is to be controlled workers in general.
actually reaches the North Sea at present (B) The delivery of letters is the primary activity of the
(E) environmental damage already inflicted on the North postal service.
Sea is reversible (C) Productivity should be ascribed to categories of
workers, not to individuals.
15. Traditionally, decision-making by managers that is
(D) The quality of services rendered can appropriately
reasoned step-by-step has been considered preferable to
be ignored in computing productivity.
intuitive decision-making. However, a recent study found
that top managers used intuition significantly more than (E) The number of letters delivered is relevant to
did most middle- or lower-level managers. This confirms measuring the productivity of postal workers.
the alternative view that intuition is actually more 18. Male bowerbirds construct elaborately decorated
effective than careful, methodical reasoning. nests, or bowers. Basing their judgment on the fact that
The conclusion above is based on which of the different local populations of bowerbirds of the same
following assumptions? species build bowers that exhibit different building and
(A) Methodical, step-by-step reasoning is decorative styles, researchers have concluded that the
inappropriate for making many real-life bowerbirds’ building styles are a culturally acquired,
management decisions. rather than a genetically transmitted, trait.
(B) Top managers have the ability to use either intuitive Which of the following, if true, would most strengthen
reasoning or methodical, step-by-step reasoning in the conclusion drawn by the researchers?
making decisions. (A) There are more common characteristics than there
(C) The decisions made by middle- and lower-level are differences among the bower-building styles of
managers can be made as easily by using the local bowerbird population that has been
methodical reasoning as by using intuitive studied most extensively.
reasoning. (B) Young male bowerbirds are inept at bower-building
(D) Top managers use intuitive reasoning in making the and apparently spend years watching their elders
majority of their decisions. before becoming accomplished in the local bower
(E) Top managers are more effective at decision-making style.
than middle- or lower-level managers. (C) The bowers of one species of bowerbird lack the
towers and ornamentation characteristic of the
16. The imposition of quotas limiting imported steel bowers of most other species of bowerbird.
will not help the big American steel mills. In fact, the
(D) Bowerbirds are found only in New Guinea and
quotas will help “mini-mills” flourish in the United
Australia, where local populations of the birds
States. Those small domestic mills will take more
apparently seldom have contact with one another.
business from the big American steel mills than would
have been taken by the foreign steel mills in the absence (E) It is well known that the song dialects of some
of quotas. songbirds are learned rather than transmitted
genetically.
Which of the following, if true, would cast the most
serious doubt on the claim made in the last sentence 19. A greater number of newspapers are sold in Town S

( 25 )
PaGaLGuYS .com Education
TM
than in Town T. Therefore, the citizens of Town S are plantings yearly.
better informed about major world events than are the To receive governmental price-support benefits for a
citizens of Town T. crop, farmers must have produced that same crop for
Each of the following, if true, weakens the conclusion the past several years.
above EXCEPT: The statements above, if true, best support which of
(A) Town S has a larger population than Town T. the following conclusions?
(B) Most citizens of Town T work in Town S and buy (A) The rules for governmental support of farm prices
their newspapers there. work against efforts to reduce water pollution.
(C) The average citizen of Town S spends less time (B) The only solution to the problem of water pollution
reading newspapers than does the average citizen from fertilizers and pesticides is to take farmland
of Town T. out of production.
(D) A weekly newspaper restricted to the coverage of (C) Farmers can continue to make a profit by rotating
local events is published in Town S. diverse crops, thus reducing costs for chemicals,
(E) The average newsstand price of newspapers sold in but not by planting the same crop each year.
Town S is lower than the average price of (D) New farming techniques will be developed to make
newspapers sold in Town T. it possible for farmers to reduce the application of
fertilizers and pesticides.
20. One analyst predicts that Hong Kong can retain its
capitalist ways after it becomes part of mainland China in (E) Governmental price supports for farm products are
1997 as long as a capitalist Hong Kong is useful to China; set at levels that are not high enough to allow
that a capitalist Hong Kong will be useful to China as farmers to get out of debt.
long as Hong Kong is prosperous; and that Hong Kong 3. Shelby Industries manufactures and sells the same
will remain prosperous as long as it retains its capitalist gauges as Jones Industries. Employee wages account for
ways. forty percent of the cost of manufacturing gauges at both
If the predictions above are correct, which of the Shelby Industries and Jones Industries. Shelby Industries
following further predictions can logically be derived is seeking a competitive advantage over Jones Industries.
from them? Therefore, to promote this end, Shelby Industries should
(A) If Hong Kong fails to stay prosperous, it will no lower employee wages.
longer remain part of mainland China. Which of the following, if true, would most weaken the
(B) If Hong Kong retains its capitalist ways until 1997, argument above?
it will be allowed to do so afterward. (A) Because they make a small number of precision
(C) If there is a world economic crisis after 1997, it will instruments, gauge manufacturers cannot receive
not adversely affect the economy of Hong Kong. volume discounts on raw materials.
(D) Hong Kong will be prosperous after 1997. (B) Lowering wages would reduce the quality of
(E) The citizens of Hong Kong will have no restrictions employee work, and this reduced quality would
placed on them by the government of mainland lead to lowered sales.
China. (C) Jones Industries has taken away twenty percent of
Shelby Industries’ business over the last year.
TEST 8
(D) Shelby Industries pays its employees, on average,
30 Minutes 20 Questions
ten percent more than does Jones Industries.
1. A drug that is highly effective in treating many (E) Many people who work for manufacturing plants
types of infection can, at present, be obtained only from live in areas in which the manufacturing plant they
the bark of the ibora, a tree that is quite rare in the wild. work for is the only industry.
It takes the bark of 5,000 trees to make one kilogram of
the drug. It follows, therefore, that continued production 4. Some communities in Florida are populated almost
of the drug must inevitably lead to the ibora’s extinction. exclusively by retired people and contain few, if any,
families with small children. Yet these communities are
Which of the following, if true, most seriously weakens
home to thriving businesses specializing in the rental of
the argument above?
furniture for infants and small children.
(A) The drug made from ibora bark is dispensed to
doctors from a central authority. Which of the following, if true, best reconciles the
seeming discrepancy described above?
(B) The drug made from ibora bark is expensive to
produce. (A) The businesses specializing in the rental of
children’s furniture buy their furniture from
(C) The leaves of the ibora are used in a number of
distributors outside of Florida.
medical products.
(B) The few children who do reside in these
(D) The ibora can be propagated from cuttings and
communities all know each other and often make
grown under cultivation.
overnight visits to one another’s houses.
(E) The ibora generally grows in largely inaccessible
(C) Many residents of these communities who move
places.
frequently prefer renting their furniture to buying
2. High levels of fertilizer and pesticides, needed when it outright.
farmers try to produce high yield of the same crop year (D) Many residents of these communities must provide
after year, pollute water supplies. Experts therefore urge for the needs of visiting grandchildren several
farmers to diversify their crops and to rotate their weeks a year.

( 26 )
PaGaLGuYS .com Education
TM
(E) Children’s furniture available for rental is of the are removed during processing.
same quality as that available for sale in the stores. (D) Unprocessed cereals are naturally high in several of
the vitamins that are usually added to fortified
5. Large national budget deficits do not cause large
breakfast cereals.
trade deficits. If they did, countries with the largest
budget deficits would also have the largest trade deficits. (E) Cereals containing vitamin supplements are no
In fact, when deficit figures are adjusted so that different harder to digest than similar cereals without added
countries are reliably comparable to each other, there is vitamins.
no such correlation. 8. Which of the following best completes the passage
If the statements above are all true, which of the below?
following can properly be inferred on the basis of them? The more worried investors are about losing their
(A) Countries with large national budget deficits tend to money, the more they will demand a high potential
restrict foreign trade. return on their investment; great risks must be offset by
(B) Reliable comparisons of the deficit figures of one the chance of great rewards. This principle is the
country with those of another are impossible. fundamental one in determining interest rates, and it is
(C) Reducing a country’s national budget deficit will not illustrated by the fact that______
necessarily result in a lowering of any trade deficit (A) successful investors are distinguished by an ability
that country may have. to make very risky investments without worrying
(D) When countries are ordered from largest to smallest about their money
in terms of population, the smallest countries (B) lenders receive higher interest rates on unsecured
generally have the smallest budget and trade loans than on loans backed by collateral
deficits. (C) in times of high inflation, the interest paid to
(E) Countries with the largest trade deficits never have depositors by banks can actually be below the rate
similarly large national budget deficits. of inflation
(D) at any one time, a commercial bank will have a
6. “Fast cycle time” is a strategy of designing a
single rate of interest that it will expect all of its
manufacturing organization to eliminate bottlenecks and
individual borrowers to pay
delays in production. Not only does it speed up
production, but it also assures quality. The reason is that (E) the potential return on investment in a new
the bottlenecks and delays cannot be eliminated unless all company is typically lower than the potential
work is done right the first time. return on investment in a well-established
company
The claim about quality made above rests on a
questionable presupposition that 9. A famous singer recently won a lawsuit against an
(A) any flaw in work on a product would cause a advertising firm for using another singer in a commercial
bottleneck or delay and so would be prevented to evoke the famous singer’s well-known rendition of a
from occurring on a “fast cycle” production line certain song. As a result of the lawsuit, advertising firms
(B) the strategy of “fast cycle time” would require will stop using imitators in commercials. Therefore,
fundamental rethinking of product design advertising costs will rise, since famous singers’ services
cost more than those of their imitators.
(C) the primary goal of the organization is to produce a
product of unexcelled quality, rather than to The conclusion above is based on which of the
generate profits for stockholders following assumptions?
(D) “fast cycle time” could be achieved by shaving time (A) Most people are unable to distinguish a famous
off each of the component processes in production singer’s rendition of a song from a good imitator’s
cycle rendition of the same song.
(E) “fast cycle time” is a concept in business strategy (B) Commercials using famous singers are usually more
that has not yet been put into practice in a factory effective than commercials using imitators of
famous singers.
7. Many breakfast cereals are fortified with vitamin (C) The original versions of some well-known songs are
supplements. Some of these cereals provide 100 percent unavailable for use in commercials.
of the recommended daily requirement of vitamins.
(D) Advertising firms will continue to use imitators to
Nevertheless, a well-balanced breakfast, including a
mimic the physical mannerisms of famous singers.
variety of foods, is a better source of those vitamins than
are such fortified breakfast cereals alone. (E) The advertising industry will use well-known
renditions of songs in commercials.
Which of the following, if true, would most strongly
support the position above? 10. A certain mayor has proposed a fee of five dollars
(A) In many foods, the natural combination of vitamins per day on private vehicles entering the city, claiming that
with other nutrients makes those vitamins more the fee will alleviate the city’s traffic congestion. The
usable by the body than are vitamins added in mayor reasons that, since the fee will exceed the cost of
vitamin supplements. round-trip bus fare from many nearby points, many
(B) People who regularly eat cereals fortified with people will switch from using their cars to using the bus.
vitamin supplements sometimes neglect to eat the Which of the following statements, if true, provides the
foods in which the vitamins occur naturally. best evidence that the mayor’s reasoning is flawed?
(C) Foods often must be fortified with vitamin (A) Projected increases in the price of gasoline will
supplements because naturally occurring vitamins increase the cost of taking a private vehicle into the

( 27 )
PaGaLGuYS .com Education
TM
city. successful, the wholesale price of most illegal drugs would
(B) The cost of parking fees already makes it not have dropped substantially in 1987.
considerably more expensive for most people to
13. The argument in the passage depends on which of
take a private vehicle into the city than to take a
the following assumptions?
bus.
(A) The supply of illegal drugs dropped substantially in
(C) Most of the people currently riding the bus do not
1987.
own private vehicles.
(B) The price paid for most illegal drugs by the average
(D) Many commuters opposing the mayor’s plan have
consumer did not drop substantially in 1987.
indicated that they would rather endure traffic
congestion than pay a five-dollar-per day fee. (C) Domestic production of illegal drugs increased at a
higher rate than did the entry of such drugs into the
(E) During the average workday, private vehicles owned
country.
and operated by people living within the city
account for twenty percent of the city’s traffic (D) The wholesale price of a few illegal drugs increased
congestion. substantially in 1987.
(E) A drop in demand for most illegal drugs in 1987 was
11. A group of children of various ages was read stories not the sole cause of the drop in their wholesale
in which people caused harm, some of those people doing price.
so intentionally, and some accidentally. When asked about
appropriate punishments for those who had caused harm, 14. The argument in the passage would be most
the younger children, unlike the older ones, assigned seriously weakened if it were true that
punishments that did not vary according to whether the (A) in 1987 smugglers of illegal drugs, as a group, had
harm was done intentionally or accidentally. Younger significantly more funds at their disposal than did
children, then, do not regard people’s intentions as the country’s customs agents
relevant to punishment. (B) domestic production of illegal drugs increased
Which of the following, if true, would most seriously substantially in 1987
weaken the conclusion above? (C) the author’s statements were made in order to
(A) In interpreting these stories, the listeners had to embarrass the officials responsible for the drug-
draw on a relatively mature sense of human control program
psychology in order to tell whether harm was (D) in 1987 illegal drugs entered the country by a
produced intentionally or accidentally. different set of routes than they did in 1986
(B) In these stories, the severity of the harm produced (E) the country’s citizens spent substantially more
was clearly stated. money on illegal drugs in 1987 than they did in
(C) Younger children are as likely to produce harm 1986
unintentionally as are older children.
15. Excavation of the ancient city of Kourion on the
(D) The older children assigned punishment in a way
island of Cyprus revealed a pattern of debris and
that closely resembled the way adults had assigned
collapsed buildings typical of towns devastated by
punishment in a similar experiment
earthquakes. Archaeologists have hypothesized that the
(E) The younger children assigned punishments that destruction was due to a major earthquake known to have
varied according to the severity of the harm done occurred near the island in A.D. 365.
by the agents in the stories.
Which of the following, if true, most strongly supports
12. When hypnotized subjects are told that they are the archaeologists’ hypothesis?
deaf and are then asked whether they can hear the (A) Bronze ceremonial drinking vessels that are often
hypnotist, they reply, “No.” Some theorists try to found in graves dating from years preceding and
explain this result by arguing that the selves of following A.D. 365 were also found in several
hypnotized subjects are dissociated into separate parts, graves near Kourion.
and that the part that is deaf is dissociated from the part (B) No coins minted after A.D. 365 were found in
that replies. Kourion, but coins minted before that year were
Which of the following challenges indicates the most found in abundance.
serious weakness in the attempted explanation (C) Most modern histories of Cyprus mention that an
described above? earthquake occurred near the island in A.D. 365.
(A) Why does the part that replies not answer, “Yes”? (D) Several small statues carved in styles current in
(B) Why are the observed facts in need of any special Cyprus in the century between A.D. 300 and 400
explanation? were found in Kourion.
(C) Why do the subjects appear to accept the (E) Stone inscriptions in a form of the Greek alphabet
hypnotist’s suggestion that they are deaf? that was definitely used in Cyprus after A.D. 365
(D) Why do hypnotized subjects all respond the same were found in Kourion.
way in the situation described?
16. Sales of telephones have increased dramatically over
(E) Why are the separate parts of the self the same for the last year. In order to take advantage of this increase,
all subjects? Mammoth Industries plans to expand production of its
own model of telephone, while continuing its already
Questions 13-14 are based on the following.
very extensive advertising of this product.
The program to control the entry of illegal drugs into the Which of the following, if true, provides most support
country was a failure in 1987. If the program had been
( 28 )
PaGaLGuYS .com Education
TM
for the view that Mammoth Industries cannot increase prefer using common land over using private land
its sales of telephones by adopting the plan outlined for grazing?
above? (C) Was the private land that was studied of comparable
(A) Although it sells all of the telephones that it quality to the common land before either was used
produces, Mammoth Industries’ share of all for grazing?
telephone sales has declined over the last year. (D) Were the users of the common land that was studied
(B) Mammoth Industries’ average inventory of at least as prosperous as the users of the private
telephones awaiting shipment to retailers has land?
declined slightly over the last year. (E) Were there any owners of herds who used only
(C) Advertising has made the brand name of Mammoth common land, and no private land, for grazing?
Industries’ telephones widely known, but few
consumers know that Mammoth Industries owns 19. Which of the following, if true, and known by the
this brand. ranchers, would best help explain the results of the
study?
(D) Mammoth Industries’ telephone is one of three
brands of telephone that have together accounted (A) With private grazing land, both the costs and the
for the bulk of the last year’s increase in sales. benefits of overuse fall to the individual user.
(E) Despite a slight decline in the retail price, sales of (B) The cost in reduced land quality that is attributable
Mammoth Industries’ telephones have fallen in the to any individual user is less easily measured with
last year. common land than it is with private land.
(C) An individual who overuses common grazing land
17. Many institutions of higher education suffer might be able to achieve higher returns than other
declining enrollments during periods of economic users can, with the result that he or she would
slowdown. At two-year community colleges, however, obtain a competitive advantage.
enrollment figures boom during these periods when many (D) If one user of common land overuses it even
people have less money and there is more competition for slightly, the other users are likely to do so even
jobs. more, with the consequence that the costs to each
Each of the following, if true, helps to explain the user outweigh the benefits.
enrollment increases in two-year community colleges (E) There are more acres of grazing land held privately
described above EXCEPT: than there are held in common.
(A) During periods of economic slowdown, two-year
community colleges are more likely than four-year 20. In tests for pironoma, a serious disease, a false
colleges to prepare their students for the jobs that positive result indicates that people have pironoma when,
are still available. in fact, they do not; a false negative result indicates that
people do not have pironoma when, in fact, they do. To
(B) During periods of economic prosperity, graduates of
detect pironoma most accurately, physicians should use
two-year community colleges often continue their
the laboratory test that has the lowest proportion of false
studies at four-year colleges.
positive results.
(C) Tuition at most two-year community colleges is a
fraction of that at four-year colleges. Which of the following, if true, gives the most support
to the recommendation above?
(D) Two-year community colleges devote more
resources than do other colleges to attracting those (A) The accepted treatment for pironoma does not have
students especially affected by economic damaging side effects.
slowdowns. (B) The laboratory test that has the lowest proportion
(E) Students at two-year community colleges, but not of false positive results causes the same minor side
those at most four-year colleges, can control the effects as do the other laboratory tests used to
cost of their studies by choosing the number of detect pironoma.
courses they take each term. (C) In treating pironoma patients, it is essential to begin
treatment as early as possible, since even a week of
Questions 18-19 are based on the following. delay can result in loss of life.
Hardin argued that grazing land held in common (that is, open (D) The proportion of inconclusive test results is equal
to any user) would always be used less carefully than private for all laboratory tests used to detect pironoma.
grazing land. Each rancher would be tempted to overuse (E) All laboratory tests to detect pironoma have the
common land because the benefits would accrue to the same proportion of false negative results.
individual, while the costs of reduced land quality that results
TEST 9
from overuse would be spread among all users. But a study
30 Minutes 20 Questions
comparing 217 million acres of common grazing land with 433
million acres of private grazing land showed that the common Questions 1-2 are based on the following.
land was in better condition.
Companies O and P each have the same number of employees
18. The answer to which of the following questions who work the same number of hours per week. According to
would be most useful in evaluating the significance, in records maintained by each company, the employees of
relation to Hardin’s claim, of the study described above? Company O had fewer job-related accidents last year than did
(A) Did any of the ranchers whose land was studied use the employees of Company P. Therefore, employees of
both common and private land? Company O are less likely to have job-related accidents than
(B) Did the ranchers whose land was studied tend to are employees of Company P.

( 29 )
PaGaLGuYS .com Education
TM
1. Which of the following, if true, would most 4. Which of the following, if true, would best support
strengthen the conclusion above? the conclusion that some ingredient of the sweetener was
(A) Company P manufactures products that are more responsible for the experimental results?
hazardous for workers to produce than does (A) Most consumers of the sweetener do not consume
Company O. as much of it as the experimental group members
(B) Company P holds more safety inspections than did.
does Company O. (B) The amino acid referred to in the conclusion is a
(C) Company P maintains a more modern infirmary component of all proteins, some of which must be
than does Company O. consumed for adequate nutrition.
(D) Company O paid more for new job-related medical (C) The quantity of the sweetener consumed by
claims than did Company P. individuals in the experimental group is considered
(E) Company P provides more types of health-care safe by federal food regulators.
benefits than does Company O. (D) The two groups of subjects were evenly matched
with regard to cognitive abilities prior to the
2. Which of the following, if true, would most weaken experiment.
the conclusion above? (E) A second experiment in which subjects consumed
(A) The employees of Company P lost more time at large quantities of the sweetener lacked a control
work due to job-related accidents than did the group of subjects who were not given the
employees of Company O. sweetener.
(B) Company P considered more types of accidents to
be job-related than did Company O. 5. Which of the following, if true, would best help
explain how the sweetener might produce the observed
(C) The employees of Company P were sick more often
effect?
than were the employees of Company O.
(A) The government’s analysis of the artificial
(D) Several employees of Company O each had more
sweetener determined that it was sold in relatively
than one job-related accident.
pure form.
(E) The majority of job-related accidents at Company O
(B) A high level of the amino acid in the blood inhibits
involved a single machine.
the synthesis of a substance required for normal
3. In comparison to the standard typewriter keyboard, brain functioning.
the EFCO keyboard, which places the most-used keys (C) Because the sweetener is used primarily as a food
nearest the typist’s strongest fingers, allows faster typing additive, adverse reactions to it are rarely noticed
and results in less fatigue, Therefore, replacement of by consumers.
standard keyboards with the EFCO keyboard will result (D) The amino acid that is a constituent of the
in an immediate reduction of typing costs. sweetener is also sold separately as a dietary
Which of the following, if true, would most weaken the supplement.
conclusion drawn above? (E) Subjects in the experiment did not know whether
(A) People who use both standard and EFCO they were consuming the sweetener or a second,
keyboards report greater difficulty in the transition harmless substance.
from the EFCO keyboard to the standard keyboard
6. Adult female rats who have never before
than in the transition from the standard keyboard
encountered rat pups will start to show maternal
to the EFCO keyboard.
behaviors after being confined with a pup for about seven
(B) EFCO keyboards are no more expensive to days. This period can be considerably shortened by
manufacture than are standard keyboards and disabling the female’s sense of smell or by removing the
require less frequent repair than do standard scent-producing glands of the pup.
keyboards.
Which of the following hypotheses best explains the
(C) The number of businesses and government agencies
contrast described above?
that use EFCO keyboards is increasing each year.
(A) The sense of smell in adult female rats is more acute
(D) The more training and experience an employee has
than that in rat pups.
had with the standard keyboard, the more costly it
is to train that employee to use the EFCO (B) The amount of scent produced by rat pups
keyboard. increases when they are in the presence of a female
rat that did not bear them.
(E) Novice typists can learn to use the EFCO keyboard
in about the same amount of time it takes them to (C) Female rats that have given birth are more affected
learn to use the standard keyboard. by olfactory cues than are female rats that have
never given birth.
Questions 4-5 are based on the following. (D) A female rat that has given birth shows maternal
behavior toward rat pups that she did not bear
Half of the subjects in an experiment—the experimental more quickly than does a female rat that has never
group—consumed large quantities of a popular artificial given birth.
sweetener. Afterward, this group showed lower cognitive
(E) The development of a female rat’s maternal interest
abilities than did the other half of the subjects—the control
in a rat pup that she did not bear is inhibited by
group—who did not consume the sweetener. The detrimental
the odor of the pup.
effects were attributed to an amino acid that is one of the
sweetener’s principal constituents. 7. The interview is an essential part of a successful
hiring program because, with it, job applicants who have
( 30 )
PaGaLGuYS .com Education
TM
personalities that are unsuited to the requirements of the (E) Determining the amount of time each nonprotein
job will be eliminated from consideration. drug takes to reach its target cells
The argument above logically depends on which of the 10. Country Y uses its scarce foreign-exchange reserves
following assumptions? to buy scrap iron for recycling into steel. Although the
(A) A hiring program will be successful if it includes steel thus produced earns more foreign exchange than it
interviews. costs, that policy is foolish. Country Y’s own territory
(B) The interview is a more important part of a has vast deposits of iron ore, which can be mined with
successful hiring program than is the development minimal expenditure of foreign exchange.
of a job description. Which of the following, if true, provides the strongest
(C) Interviewers can accurately identify applicants support for Country Y’s policy of buying scrap iron
whose personalities are unsuited to the abroad?
requirements of the job. (A) The price of scrap iron on international markets
(D) The only purpose of an interview is to evaluate rose significantly in 1987.
whether job applicants’ personalities are suited to (B) Country Y’s foreign-exchange reserves dropped
the requirements of the job. significantly in 1987.
(E) The fit of job applicants’ personalities to the (C) There is virtually no difference in quality between
requirements of the job was once the most steel produced from scrap iron and that produced
important factor in making hiring decisions. from iron ore.
8. An overly centralized economy, not the changes in (D) Scrap iron is now used in the production of roughly
the climate, is responsible for the poor agricultural half the steel used in the world today, and experts
production in Country X since its new government came predict that scrap iron will be used even more
to power. Neighboring Country Y has experienced the extensively in the future.
same climatic conditions, but while agricultural (E) Furnaces that process scrap iron can be built and
production has been falling in Country X, it has been operated in Country Y with substantially less
rising in Country Y. foreign exchange than can furnaces that process
Which of the following, if true, would most weaken the iron ore.
argument above? 11. Last year the rate of inflation was 1.2 percent, but
(A) Industrial production also is declining in Country for the current year it has been 4 percent. We can
X. conclude that inflation is on an upward trend and the rate
(B) Whereas Country Y is landlocked, Country X has a will be still higher next year.
major seaport. Which of the following, if true, most seriously weakens
(C) Both Country X and Country Y have been the conclusion above?
experiencing drought conditions. (A) The inflation figures were computed on the basis of
(D) The crops that have always been grown in Country a representative sample of economic data rather
X are different from those that have always been than all of the available data.
grown in Country Y. (B) Last year a dip in oil prices brought inflation
(E) Country X’s new government instituted a temporarily below its recent stable annual level of
centralized economy with the intention of ensuring 4 percent.
an equitable distribution of goods. (C) Increases in the pay of some workers are tied to the
9. Useful protein drugs, such as insulin, must still be level of inflation, and at an inflation rate of 4
administered by the cumbersome procedure of injection percent or above, these pay raises constitute a
under the skin. If proteins are taken orally, they are force causing further inflation.
digested and cannot reach their target cells. Certain (D) The 1.2 percent rate of inflation last year
nonprotein drugs, however, contain chemical bonds that represented a ten-year low.
are not broken down by the digestive system. They can, (E) Government intervention cannot affect the rate of
thus, be taken orally. inflation to any significant degree.
The statements above most strongly support a claim 12. Because no employee wants to be associated with
that a research procedure that successfully bad news in the eyes of a superior, information about
accomplishes which of the following would be beneficial serious problems at lower levels is progressively softened
to users of protein drugs? and distorted as it goes up each step in the management
(A) Coating insulin with compounds that are broken hierarchy. The chief executive is, therefore, less well
down by target cells, but whose chemical bonds are informed about problems at lower levels than are his or
resistant to digestion her subordinates at those levels.
(B) Converting into protein compounds, by procedures The conclusion drawn above is based on the assumption
that work in the laboratory, the nonprotein drugs that
that resist digestion
(A) problems should be solved at the level in the
(C) Removing permanently from the digestive system management hierarchy at which they occur
any substances that digest proteins
(B) employees should be rewarded for accurately
(D) Determining, in a systematic way, what enzymes reporting problems to their superiors
and bacteria are present in the normal digestive
(C) problem-solving ability is more important at higher
system and whether they tend to be broken down
levels than it is at lower levels of the management
within the body
( 31 )
PaGaLGuYS .com Education
TM
hierarchy (C) If funding for the new police officers’ salaries is
(D) chief executives obtain information about problems approved, support for other city services will have
at lower levels from no source other than their to be reduced during the next fiscal year.
subordinates (D) In most United States cities, not all arrests result in
(E) some employees are more concerned about truth convictions, and not all convictions result in prison
than about the way they are perceived by their terms.
superiors (E) Middletown’s ratio of police officers to citizens has
reached a level at which an increase in the number
13. In the United States in 1986, the average rate of of officers will have a deterrent effect on crime.
violent crime in states with strict gun-control laws was
645 crimes per 100,000 persons—about 50 percent 16. A recent report determined that although only three
higher than the average rate in the eleven states where percent of drivers on Maryland highways equipped their
strict gun-control laws have never been passed. Thus one vehicles with radar detectors, thirty-three percent of all
way to reduce violent crime is to repeal strict gun control vehicles ticketed for exceeding the speed limit were
laws. equipped with them. Clearly, drivers who equip their
Which of the following, if true, would most weaken the vehicles with radar detectors are more likely to exceed the
argument above? speed limit regularly than are drivers who do not.
(A) The annual rate of violent crime in states with strict The conclusion drawn above depends on which of the
gun-control laws has decreased since the passage of following assumptions?
those laws. (A) Drivers who equip their vehicles with radar
(B) In states with strict gun-control laws, few detectors are less likely to be ticketed for exceeding
individuals are prosecuted for violating such laws. the speed limit than are drivers who do not.
(C) In states without strict gun-control laws, many (B) Drivers who are ticketed for exceeding the speed
individuals have had no formal training in the use limit are more likely to exceed the speed limit
of firearms. regularly than are drivers who are not ticketed.
(D) The annual rate of nonviolent crime is lower in (C) The number of vehicles that were ticketed for
states with strict gun-control laws than in states exceeding the speed limit was greater than the
without such laws. number of vehicles that were equipped with radar
(E) Less than half of the individuals who reside in states detectors.
without strict gun-control laws own a gun. (D) Many of the vehicles that were ticketed for
exceeding the speed limit were ticketed more than
14. Corporate officers and directors commonly buy and once in the time period covered by the report.
sell, for their own portfolios, stock in their own (E) Drivers on Maryland highways exceeded the speed
corporations. Generally, when the ratio of such inside limit more often than did drivers on other state
sales to inside purchases falls below 2 to 1 for a given highways not covered in the report.
stock, a rise in stock prices is imminent. In recent days,
while the price of MEGA Corporation stock has been 17. There is a great deal of geographical variation in the
falling, the corporation’s officers and directors have frequency of many surgical procedures—up to tenfold
bought up to nine times as much of it as they have sold. variation per hundred thousand between different areas in
The facts above best support which of the following the numbers of hysterectomies, prostatectomies, and
predictions? tonsillectomies.
(A) The imbalance between inside purchases and inside To support a conclusion that much of the variation is
sales of MEGA stock will grow even further. due to unnecessary surgical procedures, it would be
(B) Inside purchases of MEGA stock are about to cease most important to establish which of the following?
abruptly. (A) A local board of review at each hospital examines
(C) The price of MEGA stock will soon begin to go up. the records of every operation to determine
whether the surgical procedure was necessary.
(D) The price of MEGA stock will continue to drop,
but less rapidly. (B) The variation is unrelated to factors (other than the
surgical procedures themselves) that influence the
(E) The majority of MEGA stock will soon be owned
incidence of diseases for which surgery might be
by MEGA’s own officers and directors.
considered.
15. The proposal to hire ten new police officers in (C) There are several categories of surgical procedure
Middletown is quite foolish. There is sufficient funding (other than hysterectomies, prostatectomies, and
to pay the salaries of the new officers, but not the tonsillectomies) that are often performed
salaries of additional court and prison employees to unnecessarily.
process the increased caseload of arrests and convictions (D) For certain surgical procedures, it is difficult to
that new officers usually generate. determine after the operation whether the
Which of the following, if true, will most seriously procedures were necessary or whether alternative
weaken the conclusion drawn above? treatment would have succeeded.
(A) Studies have shown that an increase in a city’s (E) With respect to how often they are performed
police force does not necessarily reduce crime. unnecessarily, hysterectomies, prostatectomies,
and tonsillectomies are representative of surgical
(B) When one major city increased its police force by
procedures in general.
19 percent last year, there were 40 percent more
arrests and 13 percent more convictions. 18. Researchers have found that when very overweight
( 32 )
PaGaLGuYS .com Education
TM
people, who tend to have relatively low metabolic rates, theoretically parasitize can be determined from the
lose weight primarily through dieting, their metabolisms wasp’s egg-laying behavior.
generally remain unchanged. They will thus burn (B) Host insects lack any effective defenses against the
significantly fewer calories at the new weight than do form of predation practiced by parasitic wasps.
people whose weight is normally at that level. Such (C) Parasitic wasps learn from experience how many
newly thin persons will, therefore, ultimately regain eggs to lay into the eggs of different host species.
weight until their body size again matches their metabolic
(D) Failure to lay enough eggs would lead to the death
rate.
of the developing wasp larvae more quickly than
The conclusion of the argument above depends on would laying too many eggs.
which of the following assumptions? (E) Parasitic wasps use visual clues to calculate the size
(A) Relatively few very overweight people who have of a host egg.
dieted down to a new weight tend to continue to
consume substantially fewer calories than do TEST 10
people whose normal weight is at that level. 30 Minutes 20 Questions
(B) The metabolisms of people who are usually not 1. In 1985 in the country of Alissia, farmers brought
overweight are much more able to vary than the to market a broccoli crop that was one-and-a-half times as
metabolisms of people who have been very large as the 1985 broccoli crop in its neighbor country,
overweight. Barbera. Yet total quantities of broccoli available for sale
(C) The amount of calories that a person usually burns to consumers in Alissia were smaller than were total
in a day is determined more by the amount that is quantities in Barbera in 1985.
consumed that day than by the current weight of
Which of the following, if true, in 1985, contributes
the individual.
most to an explanation of why there was less broccoli
(D) Researchers have not yet determined whether the available for sale to consumers in Alissia than in
metabolic rates of formerly very overweight Barbera?
individuals can be accelerated by means of chemical
(A) Barbera’s farmers produced much more cabbage
agents.
than did Alissia’s farmers.
(E) Because of the constancy of their metabolic rates,
(B) Barbera’s farmers produced fewer heads of broccoli
people who are at their usual weight normally have
per acre than did Alissia’s farmers.
as much difficulty gaining weight as they do losing
it. (C) Alissia exported a much higher proportion of its
broccoli crop than did Barbera.
19. In 1987 sinusitis was the most common chronic (D) Broccoli was much more popular among consumers
medical condition in the United States, followed by in Alissia than in Barbera.
arthritis and high blood pressure, in that order. (E) Alissia had more land suitable for growing broccoli
The incidence rates for both arthritis and high blood than did Barbera.
pressure increase with age, but the incidence rate for
sinusitis is the same for people of all ages. 2. A manufacturer of men’s dress socks sought to
increase profits by increasing sales. The size of its
The average age of the United States population will customer pool was remaining steady, with the average
increase between 1987 and 2000. customer buying twelve pairs of dress socks per year.
Which of the following conclusions can be most The company’s plan was to increase the number of
properly drawn about chronic medical conditions in the promotional discount-sale periods to one every six
United States from the information given above? months.
(A) Sinusitis will be more common than either arthritis Which of the following, if it is a realistic possibility,
or high blood pressure in 2000. casts the most serious doubt on the viability of the
(B) Arthritis will be the most common chronic medical company’s plan?
condition in 2000. (A) New manufacturing capacity would not be required
(C) The average age of people suffering from sinusitis if the company were to increase the number of
will increase between 1987 and 2000. pairs of socks sold.
(D) Fewer people will suffer from sinusitis in 2000 than (B) Inventory stocks of merchandise ready for sale
suffered from it in 1987. would be high preceding the increase in the number
(E) A majority of the population will suffer from at of discount-sale periods.
least one of the medical conditions mentioned (C) The manufacturer’s competitors would match its
above by the year 2000. discounts during sale periods, and its customers
would learn to wait for those times to make their
20. Parasitic wasps lay their eggs directly into the eggs purchases.
of various host insects in exactly the right numbers for (D) New styles and colors would increase customers’
any suitable size of host egg. If they laid too many eggs consciousness of fashion in dress socks, but the
in a host egg, the developing wasp larvae would compete customers’ requirements for older styles and colors
with each other to the death for nutrients and space. If would not be reduced.
too few eggs were laid, portions of the host egg would
(E) The cost of the manufacturer’s raw materials would
decay, killing the wasp larvae.
remain steady, and its customers would have more
Which of the following conclusions can properly be disposable income.
drawn from the information above?
(A) The size of the smallest host egg that a wasp could 3. Previous studies have indicated that eating
( 33 )
PaGaLGuYS .com Education
TM
chocolate increases the likelihood of getting heart disease. stockholders’ investments in the company will
However, a new, more reliable study has indicated that increase from year to year
eating chocolate does not increase the likelihood of getting (C) profits rarely exceed a ten percent return on
heart disease. When the results of the new study become stockholders’ investments in the company
known, consumption of chocolate will undoubtedly (D) profits in excess of a ten percent return on
increase. stockholders’ investments in the company are all
Which of the following is an assumption on which the distributed in the form of bonuses
conclusion above is based? (E) bonuses at DSR never drop to zero
(A) Most people who eat a great deal of chocolate will
not get heart disease. Questions 6-7 are based on the following.
(B) Although they believe that eating chocolate Suitable habitats for gray wolves have greatly diminished in
increases the likelihood of getting heart disease, area. In spite of this fact, the most sensible course would be
some people still eat as much chocolate as they to refrain from reestablishing gray wolves in places where
want. previously they have been hunted out of existence. Striving to
(C) People who have heard that eating chocolate bring back these animals to places where they will only face
increases the likelihood of getting heart disease do lethal human hostility is immoral.
not believe it.
(D) There are people who currently eat as much 6. The argument above depends on
chocolate as they want because they have not (A) an appeal to an authority
heard that eating chocolate increases the likelihood (B) a belief that gray wolves are dangerous to human
of getting heart disease. beings and livestock
(E) There are people who currently limit their (C) an assumption that two events that occur together
consumption of chocolate only because they must be causally connected
believe that eating chocolate increases the (D) an assumption that the future will be like the past
likelihood of getting heart disease. (E) a threat of violence against those persons presenting
4. The fossil record shows that the climate of North the opposing view
America warmed and dried at the end of the Pleistocene
7. The argument above would be most significantly
period. Most of the species of large mammals then living
weakened if which of the following were true?
on the continent became extinct, but the smaller
mammalian species survived. (A) Effective laws against the hunting of gray wolves
have been enacted.
Which of the following, if true, provides the best basis (B) Ranchers, farmers, and hunters still have an
for an explanation of the contrast described above ingrained bias against gray wolves.
between species of large mammals and species of small
(C) By the 1930’s bounty hunters had exterminated
mammals?
most of the gray wolves in the United States.
(A) Individual large mammals can, in general, travel
further than small mammals and so are more able to (D) Programs for increasing the gray wolf population
migrate in search of a hospitable environment. are not aided by federal laws that require the
licensing of hunters of certain predators.
(B) The same pattern of comparative success in smaller,
as opposed to larger, species that is observed in (E) Suggested programs for increasing the gray wolf
mammals is also found in bird species of the same population have been criticized by
period. environmentalists and biologists.
(C) The fossil record from the end of Pleistocene period 8. For the safety-conscious Swedish market, a United
is as clear for small mammals as it is for large States manufacturer of desktop computers developed a
mammals. special display screen that produces a much weaker
(D) Larger mammals have greater food and space electromagnetic field surrounding the user than do
requirements than smaller mammals and are thus ordinary screens. Despite an advantage in this respect
less able to withstand environmental change. over its competitors, the manufacturer is introducing the
(E) Many more of the species of larger mammals than screen into the United States market without advertising
of the species of smaller mammals living in North it as a safety improvement.
America in that period had originated in climates Which of the following, if true, provides a rationale for
that were warmer than was that of North America the manufacturer’s approach to advertising the screen in
before the end of the Pleistocene period. the United States?
(A) Many more desktop computers are sold each year
5. Bonuses at DSR Industries cannot be awarded
in the United States market than are sold in the
unless profits exceed a ten percent return on
Swedish market.
stockholders’ investments in the company. Higher profits
mean higher bonuses. Therefore, bonuses in a year of (B) The manufacturer does not want its competitors to
general economic recession will be considerably lower become aware of the means by which the company
than bonuses in a year of peak profits at DSR. has achieved this advance in technology.
The conclusion above depends on the assumption that (C) Most business and scientific purchasers of desktop
computers expect to replace such equipment
(A) the firm will have relatively low profits in recession eventually as better technology becomes available
years on the market.
(B) the amount represented by a ten percent return on (D) An emphasis on the comparative safety of the new
( 34 )
PaGaLGuYS .com Education
TM
screen would call into question the safety of the of wildland left.
many screens the manufacturer has already sold in Which of the following, if true, most significantly
the United States. weakens the argument of the opponents of conservation
(E) Concern has been expressed in the United States efforts?
over the health effects of the large electromagnetic (A) As much, if not more, effort is required to restore a
fields surrounding electric power lines. wild habitat as to preserve an intact habitat.
9. In the suburbs surrounding Middletown, there is an (B) The opponents of restoration efforts are, for the
average of 2.4 automobiles per family, and thus very few most part, members of the wealthier classes in
suburban residents use public buses. The suburban their own villages and cities.
communities, therefore, would derive little benefit from (C) Existing conservation laws have been very effective
continuing to subsidize the portion of Middletown’s in preserving biological diversity within the
public bus system that serves the suburbs. wildlands that remain intact.
Which of the following, if true, casts the most serious (D) For many tropical species native to that country,
doubt on the conclusion drawn above? the tropical wildlands that are still relatively intact
(A) The real-estate tax rate in Middletown is higher do not provide appropriate habitats for
than it is in the suburbs. reproduction.
(B) Last year voters in the suburban communities (E) If a suitable population of plants and animals is
defeated by a narrow margin a bill designed to introduced and is permitted to disperse and grow,
increase subsidies for public bus routes. tropical habitats can most certainly be restored.
(C) Many suburban shops can attract enough 12. A study comparing a group of chronically
employees to remain in business only because depressed individuals with an otherwise matched group
subsidized public transportation from Middletown of individuals free from depression found significantly
is available. more disorders of the immune system among the
(D) Public buses operated with less than a 35 percent depressed group. According to the researchers, these
occupancy rate produce more pollution per results strongly support the hypothesis that mental
passenger mile than would the operation of private states influence the body’s vulnerability to infection.
automobiles for each passenger. Which of the following, if true, casts the most serious
(E) Most voters in Middletown’s suburban doubt on the researchers’ interpretation of their
communities are unwilling to continue subsidies for findings?
public buses next year if ridership on those buses (A) The researchers’ view does little more than echo a
drops below current levels. familiar theme in folklore and literature.
10. Any tax relief received by the solar industry would (B) Chronically depressed individuals are no less careful
not benefit the homeowner who installs a solar-energy than others to avoid exposure to infections.
system. Even though homeowners would pay a lower (C) Disorders of the immune system cause many of
price for solar-energy system installations because of this those individuals who have them to become
tax relief, with the government paying the balance, chronically depressed.
government revenues come from the public. (D) Individuals who have previously been free from
The argument above is based on which of the following depression can become depressed quite suddenly.
assumptions? (E) A high frequency of infections can stem from an
(A) The tax relief would cause the homeowner to lose, unusually high level of exposure rather than from
through taxes or reduced government benefits or any disorder of the immune system.
both, an amount at least equal to the reduction in
13. Exports of United States wood pulp will rise
the price of that homeowner’s solar-energy system
considerably during this year. The reason for the rise is
installation.
that the falling value of the dollar will make it cheaper for
(B) The tax relief that would be received by solar- paper manufacturers in Japan and Western Europe to buy
energy industries would not be offered at the same American wood pulp than to get it from any other source.
time as any tax relief for other industries.
Which of the following is an assumption made in
(C) Advertisements of the solar-energy industry, by
drawing the conclusion above?
failing to identify the source of government
revenues explicitly to the public, mask the (A) Factory output of paper products in Japan and
advantage the industry receives from the public. Western Europe will increase sharply during this
year.
(D) Homeowners generally believe that they benefit
from any tax relief offered to the solar-energy (B) The quality of the wood pulp produced in the
industry. United States would be adequate for the purposes
of Japanese and Western European paper
(E) Tax relief would encourage solar industries to sell
manufacturers.
solar-energy systems at higher prices.
(C) Paper manufacturers in Japan and Western Europe
11. Less than 50 percent of a certain tropical country’s would prefer to use wood pulp produced in the
wildlands remains intact. Efforts are under way to restore United States if cost were not a factor.
biological diversity in that country by restoring some (D) Demand for paper products made in Japan and
destroyed wild habitats and extending some relatively Western Europe will not increase sharply during
intact portions of forests. However, opponents argue that this year.
these efforts are not needed because there is still plenty (E) Production of wood pulp by United States
( 35 )
PaGaLGuYS .com Education
TM
companies will not increase sharply during this (B) The quality of high school education improved.
year. (C) Compared to the 1970’s, a greater number of high
schools offered vocational guidance programs for
14. A company’s personnel director surveyed
their students.
employees about their satisfaction with the company’s
system for awarding employee performance ratings. The (D) The proportion of the population with at least a
survey data indicated that employees who received high college-level education increased.
ratings were very satisfied with the system. The (E) There was for the first time in 20 years an
personnel director concluded from these data that the oversupply of job seekers with only high school
company’s best-performing employees liked the system. diplomas.
The personnel director’s conclusion assumes which of 17. Working shorter workweeks causes managers to feel
the following? less stress than does working longer workweeks. In
(A) No other performance rating system is as good as addition, greater perceived control over one’s work life
the current system. reduces stress levels. It can be concluded, therefore, that
(B) The company’s best-performing employees shorter workweeks cause managers to feel they have more
received high ratings. control over their work life.
(C) Employees who received low ratings were The argument made above uses which of the following
dissatisfied with the system. questionable techniques?
(D) Employees who receive high ratings from a (A) Associating two conditions as cause and effect on
performance-rating system will like that system. the basis of their being causally associated with the
(E) The company’s best-performing employees were same phenomenon
motivated to perform well by the knowledge that (B) Taking for granted that two factors that have a
they would receive performance ratings. certain effect individually produce that effect more
strongly when both act together
15. In Argonia the average rate drivers pay for car
(C) Assuming what it sets out to prove
accident insurance is regulated to allow insurance
companies to make a reasonable profit. Under the (D) Using an irrelevant point in order to draw a
regulations, the rate any individual driver pays never conclusion
depends on the actual distance driven by that driver each (E) Basing a conclusion on preconceived views about
year. Therefore, Argonians who drive less than average the needs of managers
partially subsidize the insurance of those who drive more
18. There are fundamentally two possible changes in an
than average.
economy that will each cause inflation unless other
The conclusion above would be properly drawn if it compensating changes also occur. These changes are either
were also true that in Argonia reductions in the supply of goods and services or
(A) the average accident insurance rate for all drivers increases in demand. In a prebanking economy the
rises whenever a substantial number of new drivers quantity of money available, and hence the level of
buy insurance demand, is equivalent to the quantity of gold available.
(B) the average cost to insurance companies of insuring If the statements above are true, then it is also true that
drivers who drive less than the annual average is in a prebanking economy
less than the average cost of insuring drivers who (A) any inflation is the result of reductions in the
drive more than the annual average supply of goods and services
(C) the lower the age of a driver, the higher the (B) if other factors in the economy are unchanged,
insurance rate paid by that driver increasing the quantity of gold available will lead to
(D) insurance company profits would rise substantially inflation
if drivers were classified in terms of the actual (C) if there is a reduction in the quantity of gold
number of miles they drive each year available, then, other things being equal, inflation
(E) drivers who have caused insurance companies to must result
pay costly claims generally pay insurance rates (D) the quantity of goods and services purchasable by a
that are equal to or lower than those paid by other given amount of gold is constant
drivers
(E) whatever changes in demand occur, there will be
16. In the 1970’s there was an oversupply of college compensating changes in the supply of goods and
graduates. The oversupply caused the average annual services
income of college graduates to fall to a level only 18
19. Industrialists from the country Distopia were
percent greater than that of workers with only high
accused of promoting the Distopian intervention in the
school diplomas. By the late 1980’s the average annual
Arcadian civil war merely to insure that the industrialists’
income of college graduates was 43 percent higher than
facilities in Arcadia made substantial profits during the
that of workers with only high school diplomas, even
war. Yet this cannot be the motive since, as the
though between the 1970’s and the late 1980’s the supply
Distopians foresaw, Distopia’s federal expenses for the
of college graduates did not decrease.
intervention were eight billion dollars, whereas, during the
Which of the following, if true, in the late 1980’s, best war, profits from the Distopian industrialists’ facilities in
reconciles the apparent discrepancy described above? Arcadia totaled only four billion dollars.
(A) The economy slowed, thus creating a decreased Which of the following, if true, exposes a serious flaw in
demand for college graduates. the argument made in the second sentence above?

( 36 )
PaGaLGuYS .com Education
TM
(A) During the Arcadian war, many Distopian increased emphasis in universities on applied research.
industrialists with facilities located in Arcadia But we must not give too little attention to basic
experienced a significant rise in productivity in research, even though it may have no foreseeable
their facilities located in Distopia. application, for tomorrow’s applied research will depend
(B) The largest proportion of Distopia’s federal on the basic research of today.
expenses is borne by those who receive no If the statements above are true, which of the following
significant industrial profits. can be most reliably inferred?
(C) Most Distopian industrialists’ facilities located in (A) If future technological advancement is desired, basic
Arcadia are expected to maintain the level of research should receive greater emphasis than
profits they achieved during the war. applied research.
(D) Distopian industrialists’ facilities in Arcadia made (B) If basic research is valued in universities, applied
substantial profits before the events that triggered research should be given less emphasis than it
the civil war. currently has.
(E) Many Distopians expressed concern over the (C) If future technological advancement is desired,
suffering that Arcadians underwent during the civil research should be limited to that with some
war. foreseeable application.
20. In the United States, injuries to passengers involved (D) If too little attention is given to basic research
in automobile accidents are typically more severe than in today, future technological advancement will be
Europe, where laws require a different kind of safety belt. jeopardized.
It is clear from this that the United States needs to adopt (E) If technological advancement is given insufficient
more stringent standards for safety belt design to protect emphasis, basic research will also receive too little
automobile passengers better. attention.
Each of the following, if true, weakens the argument 3. The First Banking Group’s decision to invest in an
above EXCEPT: electronic network for transferring funds was based on a
(A) Europeans are more likely to wear safety belts than cost advantage over a nonelectronic system of about ten
are people in the United States. dollars per transaction in using an electronic system.
(B) Unlike United States drivers, European drivers Executives reasoned further that the system would give
receive training in how best to react in the event of them an advantage over competitors.
an accident to minimize injuries to themselves and Which of the following, if it is a realistic possibility,
to their passengers. most seriously weakens the executives’ projection of an
(C) Cars built for the European market tend to have advantage over competitors?
more sturdy construction than do cars built for the (A) The cost advantage of using the electronic system
United States market. will not increase sufficiently to match the pace of
(D) Automobile passengers in the United States have a inflation.
greater statistical chance of being involved in an (B) Competitors will for the same reasons install
accident than do passengers in Europe. electronic systems, and the resulting overcapacity
(E) States that have recently begun requiring the will lead to mutually damaging price wars.
European safety belt have experienced no (C) The electronic system will provide a means for
reduction in the average severity of injuries faster transfer of funds, if the First Banking Group
suffered by passengers in automobile accidents. wishes to provide faster transfer to its customers.
TEST 11 (D) Large banks from outside the area served by the
30 Minutes 20 Questions First Banking Group have recently established
branches in that area as competitors to the First
1. The school board has determined that it is Banking Group.
necessary to reduce the number of teachers on the staff. (E) Equipment used in the electronic network for
Rather than deciding which teachers will be laid off on the transferring funds will be compatible with
basis of seniority, the school board plans to lay off the equipment used in other such networks.
least effective teachers first.
The school board’s plan assumes that 4. Which of the following best completes the argument
below?
(A) there is a way of determining the effectiveness of
teachers One effect of the introduction of the electric refrigerator
(B) what one individual defines as effective teaching will was a collapse in the market for ice. Formerly
not be defined as effective teaching by another householders had bought ice to keep their iceboxes cool
individual and the food stored in the iceboxes fresh. Now the
iceboxes cool themselves. Similarly, the introduction of
(C) those with the most experience teaching are the best
crops genetically engineered to be resistant to pests
teachers
will______
(D) those teachers who are paid the most are generally
(A) increase the size of crop harvests
the most qualified
(B) increase the cost of seeds
(E) some teachers will be more effective working with
some students than with other students (C) reduce demand for chemical pesticides
(D) reduce the value of farmland
2. Since applied scientific research is required for (E) reduce the number of farmers keeping livestock
technological advancement, many have rightly urged an
( 37 )
PaGaLGuYS .com Education
TM
5. In 1985 the city’s Fine Arts Museum sold 30,000 and without employment, increased between 1981
single-entry tickets. In 1986 the city’s Folk Arts and and 1986
Interior Design museums opened, and these three (E) the average time that employees stay in any one job
museums together sold over 80,000 such tickets that year. dropped during the period 1981 to 1986
These museums were worth the cost, since more than
twice as many citizens are now enjoying the arts. 8. To reduce costs, a company is considering a drastic
reduction in the number of middle-level managers. This
Which of the following, if true, most seriously weakens
reduction would be accomplished by first offering early
the author’s assertion that more than twice as many
retirement to those 50 years of age or older with 15 years
citizens are now enjoying the arts?
of service, and then by firing enough of the others to
(A) Most visitors to one museum also visit the other bring the overall reduction to 50 percent.
two.
Each of the following, assuming that it is a realistic
(B) The cost of building the museums will not be
possibility, is a possible disadvantage to the company
covered by revenues generated by the sale of
of the plan EXCEPT:
museum tickets.
(A) Loyalty to the company will be reduced among
(C) As the two new museums become better known,
those surviving the reduction, because they will
even more citizens will visit them.
perceive the status of even good managers as
(D) The city’s Fine Arts Museum did not experience a uncertain.
decrease in single-entry tickets sold in 1986.
(B) The restructuring of managerial jobs will allow
(E) Fewer museum entry tickets were sold in 1986 than business units to be adapted to fit a changing
the museum planners had hoped to sell. business environment.
6. F: We ought not to test the safety of new drugs on (C) The company will have a smaller pool of managers
sentient animals, such as dogs and rabbits. Our benefit from which to choose in selecting future senior
means their pain, and they are equal to us in the capacity managers.
to feel pain. (D) Some of the best managers, unsure of their security
G: We must carry out such tests; otherwise, we would against being fired, will choose early retirement.
irresponsibly sacrifice the human lives that could have (E) The increased workload of managers remaining with
been saved by the drugs. the company will subject them to stress that will
eventually affect their performance.
Which of the following, if true, is the best objection that
could be made from F’s point of view to counter G’s 9. In order to relieve congestion in the airspace near
point? the airports of a certain country, transportation officials
(A) Even though it is not necessary for people to use propose sending passengers by new rapid trains between
cosmetics, cosmetics are also being tested on the country’s major airport and several small cities within
sentient animals. a 300-mile radius of it. This plan was proposed even
(B) Medical science already has at its disposal a great though the officials realized that it is the major airport
number of drugs and other treatments for serious that is congested, not those in the small cities.
illnesses. The plan to relieve congestion would work best if which
(C) It is not possible to obtain scientifically adequate of the following were true about the major airport?
results by testing drugs in the test tube, without (A) Rail tickets between the airport and the small cities
making tests on living tissue. will most likely cost more than the current air
(D) Some of the drugs to be tested would save human tickets for those routes.
beings from great pain. (B) Most passengers who frequently use the airport
(E) Many tests now performed on sentient animals can prefer to reach their cities of destination
be performed equally well on fertilized chicken exclusively by air, even if they must change planes
eggs that are at a very early stage of development. twice.
(C) There are feasible changes in the airport’s traffic
7. Which of the following best completes the passage
control system which would significantly relieve
below?
congestion.
The unemployment rate in the United States fell from (D) Some of the congestion the airport experiences
7.5 percent in 1981 to 6.9 percent in 1986. It cannot, could be relieved if more flights were scheduled at
however, be properly concluded from these statistics night and at other off-peak hours.
that the number of unemployed in 1986 was lower than
(E) A significant proportion of the airport’s traffic
it had been in 1981 because______
consists of passengers transferring between
(A) help-wanted advertisements increased between international flights and flights to the small cities.
1981 and 1986
(B) many of the high-paying industrial jobs available in Questions 10-11 are based on the following.
1981 were replaced by low-wage service jobs in
An annually conducted, nationwide survey shows a
1986, resulting in displacements of hundreds of
continuing marked decline in the use of illegal drugs by high
thousands of workers
school seniors over the last three years.
(C) in some midwestern industrial states, the
unemployment rate was much higher in 1986 than 10. Which of the following, if true, casts most doubt on
it had been in 1981 the relevance of the survey results described above for
(D) the total available work force, including those with drawing conclusions about illegal drug use in the teen-age

( 38 )
PaGaLGuYS .com Education
TM
population as a whole? prior to the restrictions’ introduction.
(A) Because of cuts in funding, no survey of illegal drug which of the following, if true, would, in conjunction
use by high school seniors will be conducted next with the statements above, best support the conclusion
year. that the government policy described above fails to
(B) The decline uncovered in the survey has occurred achieve its objective?
despite the decreasing cost of illegal drugs. (A) Because of the revenues gained from the sale of
(C) Illegal drug use by teen-agers is highest in those alcohol and tobacco, governments have no real
areas of the country where teen-agers are least interest in making these products less attractive to
likely to stay in high school for their senior year. consumers.
(D) Survey participants are more likely now than they (B) Advertisers tend to create inventive and humorous
were three years ago to describe as “heroic” people advertisements only if they have some particular
who were addicted to illegal drugs and have been reason to do so.
able to quit. (C) Banning advertising of alcohol and tobacco is a
(E) The proportion of high school seniors who say that particularly effective way of making these
they strongly disapprove of illegal drug use has products less attractive to consumers.
declined over the last three years. (D) With the policy in place, advertisements for alcohol
and tobacco have become far more inventive and
11. Which of the following, if true, would provide most
humorous than advertisements for other kinds of
support for concluding from the survey results described
products.
above that the use of illegal drugs by people below the
age of 20 is declining? (E) The more inventive an advertisement is, the more
attractive it makes the advertised product appear.
(A) Changes in the level of drug use by high school
seniors are seldom matched by changes in the level 14. Which of the following, if true, best completes the
of drug use by other people below the age of 20. argument below?
(B) In the past, high school seniors were consistently Comparisons of the average standards of living of the
the population group most likely to use illegal citizens of two countries should reflect the citizens’
drugs and most likely to use them heavily. comparative access to goods and services. Reliable
(C) The percentage of high school seniors who use figures in a country’s own currency for the average
illegal drugs is consistently very similar to the income of its citizens are easily obtained. But it is
percentage of all people below the age of 20 who difficult to get an accurate comparison of average
use illegal drugs. standards of living from these figures, because______
(D) The decline revealed by the surveys is the result of (A) there are usually no figures comparing how much of
drug education programs specifically targeted at two different currencies must be spent in order to
those below the age of 20. purchase a given quantity of goods and services
(E) The number of those surveyed who admit to having (B) wage levels for the same job vary greatly from
sold illegal drugs has declined even faster than has country to country, depending on cultural as well
the number who have used drugs. as on purely economic factors
12. President of the United States: I have received over (C) these figures must be calculated by dividing the
2,000 letters on this issue, and the vast majority of them gross national product of a country by the size of
support my current position. These letters prove that its population
most of the people in the country agree with me. (D) comparative access to goods and services is only
one of several factors relevant in determining
Which of the following, if true, most weakens the
quality of life
President’s conclusion?
(E) the wealth, and hence the standard of living, of a
(A) The issue is a very divisive one on which many
country’s citizens is very closely related to their
people have strong opinions.
income
(B) Some members of Congress disagree with the
President’s position. 15. The level of lead contamination in United States
(C) People who disagree with the President feel more rivers declined between 1975 and 1985. Federal
strongly about the issue than do people who agree regulations requiring a drop in industrial discharges of
with him. lead went into effect in 1975, but the major cause of the
(D) People who agree with the President are more likely decline was a 75 percent drop in the use of leaded
to write to him than are people who disagree with gasoline between 1975 and 1985.
him. Which of the following, if true, best supports the claim
(E) During the presidential campaign, the President that the major cause of the decline in the level of lead
stated a position on this issue that was somewhat contamination in United States rives was the decline in
different from his current position. the use of leaded gasoline?
(A) The level of lead contamination in United States
13. Some governments have tried to make alcohol and rivers fell sharply in both 1975 and 1983.
tobacco less attractive to consumers by regulating what
(B) Most of the decline in industrial discharges of lead
can be shown in advertisements for these products, rather
occurred before 1976, but the largest decline in the
than by banning advertising of them altogether. However,
level of river contamination occurred between 1980
the need to obey the letter of these restrictions has
and 1985.
actually stimulated advertisers to create advertisements
that are more inventive and humorous than they were (C) Levels of lead contamination in rivers fell sharply in
( 39 )
PaGaLGuYS .com Education
TM
1975-1976 and rose very slightly over the next (B) The author applies to the case under discussion
nine years. facts about phenomena assumed to be similar in
(D) Levels of lead contamination rose in those rivers some relevant respect.
where there was reduced river flow due to drought. (C) A position is strengthened by showing that the
(E) Although the use of leaded gasoline declined 75 opposite of that position would have logically
percent between 1975 and 1985, 80 percent of the absurd consequences.
decline took place in 1985. (D) A line of reasoning is called into question on the
grounds that it confuses cause and effect in a causal
16. George Bernard Shaw wrote: “That any sane relation.
nation, having observed that you could provide for the
(E) An argument is analyzed by separating statements
supply of bread by giving bakers a pecuniary interest in
of fact from individual value judgments.
baking for you, should go on to give a surgeon a
pecuniary interest in cutting off your leg is enough to 19. It is widely assumed that a museum is helped
make one despair of political humanity.” financially when a generous patron donates a potential
Shaw’s statement would best serve as an illustration in exhibit. In truth, however, donated objects require storage
an argument criticizing which of the following? space, which is not free, and routine conservation, which
(A) Dentists who perform unnecessary dental work in is rather expensive. Therefore, such gifts exacerbate rather
order to earn a profit than lighten the demands made on a museum’s financial
resources.
(B) Doctors who increase their profits by specializing
only in diseases that affect a large percentage of the Which of the following, if true, most seriously weakens
population the argument above?
(C) Grocers who raise the price of food in order to (A) To keep patrons well disposed, a museum will find
increase their profit margins it advisable to put at least some donated objects on
(D) Oil companies that decrease the price of their oil in exhibit rather than merely in storage.
order to increase their market share (B) The people who are most likely to donate valuable
(E) Bakers and surgeons who earn a profit by supplying objects to a museum are also the people who are
other peoples’ basic needs most likely to make cash gifts to it.
(C) A museum cannot save money by resorting to cheap
17. Since 1975 there has been in the United States a storage under less than adequate conditions,
dramatic decline in the incidence of traditional childhood because so doing would drive up the cost of
diseases such as measles. This decline has been conservation.
accompanied by an increased incidence of Peterson’s (D) Patrons expect a museum to keep donated objects
disease, a hitherto rare viral infection, among children. in its possession rather than to raise cash by selling
Few adults, however, have been affected by the disease. them.
Which of the following, if true, would best help to (E) Objects donated by a patron to a museum are often
explain the increased incidence of Peterson’s disease of such importance that the museum would be
among children? obliged to add them to its collection through
(A) Hereditary factors determine in part the degree to purchase if necessary.
which a person is susceptible to the virus that
causes Peterson’s disease. 20. Despite the approach of winter, oil prices to
industrial customers are exceptionally low this year and
(B) The decrease in traditional childhood diseases and
likely to remain so. Therefore, unless the winter is
the accompanying increase in Peterson’s disease
especially severe, the price of natural gas to industrial
have not been found in any other country.
customers is also likely to remain low.
(C) Children who contract measles develop an
immunity to the virus that causes Peterson’s Which of the following, if true, provides the most
disease. support for the conclusion above?
(D) Persons who did not contract measles in childhood (A) Long-term weather forecasts predict a mild winter.
might contract measles in adulthood, in which case (B) The industrial users who consume most natural gas
the consequences of the disease would generally be can quickly and cheaply switch to using oil
more severe. instead.
(E) Those who have contracted Peterson’s disease are at (C) The largest sources of supply for both oil and
increased risk of contracting chicken pox. natural gas are in subtropical regions unlikely to be
affected by winter weather.
18. Many plant varieties used in industrially developed (D) The fuel requirements of industrial users of natural
nations to improve cultivated crops come from less gas are not seriously affected by the weather.
developed nations. No compensation is paid on the
(E) Oil distribution is more likely to be affected by
grounds that the plants used are “the common heritage of
severe winter weather than is the distribution of
humanity.” Such reasoning is, however, flawed. After all,
natural gas.
no one suggests that coal, oil, and ores should be
extracted without payment. TEST 12
Which of the following best describes an aspect of the 30 Minutes 20 Questions
method used by the author in the argument above? 1. The country of Maravia has severe air pollution, 80
(A) The author proceeds from a number of specific percent of which is caused by the exhaust fumes of cars.
observations to a tentative generalization. In order to reduce the number of cars on the road, the
( 40 )
PaGaLGuYS .com Education
TM
government is raising taxes on the cost of buying and manufacturing companies were filled by workers
running a car by 20 percent. This tax increase, therefore, who moved to Alameda because they had skills for
will significantly reduce air pollution in Maravia. which there was no demand in Alameda prior to
Which of the following, if true, most seriously weakens the introduction of microelectronics there.
the argument above? (E) Many workers who have retired from the
(A) The government of Maravia is in the process of manufacturing industry in Alameda since 1979
building a significant number of roadways. have not been replaced by younger workers.
(B) Maravia is an oil-producing country and is able to 4. Which of the following, if true, would most
refine an amount of gasoline sufficient for the strengthen the labor leaders’ claim concerning the
needs of its population. manufacturing industry in Alameda?
(C) Maravia has had an excellent public transportation (A) From 1979 to 1990, fewer employees of
system for many years. manufacturing companies in Alameda lost their
(D) Ninety percent of the population of Maravia is jobs because of the introduction of
very prosperous and has a substantial amount of microelectronics than did employees of
disposable income. manufacturing companies in the nearby community
(E) In Maravia, cars that emit relatively low levels of of Rockside.
pollutants cost 10 percent less to operate, on (B) The figures on the use of microelectronics that were
average, than do cars that emit high levels of made public are the result of inquiries made of
pollutants. managers in the manufacturing industry in
Alameda.
2. Consumer income reports produced by the
(C) The organizational changes that led to job losses in
government distinguish between households and families
all sectors of the manufacturing industry in
by means of the following definition: “A family is a
Alameda were primarily the result of the
household containing a householder and at least one
introduction of microelectronics.
person related to the householder.” Except for the
homeless and people in group living quarters, most (D) Figures on job losses in the manufacturing industry
people live in households. in Alameda for the late sixties and early seventies
have not been made available.
According to the definition above, which of the
(E) A few jobs in the manufacturing industry in Alameda
following must be true?
could have been saved if workers had been willing
(A) All householders are members of families. to become knowledgeable in microelectronics.
(B) All families include a householder.
(C) All of the people related to a householder form a 5. The number of musicians employed to play
family. accompaniment for radio and television commercials has
sharply decreased over the past ten years. This has
(D) Some people residing in group living quarters are
occurred even though the number of commercials
members of families.
produced each year has not significantly changed for the
(E) Some homeless people reside in group living last ten years.
quarters.
Which of the following, if it occurred during the past ten
Questions 3-4 are based on the following. years, would contribute LEAST to an explanation of the
facts above?
The proportion of manufacturing companies in Alameda that (A) The type of music most popular for use in
use microelectronics in their manufacturing processes commercials has changed from a type that requires
increased from 6 percent in 1979 to 66 percent in 1990. a large number of instruments to a type that
Many labor leaders say that the introduction of requires very few instruments.
microelectronics is the principal cause of the great increase in
(B) There has been an increase in the number of
unemployment during that period in Alameda. In actual fact,
commercials that use only the spoken word and
however, most of the job losses were due to organizational
sound effects, rather than musical accompaniment.
changes. Moreover, according to new figures released by the
labor department, there were many more people employed in (C) There has been an increase in the number of
Alameda in the manufacturing industry in 1990 than in 1979. commercials that use a synthesizer, an instrument
on which one musician can reproduce the sound of
3. Which of the following, if true, best reconciles the many musicians playing together.
discrepancy between the increase in unemployment and (D) There has been an increase in the number of
the increase in jobs in the manufacturing industry of commercials that use prerecorded music as their
Alameda? only source of music.
(A) Many products that contain microelectronic (E) There has been an increase in the number of
components are now assembled completely by commercials that use musicians just starting in the
machine. music industry rather than musicians experienced
(B) Workers involved in the various aspects of the in accompanying commercials.
manufacturing processes that use microelectronic
technology need extensive training. 6. Recent audits revealed that BanqueCard, a credit
service, has erred in calculating the interest it charges its
(C) It is difficult to evaluate numerically what impact
clients. But BanqueCard’s chief accountant reasoned that
on job security the introduction of microelectronics
the profits that the company shows would remain
in the workplace had before 1979.
unaffected by a revision of its clients’ credit statements
(D) In 1990 over 90 percent of the jobs in Alameda’s
( 41 )
PaGaLGuYS .com Education
TM
to correct its previous billing errors, since just as many (D) None of the applicants had applied for jobs at
clients had been overcharged as undercharged. places other than Evco and Radeco.ÿCÿ
Which of the following is a reasoning error that the (E) None of the applicants had previously worked for
accountant makes in concluding that correcting its either Evco or Radeco.
clients’ statements would leave BanqueCard’s profits
10. The geese that gather at the pond of a large
unaffected?
corporation create a hazard for executives who use the
(A) Relying on the reputation of BanqueCard as a corporate helicopter, whose landing site is 40 feet away
trustworthy credit service to maintain the from the pond. To solve the problem, the corporation
company’s clientele after the error becomes widely plans to import a large number of herding dogs to keep
known the geese away from the helicopter.
(B) Failing to establish that BanqueCard charges the
Which of the following, if a realistic possibility, would
same rates of interest for all of its clients
cast the most serious doubt on the prospects for
(C) Overlooking the possibility that the amount by success of the corporation’s plan?
which BanqueCard’s clients had been overcharged
(A) The dogs will form an uncontrollable pack.
might be greater than the amount by which they
had been undercharged (B) The dogs will require training to learn to herd the
geese.
(D) Assuming that the clients who had been overcharged
by BanqueCard had not noticed the error in their (C) The dogs will frighten away foxes that prey on old
credit bills and sick geese.
(E) Presupposing that each one of BanqueCard’s clients (D) It will be necessary to keep the dogs in quarantine
had either been overcharged or else had been for 30 days after importing them.
undercharged by the billing error (E) Some of the geese will move to the pond of another
corporation in order to avoid being herded by the
7. Not Scored dogs.
8. Residents of an apartment complex are considering 11. When a person is under intense psychological
two possible plans for collecting recyclable trash. stress, his or her cardiovascular response is the same as it
Plan 1 - Residents will deposit recyclable trash in is during vigorous physical exercise. Psychological stress,
municipal dumpsters located in the parking lot. The then, must be beneficial for the heart as is vigorous
trash will be collected on the first and the fifteenth days physical exercise.
of each month. The argument above relies on which of the following
Plan 2 - Residents will be given individual containers for assumptions?
recyclable trash. The containers will be placed at the (A) Exercise is an effective means of relieving
curb twice a week for trash collection. psychological stress.
Which of the following points raised at a meeting of the (B) The body’s short-term cardiovascular response to
residents, if valid, would most favor one of the recycling any activity indicates that activity’s long-term
plans over the other? effect on the body.
(A) Residents will be required to exercise care in (C) Cardiovascular response during an activity is an
separating recyclable trash from nonrecyclable adequate measure of how beneficial the activity is
trash. for the heart.
(B) For trash recycling to be successful, residents must (D) Psychological stress can have a positive effect on
separate recyclable bottles and cans from the body.
recyclable paper products. (E) Vigorous exercise is the most reliable method of
(C) Penalties will be levied against residents who fail to maintaining a healthy heart.
sort their trash correctly.
12. After graduating from high school, people rarely
(D) Individual recycling containers will need to be made multiply fractions or discuss ancient Rome, but they are
of a strong and durable material. confronted daily with decisions relating to home
(E) Recyclable trash that is allowed to accumulate for economics. Yet whereas mathematics and history are
two weeks will attract rodents. required courses in the high school curriculum, home
economics is only an elective, and few students choose to
9. In 1990 all of the people who applied for a job at
take it.
Evco also applied for a job at Radeco, and Evco and
Radeco each offered jobs to half of these applicants. Which of the following positions would be best
Therefore, every one of these applicants must have been supported by the considerations above?
offered a job in 1990. (A) If mathematics and history were not required
The argument above is based on which of the following courses, few students would choose to take them.
assumptions about these job applicants? (B) Whereas home economics would be the most useful
(A) All of the applicants were very well qualified for a subject for people facing the decisions they must
job at either Evco or Radeco. make in daily life, often mathematics and history
can also help them face these decisions.
(B) All of the applicants accepted a job at either Evco or
Radeco. (C) If it is important to teach high school students
subjects that relate to decisions that will confront
(C) None of the applicants was offered a job by both
them in their daily lives, then home economics
Evco and Radeco.
should be made an important part of the high
( 42 )
PaGaLGuYS .com Education
TM
school curriculum. Which of the following, if true, would best support the
(D) Mathematics, history, and other courses that are prediction above?
not directly relevant to a person’s daily life should (A) Human beings are dependent on visual cues from
not be a required part of the high school motion in order to detect spatial relations.
curriculum. (B) Human beings can often easily detect the spatial
(E) Unless high schools put more emphasis on relations among objects, even when those objects
nonacademic subjects like home economics, people are in motion.
graduating from high school will never feel (C) Detecting spatial relations among objects requires
comfortable about making the decisions that will drawing inferences from the information conveyed
confront them in their daily lives. by light.
13. Houses built during the last ten years have been (D) Although human beings can discern spatial relations
found to contain indoor air pollution at levels that are, on through their sense of hearing, vision is usually the
average, much higher than the levels found in older most important means of detecting spatial
houses. The reason air-pollution levels are higher in the relations.
newer houses is that many such houses are built near the (E) Information about the spatial relations among
sites of old waste dumps or where automobile emissions objects can be obtained by noticing such things as
are heavy. shadows and the relative sizes of objects.
Which of the following, if true, calls into question the 16. In a study of the effect of color on productivity, 50
explanation above? of 100 factory workers were moved from their drab
(A) Many new houses are built with air-filtration workroom to a brightly colored workroom. Both these
systems that remove from the house pollutants workers and the 50 who remained in the drab workroom
that are generated indoors. increased their productivity, probably as a result of the
(B) The easing of standards for smokestack emissions interest taken by researchers in the work of both groups
has led to an increase in air-pollution levels in during the study.
homes. Which of the following, if true, would cast most doubt
(C) New houses built in secluded rural areas are upon the author’s interpretation of the study results
relatively free of air pollutants. given above?
(D) Warm-weather conditions tend to slow down the (A) The 50 workers moved to the brightly colored room
movement of air, thus keeping pollution trapped performed precisely the same manufacturing task
near its source. as the workers who remained in the drab
(E) Pressboard, an inexpensive new plywood substitute workroom.
now often used in the construction of houses, (B) The drab workroom was designed to provide
emits the pollutant formaldehyde into the house. adequate space for at most 65 workers.
(C) The 50 workers who moved to the brightly colored
14. The most important aspect of moviemaking is
workroom were matched as closely as possible in
conveying a scene’s rhythm. Conveying rhythm depends
age and level of training to the 50 workers who
less on the artistic quality of the individual photographic
remained in the drab work-room.
images than on how the shots go together and the order in
which they highlight different aspects of the action taking (D) Nearly all the workers in both groups had
place in front of the camera. volunteered to move to the brightly colored
workroom.
If the statements above are true, which of the following
(E) Many of the workers who moved to the brightly
must be true on the basis of them?
colored workroom reported that they liked the
(A) The artistic quality of the individual photographic drab workroom as well as or better than they liked
image is unimportant in movie photography. the brightly colored workroom.
(B) Photographers known for the superb artistic quality
of their photographs are seldom effective as 17. Not Scored
moviemakers.
18. Manager: Accounting and Billing are located right
(C) Having the ability to produce photographs of next to each other and the two departments do similar
superb artistic quality does not in itself guarantee kinds of work; yet expenditures for clerical supplies
having the ability to be a good moviemaker. charged to Billing are much higher. Is Billing wasting
(D) Movie photographers who are good at their jobs supplies?
rarely give serious thought to the artistic quality of
Head of Billing: Not at all.
the photographs they take.
(E) To convey a scene’s rhythm effectively, a Which of the following, if true, best supports the
moviemaker must highlight many different aspects position of the Head of Billing?
of the action taking place. (A) There are more staff members in Accounting than in
Billing.
15. Human beings can see the spatial relations among (B) Two years ago, expenditures in Accounting for
objects by processing information conveyed by light. clerical supplies were the same as were
Scientists trying to build computers that can detect expenditures that year in Billing for clerical
spatial relations by the same kind of process have so far supplies.
designed and built stationary machines. However, these
(C) The work of Billing now requires a wider variety of
scientists will not achieve their goal until they produce
clerical supplies than it did in the past.
such a machine that can move around in its environment.
( 43 )
PaGaLGuYS .com Education
TM
(D) Some of the paper-and-pencil work of both likely to be influenced in their purchasing decisions
Accounting and Billing has been replaced by work by television advertising than are consumers who
done on computers. own television sets.
(E) Members of Accounting found the clerical supplies (B) Subscriptions to cable television include access to
cabinet of Billing more convenient to go to for some public-television channels, which do not
supplies than their own department’s cabinet. accept advertising.
(C) For locations with poor television reception, cable
19. Most geologists believe oil results from chemical
television provides picture quality superior to that
transformations of hydrocarbons derived from organisms
provided by free television.
buried under ancient seas. Suppose, instead, that oil
actually results from bacterial action on other complex (D) There is as much advertising on many cable-
hydrocarbons that are trapped within the Earth. As is television channels as there is on “free” television
well known, the volume of these hydrocarbons exceeds channels.
that of buried organisms. Therefore, our oil reserves (E) Cable-television subscribers can choose which
would be greater than most geologists believe. channels they wish to receive, and the fees vary
accordingly.
Which of the following, if true, gives the strongest
support to the argument above about our oil reserves? 2. Woodsmoke contains dangerous toxins that cause
(A) Most geologists think optimistically about the changes in human cells. Because woodsmoke presents
Earth’s reserves of oil. such a high health risk, legislation is needed to regulate
(B) Most geologists have performed accurate chemical the use of open-air fires and wood-burning stoves.
analyses on previously discovered oil reserves. Which of the following, if true, provides the most
(C) Ancient seas are buried within the Earth at many support for the argument above?
places where fossils are abundant. (A) The amount of dangerous toxins contained in
(D) The only bacteria yet found in oil reserves could woodsmoke is much less than the amount
have leaked down drill holes from surface contained in an equal volume of automobile
contaminants. exhaust.
(E) Chemical transformations reduce the volume of (B) Within the jurisdiction covered by the proposed
buried hydrocarbons derived from organisms by legislation, most heating and cooking is done with
roughly the same proportion as bacterial action oil or natural gas.
reduces the volume of other complex (C) Smoke produced by coal-burning stoves is
hydrocarbons. significantly more toxic than smoke from wood-
burning stoves.
20. The wild mouflon sheep of the island of Corsica are
direct descendants of sheep that escaped from (D) No significant beneficial effect on air quality would
domestication on the island 8,000 years ago. They result if open-air fires were banned within the
therefore provide archaeologists with a picture of what jurisdiction covered by the proposed legislation.
some early domesticated sheep looked like, before the (E) In valleys where wood is used as the primary
deliberate selective breeding that produced modern heating fuel, the concentration of smoke results in
domesticated sheep began. poor air quality.
The argument above makes which of the following 3. Within 20 years it will probably be possible to
assumptions? identify the genetic susceptibility an individual may have
(A) The domesticated sheep of 8,000 years ago were toward any particular disease. Eventually, effective
quite dissimilar from the wild sheep of the time. strategies will be discovered to counteract each such
(B) There are no other existing breeds of sheep that susceptibility. Once these effective strategies are found,
escaped from domestication at about the same time therefore, the people who follow them will never get sick.
as the forebears of the mouflon. The argument above is based on which of the following
(C) Modern domesticated sheep are direct descendants assumptions?
of sheep that were wild 8,000 years ago. (A) For every disease there is only one strategy that can
(D) Mouflon sheep are more similar to their forebears prevent its occurrence.
of 8,000 years ago than modern domesticated (B) In the future, genetics will be the only medical
sheep are to theirs. specialty of any importance.
(E) The climate of Corsica has not changed at all in the (C) All human sicknesses are in part the result of
last 8,000 years. individuals’ genetic susceptibilities.
TEST 13 (D) All humans are genetically susceptible to some
25 Minutes 16 Questions diseases.
(E) People will follow medical advice when they are
1. Cable-television spokesperson: Subscriptions to convinced that it is effective.
cable television are a bargain in comparison to “free”
television. Remember that “free” television is not really 4. Most employees in the computer industry move
free. It is consumers, in the end, who pay for the costly from company to company, changing jobs several times in
advertising that supports “free” television. their careers. However, Summit Computers is known
Which of the following, if true, is most damaging to the throughout the industry for retaining its employees.
position of the cable-television spokesperson? Summit credits its success in retaining employees to its
informal, nonhierarchical work environment.
(A) Consumers who do not own television sets are less
( 44 )
PaGaLGuYS .com Education
TM
Which of the following, if true, most strongly supports under age four may not want to spend their income
Summit’s explanation of its success in retaining tax refund on child care.
employees? (C) The reduction in government revenues stemming
(A) Some people employed in the computer industry from the income tax refund will necessitate cuts in
change jobs if they become bored with their current other government programs, such as grants for
projects. higher education.
(B) A hierarchical work environment hinders the (D) Many low-income families with children under age
cooperative exchange of ideas that computer four do not pay any income taxes because their
industry employees consider necessary for their total income is too low to be subject to such taxes.
work. (E) Income taxes have increased substantially over the
(C) Many of Summit’s senior employees had past twenty years, reducing the money that low-
previously worked at only one other computer income families have available to spend on child
company. care.
(D) In a nonhierarchical work environment, people
7. Not scored
avoid behavior that might threaten group harmony
and thus avoid discussing with their colleagues any 8. Although parapsychology is often considered a
dissatisfaction they might have with their jobs. pseudoscience, it is in fact a genuine scientific enterprise,
(E) The cost of living near Summit is relatively low for it uses scientific methods such as controlled
compared to areas in which some other computer experiments and statistical tests of clearly stated
companies are located. hypotheses to examine the questions it raises.
5. Financing for a large construction project was The conclusion above is properly drawn if which of the
provided by a group of banks. When the money was gone following is assumed?
before the project was completed, the banks approved (A) If a field of study can conclusively answer the
additional loans. Now, with funds used up again and questions it raises, then it is a genuine science.
completion still not at hand, the banks refuse to extend (B) Since parapsychology uses scientific methods, it
further loans, although without those loans, the project is will produce credible results.
doomed. (C) Any enterprise that does not use controlled
Which of the following, if true, best explains why the experiments and statistical tests is not genuine
bank’s current reaction is different from their reaction in science.
the previous instance of depletion of funds? (D) Any field of study that employs scientific methods
(A) The banks have reassessed the income potential of is a genuine scientific enterprise.
the completed project and have concluded that (E) Since parapsychology raises clearly statable
total income generable would be less than total questions, they can be tested in controlled
interest due on the old plus the needed new loans. experiments.
(B) The banks have identified several other projects that
offer faster repayment of the principal if loans are 9. Hotco oil burners, designed to be used in asphalt
approved now to get those projects started. plants, are so efficient that Hotco will sell one to the
Clifton Asphalt plant for no payment other than the cost
(C) The banks had agreed with the borrowers that the
savings between the total amount the asphalt plant
construction loans would be secured by the
actually paid for oil using its former burner during the last
completed project.
two years and the total amount it will pay for oil using
(D) The cost overruns were largely due to unforeseeable the Hotco burner during the next two years. On
problems that arose in the most difficult phase of installation, the plant will make an estimated payment,
the construction work. which will be adjusted after two years to equal the actual
(E) The project stimulated the development and cost savings.
refinement of several new construction techniques,
Which of the following, if it occurred, would constitute
which will make it easier and cheaper to carry out
a disadvantage for Hotco of the plan described above?
similar projects in the future.
(A) Another manufacturer’s introduction to the market
6. Low-income families are often unable to afford as of a similarly efficient burner
much child care as they need. One government program (B) The Clifton Asphalt plant’s need for more than one
would award low-income families a refund on the income new burner
taxes they pay of as much as $1,000 for each child under (C) Very poor efficiency in the Clifton Asphalt plant’s
age four. This program would make it possible for all low- old burner
income families with children under age four to obtain
(D) A decrease in the demand for asphalt
more child care than they otherwise would have been able
to afford. (E) A steady increase in the price of oil beginning soon
after the new burner is installed
Which of the following, if true, most seriously calls into
question the claim that the program would make it 10. Today’s low gasoline prices make consumers willing
possible for all low-income families to obtain more child to indulge their preference for larger cars, which consume
care? greater amounts of gasoline as fuel. So United States
(A) The average family with children under age four automakers are unwilling to pursue the development of
spends more than $1,000 a year on child care. new fuel-efficient technologies aggressively. The
(B) Some low-income families in which one of the particular reluctance of the United States automobile
parents is usually available to care for children industry to do so, however, could threaten the industry’s
( 45 )
PaGaLGuYS .com Education
TM
future. (B) When they are imprisoned, drug addicts often use
Which of the following, if true, would provide the most their ability to manipulate other people to obtain
support for the claim above about the future of the better living conditions.
United States automobile industry? (C) Some nonaddicts manipulate other people more
(A) A prototype fuel-efficient vehicle, built five years than some addicts do.
ago, achieves a very high 81 miles per gallon on the (D) People who are likely to become addicts exhibit
highway and 63 in the city, but its materials are unusual behavior patterns other than frequent
relatively costly. manipulation of other people.
(B) Small cars sold by manufacturers in the United (E) The addicts that the researcher studied were often
States are more fuel efficient now than before the unsuccessful in obtaining what they wanted when
sudden jump in oil prices in 1973. they manipulated other people.
(C) Automakers elsewhere in the world have slowed the 13. One way to judge the performance of a company is
introduction of fuel-efficient technologies but have to compare it with other companies. This technique,
pressed ahead with research and development of commonly called “benchmarking,” permits the manager of
them in preparation for a predicted rise in world oil a company to discover better industrial practices and can
prices. provide a justification for the adoption of good practices.
(D) There are many technological opportunities for
Any of the following, if true, is a valid reason for
reducing the waste of energy in cars and light
benchmarking the performance of a company against
trucks through weight, aerodynamic drag, and
companies with which it is not in competition rather
braking friction.
than against competitors EXCEPT:
(E) The promotion of mass transit over automobiles as
(A) Comparisons with competitors are most likely to
an alternative mode of transportation has
focus on practices that the manager making the
encountered consumer resistance that is due in part
comparisons already employs.
to the failure of mass transit to accommodate the
wide dispersal of points of origin and destinations (B) Getting “inside” information about the unique
for trips. practices of competitors is particularly difficult.
(C) Since companies that compete with each other are
11. An experiment was done in which human subjects likely to have comparable levels of efficiency, only
recognize a pattern within a matrix of abstract designs benchmarking against noncompetitors is likely to
and then select another design that completes that reveal practices that would aid in beating
pattern. The results of the experiment were surprising. competitors.
The lowest expenditure of energy in neurons in the brain (D) Managers are generally more receptive to new ideas
was found in those subjects who performed most that they find outside their own industry.
successfully in the experiments.
(E) Much of the success of good companies is due to
Which of the following hypotheses best accounts for their adoption of practices that take advantage of
the findings of the experiment? the special circumstances of their products of
(A) The neurons of the brain react less when a subject is markets.
trying to recognize patterns than when the subject
is doing other kinds of reasoning. 14. Among the more effective kinds of publicity that
publishers can get for a new book is to have excerpts of it
(B) Those who performed best in the experiment
published in a high-circulation magazine soon before the
experienced more satisfaction when working with
book is published. The benefits of such excerption
abstract patterns than did those who performed
include not only a sure increase in sales but also a fee
less well.
paid by the magazine to the book’s publisher.
(C) People who are better at abstract pattern
recognition have more energy-efficient neural Which of the following conclusions is best supported
connections. by the information above?
(D) The energy expenditure of the subjects brains (A) The number of people for whom seeing an excerpt
increases when a design that completes the initially of a book in a magazine provides an adequate
recognized pattern is determined. substitute for reading the whole book is smaller
than the number for whom the excerpt stimulates a
(E) The task of completing a given design is more
desire to read the book.
capably performed by athletes, whose energy
expenditure is lower when they are at rest than is (B) Because the financial advantage of excerpting a new
that of the general population. book in a magazine usually accrues to the book’s
publisher, magazine editors are unwilling to
12. A researcher studying drug addicts found that, on publish excerpts from new books.
average, they tend to manipulate other people a great deal (C) In calculating the total number of copies that a book
more than nonaddicts do. The researcher concluded that has sold, publishers include sales of copies of
people who frequently manipulate other people are likely magazines that featured an excerpt of the book.
to become addicts. (D) The effectiveness of having excerpts of a book
Which of the following, if true, most seriously weakens published in a magazine, measured in terms of
the researcher’s conclusion? increased sales of a book, is proportional to the
(A) After becoming addicted to drugs, drug addicts learn circulation of the magazine in which the excerpts
to manipulate other people as a way of obtaining are published.
drugs. (E) Books that are suitable for excerpting in high-

( 46 )
PaGaLGuYS .com Education
TM
circulation magazines sell more copies than books important in understanding the technology used in
that are not suitable for excerpting. today’s world.
(C) Equipment that a large producer of photographic
15. In Swartkans territory, archaeologists discovered
equipment has donated to the high school could be
charred bone fragments dating back 1 million years.
used in the proposed curriculum.
Analysis of the fragments, which came from a variety of
animals, showed that they had been heated to (D) The number of students interested in physics today
temperatures no higher than those produced in is much lower than the number of students
experimental campfires made from branches of white interested in physics 50 years ago.
stinkwood, the most common tree around Swartkans. (E) In today’s world the production and analysis of
visual images is of major importance in
Which of the following, if true, would, together with the
communications, business, and recreation.
information above, provide the best basis for the claim
that the charred bone fragments are evidence of the use 2. Many companies now have employee assistance
of fire by early hominids? programs that enable employees, free of charge, to
(A) The white stinkwood tree is used for building improve their physical fitness, reduce stress, and learn
material by the present-day inhabitants of ways to stop smoking. These programs increase worker
Swartkans. productivity, reduce absenteeism, and lessen insurance
(B) Forest fires can heat wood to a range of costs for employee health care. Therefore, these programs
temperatures that occur in campfires. benefit the company as well as the employee.
(C) The bone fragments were fitted together by the Which of the following, if true, most significantly
archaeologists to form the complete skeletons of strengthens the conclusion above?
several animals. (A) Physical fitness programs are often the most
(D) Apart from the Swartkans discovery, there is popular services offered to employees.
reliable evidence that early hominids used fire as (B) Studies have shown that training in stress
many as 500 thousand years ago. management is not effective for many people.
(E) The bone fragments were found in several distinct (C) Regular exercise reduces people’s risk of heart
layers of limestone that contained primitive cutting disease and provides them with increased energy.
tools known to have been used by early hominids. (D) Physical injuries sometimes result from entering a
16. For a trade embargo against a particular country to strenuous physical fitness program too quickly.
succeed, a high degree of both international accord and (E) Employee assistance programs require companies to
ability to prevent goods from entering or leaving that hire people to supervise the various programs
country must be sustained. A total blockade of Patria’s offered.
ports is necessary to an embargo, but such an action
3. Unlike the wholesale price of raw wool, the
would be likely to cause international discord over the
wholesale price of raw cotton has fallen considerably in
embargo.
the last year. Thus, although the retail price of cotton
The claims above, if true, most strongly support which clothing at retail clothing stores has not yet fallen, it will
of the following conclusions? inevitably fall.
(A) The balance of opinion is likely to favor Patria in Which of the following, if true, most seriously weakens
the event of a blockade. the argument above?
(B) As long as international opinion is unanimously (A) The cost of processing raw cotton for cloth has
against Patria, a trade embargo is likely to succeed. increased during the last year.
(C) A naval blockade of Patria’s ports would ensure that (B) The wholesale price of raw wool is typically higher
no goods enter or leave Patria. than that of the same volume of raw cotton.
(D) Any trade embargo against Patria would be likely to (C) The operating costs of the average retail clothing
fail at some time. store have remained constant during the last year.
(E) For a blockade of Patria’s ports to be successful, (D) Changes in retail prices always lag behind changes
international opinion must be unanimous. in wholesale prices.
TEST 14 (E) The cost of harvesting raw cotton has increased in
25 Minutes 16 Questions the last year.

1. The local board of education found that, because the 4. Small-business groups are lobbying to defeat
current physics curriculum has little direct relevance to proposed federal legislation that would substantially raise
today’s world, physics classes attracted few high school the federal minimum wage. This opposition is surprising
students. So to attract students to physics classes, the since the legislation they oppose would, for the first
board proposed a curriculum that emphasizes principles time, exempt all small businesses from paying any
of physics involved in producing and analyzing visual minimum wage.
images. Which of the following, if true, would best explain the
Which of the following, if true, provides the strongest opposition of small-business groups to the proposed
reason to expect that the proposed curriculum will be legislation?
successful in attracting students? (A) Under the current federal minimum-wage law, most
(A) Several of the fundamental principles of physics are small businesses are required to pay no less than
involved in producing and analyzing visual images. the minimum wage to their employees.
(B) Knowledge of physics is becoming increasingly (B) In order to attract workers, small companies must
( 47 )
PaGaLGuYS .com Education
TM
match the wages offered by their larger sufficiently dissimilar to the patented drug.ÿB ÿ
competitors, and these competitors would not be (E) Much recent industrial innovation has occurred in
exempt under the proposed laws. products—for example, in the computer and
(C) The exact number of companies that are currently electronics industries—for which patent protection
required to pay no less than the minimum wage but is often very ineffective.
that would be exempt under the proposed laws is
unknown. Questions 7-8 are based on the following.
(D) Some states have set their own minimum wages—in Bank depositors in the United States are all financially
some cases, quite a bit above the level of the protected against bank failure because the government insures
minimum wage mandated by current federal law— all individuals’ bank deposits. An economist argues that this
for certain key industries. insurance is partly responsible for the high rate of bank
(E) Service companies make up the majority of small failures, since it removes from depositors any financial
businesses and they generally employ more incentive to find out whether the bank that holds their money
employees per dollar of revenues than do retail or is secure against failure. If depositors were more selective,
manufacturing businesses. then banks would need to be secure in order to compete for
depositors’ money.
5. Reviewer: The book Art’s Decline argues that
European painters today lack skills that were common 7. The economist’s argument makes which of the
among European painters of preceding centuries. In this following assumptions?
the book must be right, since its analysis of 100 (A) Bank failures are caused when big borrowers default
paintings, 50 old and 50 contemporary, demonstrates on loan repayments.
convincingly that none of the contemporary paintings are
(B) A significant proportion of depositors maintain
executed as skillfully as the older paintings.
accounts at several different banks.
Which of the following points to the most serious (C) The more a depositor has to deposit, the more
logical flaw in the reviewer’s argument? careful he or she tends to be in selecting a bank.
(A) The paintings chosen by the book’s author for (D) The difference in the interest rates paid to
analysis could be those that most support the depositors by different banks is not a significant
book’s thesis. factor in bank failures.
(B) There could be criteria other than the technical skill (E) Potential depositors are able to determine which
of the artist by which to evaluate a painting. banks are secure against failure.
(C) The title of the book could cause readers to accept
the book’s thesis even before they read the 8. Which of the following, if true, most seriously
analysis of the paintings that supports it. weakens the economist’s argument?
(D) The particular methods currently used by European (A) Before the government started to insure depositors
painters could require less artistic skill than do against bank failure, there was a lower rate of bank
methods used by painters in other parts of the failure than there is now.
world. (B) When the government did not insure deposits,
(E) A reader who was not familiar with the language of frequent bank failures occurred as a result of
art criticism might not be convinced by the book’s depositors’ fears of losing money in bank failures.
analysis of the 100 paintings. (C) Surveys show that a significant proportion of
depositors are aware that their deposits are insured
6. The pharmaceutical industry argues that because by the government.
new drugs will not be developed unless heavy
(D) There is an upper limit on the amount of an
development costs can be recouped in later sales, the
individual’s deposit that the government will
current 20 years of protection provided by patents
insure, but very few individuals’ deposits exceed
should be extended in the case of newly developed drugs.
this limit.
However, in other industries new-product development
continues despite high development costs, a fact that (E) The security of a bank against failure depends on
indicates that the extension is unnecessary. the percentage of its assets that are loaned out and
also on how much risk its loans involve.
Which of the following, if true, most strongly supports
the pharmaceutical industry’s argument against the 9. Passengers must exit airplanes swiftly after
challenge made above? accidents, since gases released following accidents are
(A) No industries other than the pharmaceutical toxic to humans and often explode soon after being
industry have asked for an extension of the 20-year released. In order to prevent passenger deaths from gas
limit on patent protection. inhalation, safety officials recommend that passengers be
(B) Clinical trials of new drugs, which occur after the provided with smoke hoods that prevent inhalation of the
patent is granted and before the new drug can be gases.
marketed, often now take as long as 10 years to Which of the following, if true, constitutes the strongest
complete. reason not to require implementation of the safety
(C) There are several industries in which the ratio of officials’ recommendation?
research and development costs to revenues is (A) Test evacuations showed that putting on the smoke
higher than it is in the pharmaceutical industry. hoods added considerably to the overall time it
(D) An existing patent for a drug does not legally took passengers to leave the cabin.
prevent pharmaceutical companies from bringing to (B) Some airlines are unwilling to buy the smoke hoods
market alternative drugs, provided they are
( 48 )
PaGaLGuYS .com Education
TM
because they consider them to be prohibitively and discomfort of many illnesses.
expensive. (B) Aspirin can prolong a patient’s illness by
(C) Although the smoke hoods protect passengers from eliminating moderate fever helpful in fighting some
the toxic gases, they can do nothing to prevent the diseases.
gases from igniting. (C) Aspirin inhibits the growth of white blood cells,
(D) Some experienced flyers fail to pay attention to the which are necessary for fighting some illnesses.
safety instructions given on every commercial (D) The more white blood cells a patient’s body
flight before takeoff. produces, the less severe the patient’s illness will
(E) In many airplane accidents, passengers who were be.
able to reach emergency exits were overcome by (E) The focus of modern medicine is on inhibiting the
toxic gases before they could exit the airplane. growth of disease-causing bacteria within the body.
10. In 1960, 10 percent of every dollar paid in 13. Because postage rates are rising, Home Decorator
automobile insurance premiums went to pay costs arising magazine plans to maximize its profits by reducing by
from injuries incurred in car accidents. In 1990, 50 one half the number of issues it publishes each year. The
percent of every dollar paid in automobile insurance quality of articles, the number of articles published per
premiums went toward such costs, despite the fact that year, and the subscription price will not change. Market
cars were much safer in 1990 than in 1960. research shows that neither subscribers nor advertisers
Which of the following, if true, best explains the will be lost if the magazine’s plan is instituted.
discrepancy outlined above? Which of the following, if true, provides the strongest
(A) There were fewer accidents in 1990 than in 1960. evidence that the magazine’s profits are likely to decline
(B) On average, people drove more slowly in 1990 than if the plan is instituted?
in 1960. (A) With the new postage rates, a typical issue under
(C) Cars grew increasingly more expensive to repair the proposed plan would cost about one-third
over the period in question. more to mail than a typical current issue would.
(D) The price of insurance increased more rapidly than (B) The majority of the magazine’s subscribers are less
the rate of inflation between 1960 and 1990. concerned about a possible reduction in the
(E) Health-care costs rose sharply between 1960 and quantity of the magazine’s articles than about a
1990. possible loss of the current high quality of its
articles.
11. Caterpillars of all species produce an identical (C) Many of the magazine’s long-time subscribers
hormone called “juvenile hormone” that maintains feeding would continue their subscriptions even if the
behavior. Only when a caterpillar has grown to the right subscription price were increased.
size for pupation to take place does a special enzyme halt (D) Most of the advertisers that purchase advertising
the production of juvenile hormone. This enzyme can be space in the magazine will continue to spend the
synthesized and will, on being ingested by immature same amount on advertising per issue as they have
caterpillars, kill them by stopping them from feeding. in the past.
Which of the following, if true, most strongly supports (E) Production costs for the magazine are expected to
the view that it would not be advisable to try to remain stable.
eradicate agricultural pests that go through a caterpillar
stage by spraying croplands with the enzyme 14. A study of marital relationships in which one
mentioned above? partner’s sleeping and waking cycles differ from those of
(A) Most species of caterpillar are subject to some the other partner reveals that such couples share fewer
natural predation. activities with each other and have more violent
arguments than do couples in a relationship in which both
(B) Many agricultural pests do not go through a
partners follow the same sleeping and waking patterns.
caterpillar stage.
Thus, mismatched sleeping and waking cycles can
(C) Many agriculturally beneficial insects go through a seriously jeopardize a marriage.
caterpillar stage.
Which of the following, if true, most seriously weakens
(D) Since caterpillars of different species emerge at
the argument above?
different times, several sprayings would be
necessary. (A) Married couples in which both spouses follow the
same sleeping and waking patterns also
(E) Although the enzyme has been synthesized in the
occasionally have arguments than can jeopardize
laboratory, no large-scale production facilities exist
the couple’s marriage.
as yet.
(B) The sleeping and waking cycles of individuals tend
12. Although aspirin has been proven to eliminate to vary from season to season.
moderate fever associated with some illnesses, many (C) The individuals who have sleeping and waking
doctors no longer routinely recommend its use for this cycles that differ significantly from those of their
purpose. A moderate fever stimulates the activity of the spouses tend to argue little with colleagues at
body’s disease-fighting white blood cells and also inhibits work.
the growth of many strains of disease-causing bacteria. (D) People in unhappy marriages have been found to
If the statements above are true, which of the following express hostility by adopting a different sleeping
conclusions is most strongly supported by them? and waking cycle from that of their spouses.
(A) Aspirin, an effective painkiller, alleviates the pain (E) According to a recent study, most people’s sleeping

( 49 )
PaGaLGuYS .com Education
TM
and waking cycles can be controlled and modified been declining because of the introduction of advanced
easily. manufacturing technology (AMT). Given the rate at
which AMT is currently being introduced in
Questions 15-16 are based on the following. manufacturing, the average worker’s old skills become
obsolete and new skills are required within as little as five
Roland: The alarming fact is that 90 percent of the people in
years.
this country now report that they know someone who is
unemployed. Which of the following plans, if feasible, would allow a
company to prepare most effectively for the rapid
Sharon: But a normal, moderate level of unemployment is 5 obsolescence of skills described above?
percent, with 1 out of 20 workers unemployed. So at any (A) The company will develop a program to offer
given time if a person knows approximately 50 workers, 1 or selected employees the opportunity to receive
more will very likely be unemployed. training six years after they were originally hired.
15. Sharon’s argument is structured to lead to which of (B) The company will increase its investment in AMT
the following as a conclusion? every year for a period of at least five years.
(A) The fact that 90% of the people know someone (C) The company will periodically survey its
who is unemployed is not an indication that employees to determine how the introduction of
unemployment is abnormally high. AMT has affected them.
(B) The current level of unemployment is not moderate. (D) Before the introduction of AMT, the company will
(C) If at least 5% of workers are unemployed, the result institute an educational program to inform its
of questioning a representative group of people employees of the probable consequences of the
cannot be the percentage Roland cites. introduction of AMT.
(D) It is unlikely that the people whose statements (E) The company will ensure that it can offer its
Roland cites are giving accurate reports. employees any training necessary for meeting their
job requirements.
(E) If an unemployment figure is given as a certain
percent, the actual percentage of those without 3. Installing scrubbers in smokestacks and switching
jobs is even higher. to cleaner-burning fuel are the two methods available to
Northern Power for reducing harmful emissions from its
16. Sharon’s argument relies on the assumption that plants. Scrubbers will reduce harmful emissions more
(A) normal levels of unemployment are rarely exceeded than cleaner-burning fuels will. Therefore, by installing
(B) unemployment is not normally concentrated in scrubbers, Northern Power will be doing the most that
geographically isolated segments of the population can be done to reduce harmful emissions from its plants.
(C) the number of people who each know someone who Which of the following is an assumption on which the
is unemployed is always higher than 90% of the argument depends?
population (A) Switching to cleaner-burning fuel will not be more
(D) Roland is not consciously distorting the statistics expensive than installing scrubbers.
he presents (B) Northern Power can choose from among various
(E) knowledge that a personal acquaintance is kinds of scrubbers, some of which are more
unemployed generates more fear of losing one’s job effective than others.
than does knowledge of unemployment statistics
(C) Northern Power is not necessarily committed to
TEST 15 reducing harmful emissions from its plants.
25 Minutes 16 Questions (D) Harmful emissions from Northern Power’s plants
cannot be reduced more by using both methods
1. A company is considering changing its policy together than by the installation of scrubbers alone.
concerning daily working hours. Currently, this company
(E) Aside from harmful emissions from the smokestacks
requires all employees to arrive at work at 8 a.m. The
of its plants, the activities of Northern Power do
proposed policy would permit each employee to decide
not cause significant air pollution.
when to arrive—from as early as 6 a.m. to as late as 11
a.m. 4. Some anthropologists study modern-day societies
The adoption of this policy would be most likely to of foragers in an effort to learn about our ancient
decrease employees’ productivity if the employees’ job ancestors who were also foragers. A flaw in this strategy
functions required them to is that forager societies are extremely varied. Indeed, any
(A) work without interruption from other employees forager society with which anthropologists are familiar
has had considerable contact with modern nonforager
(B) consult at least once a day with employees from
societies.
other companies
(C) submit their work for a supervisor’s eventual Which of the following, if true, would most weaken the
approval criticism made above of the anthropologists’ strategy?
(D) interact frequently with each other throughout the (A) All forager societies throughout history have had a
entire workday number of important features in common that are
absent from other types of societies.
(E) undertake projects that take several days to
complete (B) Most ancient forager societies either dissolved or
made a transition to another way of life.
2. The amount of time it takes for most of a worker’s (C) All anthropologists study one kind or another of
occupational knowledge and skills to become obsolete has modern-day society.
( 50 )
PaGaLGuYS .com Education
TM
(D) Many anthropologists who study modern-day (A) In March, many grocers removed apples from their
forager societies do not draw inferences about shelves in order to demonstrate concern about their
ancient societies on the basis of their studies. customers’ health.
(E) Even those modern-day forager societies that have (B) Because of a growing number of food-safety
not had significant contact with modern societies warnings, consumers in March were indifferent to
are importantly different from ancient forager such warnings.
societies. (C) The report was delivered on television and also
appeared in newspapers.
5. Mayor: In each of the past five years, the city has
cut school funding and each time school officials (D) The report did not mention that any other fruit
complained that the cuts would force them to reduce contains Alar, although the preservative is used on
expenditures for essential services. But each time, only other fruit.
expenditures for nonessential services were actually (E) Public health officials did not believe that apples
reduced. So school officials can implement further cuts posed a health threat because only minute traces of
without reducing any expenditures for essential services. Alar were present in affected apples.
Which of the following, if true, most strongly supports 8. A new law gives ownership of patents—documents
the mayor’s conclusion? providing exclusive right to make and sell an invention—
(A) The city’s schools have always provided essential to universities, not the government, when those patents
services as efficiently as they have provided result from government-sponsored university research.
nonessential services. Administrators at Logos University plan to sell any
(B) Sufficient funds are currently available to allow the patents they acquire to corporations in order to fund
city’s schools to provide some nonessential programs to improve undergraduate teaching.
services. Which of the following, if true, would cast most doubt
(C) Price estimates quoted to the city’s schools for the on the viability of the college administrators’ plan
provision of nonessential services have not described above?
increased substantially since the most recent (A) Profit-making corporations interested in developing
school funding cut. products based on patents held by universities are
(D) Few influential city administrators support the likely to try to serve as exclusive sponsors of
funding of costly nonessential services in the city’s ongoing university research projects.
schools. (B) Corporate sponsors of research in university
(E) The city’s school officials rarely exaggerate the facilities are entitled to tax credits under new
potential impact of threatened funding cuts. federal tax-code guidelines.
(C) Research scientists at Logos University have few or
6. Advertisement: For sinus pain, three out of four
no teaching responsibilities and participate little if
hospitals give their patients Novex. So when you want
at all in the undergraduate programs in their field.
the most effective painkiller for sinus pain, Novex is the
one to choose. (D) Government-sponsored research conducted at
Logos University for the most part duplicates
Which of the following, if true, most seriously research already completed by several profit-
undermines the advertisement’s argument? making corporations.
(A) Some competing brands of painkillers are intended (E) Logos University is unlikely to attract corporate
to reduce other kinds of pain in addition to sinus sponsorship of its scientific research.
pain.
(B) Many hospitals that do not usually use Novex will 9. Contrary to earlier predictions, demand for
do so for those patients who cannot tolerate the sugarcane has not increased in recent years. Yet, even
drug the hospitals usually use. though prices and production amounts have also been
(C) Many drug manufacturers increase sales of their stable during the last three years, sugarcane growers last
products to hospitals by selling these products to year increased their profits by more than ten percent over
the hospitals at the lowest price the manufacturers the previous year’s level.
can afford. Any of the following statements, if true, about last
(D) Unlike some competing brands of painkillers, year, helps to explain the rise in profits EXCEPT:
Novex is available from pharmacies without a (A) Many countries that are large consumers of
doctor’s prescription. sugarcane increased their production of sugarcane-
(E) In clinical trials Novex has been found more based ethanol, yet their overall consumption of
effective than competing brands of painkillers that sugarcane decreased.
have been on the market longer than Novex. (B) Sugarcane growers have saved money on wages by
switching from paying laborers an hourly wage to
7. A report that many apples contain a cancer-causing paying them by the amount harvested.
preservative called Alar apparently had little effect on
(C) The price of oil, the major energy source used by
consumers. Few consumers planned to change their
sugarcane growers in harvesting their crops,
apple-buying habits as a result of the report.
dropped by over twenty percent.
Nonetheless, sales of apples in grocery stores fell sharply
in March, a month after the report was issued. (D) Many small sugarcane growers joined together to
form an association of sugarcane producers and
Which of the following, if true, best explains the reason began to buy supplies at low group rates.
for the apparent discrepancy described above?
(E) Rainfall in sugarcane-growing regions was higher

( 51 )
PaGaLGuYS .com Education
TM
than it had been during the previous year, allowing exactly identical prints. Such systems can be adjusted to
the growers to save money on expensive artificial minimize refusals of access to legitimate access-seekers.
irrigation. Such adjustments, however, increase the likelihood of
admitting impostors.
10. If the county continues to collect residential trash at
current levels, landfills will soon be overflowing and Which of the following conclusions is most strongly
parkland will need to be used in order to create more supported by the information above?
space. Charging each household a fee for each pound of (A) If a biometric access-control system were made to
trash it puts out for collection will induce residents to work by identity, it would not produce any correct
reduce the amount of trash they create; this charge will admittance decisions.
therefore protect the remaining county parkland. (B) If a biometric access-control system reliably
Which of the following is an assumption made in prevents impostors from being admitted, it will
drawing the conclusion above? sometimes turn away legitimate access-seekers.
(A) Residents will reduce the amount of trash they put (C) Biometric access-control systems are appropriate
out for collection by reducing the number of only in situations in which admittance of
products they buy. impostors is less of a problem than is mistaken
refusal of access.
(B) The collection fee will not significantly affect the
purchasing power of most residents, even if their (D) Nonbiometric access-control systems—based, for
households do not reduce the amount of trash they example, on numerical codes—are less likely than
put out. biometric ones to admit impostors.
(C) The collection fee will not induce residents to dump (E) Anyone choosing an access-control system should
their trash in the parklands illegally. base the choice solely on the ratio of false refusals
to false admittances.
(D) The beauty of county parkland is an important
issue for most of the county’s residents. 14. Although computers can enhance people’s ability to
(E) Landfills outside the county’s borders could be used communicate, computer games are a cause of
as dumping sites for the county’s trash. underdeveloped communication skills in children. After-
school hours spent playing computer games are hours not
Questions 11-12 are based on the following. spent talking with people. Therefore, children who spend
all their spare time playing these games have less
Environmentalist: The commissioner of the Fish and Game
experience in interpersonal communication than other
Authority would have the public believe that increases in the
children have.
number of marine fish caught demonstrate that this resource
is no longer endangered. This is a specious argument, as The argument depends on which of the following
unsound as it would be to assert that the ever-increasing rate assumptions?
at which rain forests are being cut down demonstrates a lack (A) Passive activities such as watching television and
of danger to that resource. The real cause of the increased listening to music do not hinder the development
fish-catch is a greater efficiency in using technologies that of communication skills in children.
deplete resources. (B) Most children have other opportunities, in addition
to after-school hours, in which they can choose
11. Which of the following strategies is used in the
whether to play computer games or to interact
presentation of the environmentalist’s position?
with other people.
(A) Questioning the motives of an opponent
(C) Children who do not spend all of their after-school
(B) Showing that an opposing position is self- hours playing computer games spend at least some
contradictory of that time talking with other people.
(C) Attacking an argument through the use of an analogy (D) Formal instruction contributes little or nothing to
(D) Demonstrating the inaccuracy of certain data children’s acquisition of communication skills.
(E) Pointing out adverse consequences of a proposal (E) The mental skills developed through playing
computer games do not contribute significantly to
12. The environmentalist’s statements, if true, best children’s intellectual development.
support which of the following as a conclusion?
(A) The use of technology is the reason for the 15. One variety of partially biodegradable plastic
increasing encroachment of people on nature. beverage container is manufactured from small bits of
(B) It is possible to determine how many fish are in the plastic bound together by a degradable bonding agent such
sea in some way other than by catching fish. as cornstarch. Since only the bonding agent degrades,
(C) The proportion of marine fish that are caught is as leaving the small bits of plastic, no less plastic refuse per
high as the proportion of rain-forest trees that are container is produced when such containers are discarded
cut down each year. than when comparable nonbiodegradable containers are
discarded.
(D) Modern technologies waste resources by catching
inedible fish. Which of the following, if true, most strengthens the
argument above?
(E) Marine fish continue to be an endangered resource.
(A) Both partially biodegradable and nonbiodegradable
13. Biometric access-control systems—those using plastic beverage containers can be crushed
fingerprints, voiceprints, etc., to regulate admittance to completely flat by refuse compactors.
restricted areas—work by degrees of similarity, not by (B) The partially biodegradable plastic beverage
identity. After all, even the same finger will rarely leave containers are made with more plastic than

( 52 )
PaGaLGuYS .com Education
TM
comparable nonbiodegradable ones in order to things would fall off the shelves’ edges into the rear
compensate for the weakening effect of the corners.
bounding agents. Which of the following is presupposed in the argument
(C) Many consumers are ecology-minded and prefer to against introducing rotating shelves?
buy a product sold in the partially biodegradable (A) Refrigerators would not be made so that their
plastic beverage containers rather than in interior space is cylindrical.
nonbiodegradable containers, even if the price is
(B) Refrigerators would not be made to have a window
higher.
in front for easy viewing of their contents without
(D) The manufacturing process for the partially opening the door.
biodegradable plastic beverage containers results in
(C) The problem of spoilage of refrigerated food is not
less plastic waste than the manufacturing process
amenable to any solution based on design changes.
for nonbiodegradable plastic beverage containers.
(D) Refrigerators are so well designed that there are
(E) Technological problems with recycling currently
bound to be drawbacks to any design change.
prevent the reuse as food or beverage containers of
the plastic from either type of plastic beverage (E) Rotating shelves would be designed to rotate only
container. while the refrigerator door was open.

16. Commentator: The theory of trade retaliation states 3. It would cost Rosetown one million dollars to
that countries closed out of any of another country’s repair all of its roads. In the year after completion of
markets should close some of their own markets to the those repairs, however, Rosetown would thereby avoid
other country in order to pressure the other country to incurring three million dollars worth of damages, since
reopen its markets. If every country acted according to currently Rosetown pays that amount annually in
this theory, no country would trade with any other. compensation for damage done to cars each year by its
unrepaired roads.
The commentator’s argument relies on which of the
following assumptions? Which of the following, if true, gives the strongest
support to the argument above?
(A) No country actually acts according to the theory of
trade retaliation. (A) Communities bordering on Rosetown also pay
compensation for damage done to cars by their
(B) No country should block any of its markets to
unrepaired roads.
foreign trade.
(B) After any Rosetown road has been repaired, several
(C) Trade disputes should be settled by international
years will elapse before that road begins to damage
tribunal.
cars.
(D) For any two countries, at least one has some market
(C) Rosetown would need to raise additional taxes if it
closed to the other.
were to spend one million dollars in one year on
(E) Countries close their markets to foreigners to road repairs.
protect domestic producers.
(D) The degree of damage caused to Rosetown’s roads
TEST 16 by harsh weather can vary widely from year to
25 Minutes 16 Questions year.
(E) Trucks cause much of the wear on Rosetown’s
1. The chanterelle, a type of wild mushroom, grows
roads, but owners of cars file almost all of the
beneath host trees such as the Douglas fir, which provide
claims for compensation for damage caused by
it with necessary sugars. The underground filaments of
unrepaired roads.
chanterelles, which extract the sugars, in turn provide
nutrients and water for their hosts. Because of this 4. Two experimental garden plots were each planted
mutually beneficial relationship, harvesting the with the same number of tomato plants. Magnesium salts
chanterelles growing beneath a Douglas fir seriously were added to the first plot but not to the second. The
endangers the tree. first plot produced 20 pounds of tomatoes and the
Which of the following, if true, casts the most doubt on second plot produced 10 pounds. Since nothing else but
the conclusion drawn above? water was added to either plot, the higher yields in the
(A) The number of wild mushrooms harvested has first plot must have been due to the magnesium salts.
increased in recent years. Which of the following, if true, most seriously weakens
(B) Chanterelles grow not only beneath Douglas firs but the argument above?
also beneath other host trees. (A) A small amount of the magnesium salts from the
(C) Many types of wild mushrooms are found only in first plot leached into the second plot.
forests and cannot easily be grown elsewhere. (B) Tomato plants in a third experimental plot, to which
(D) The harvesting of wild mushrooms stimulates a high-nitrogen fertilizer was added, but no
future growth of those mushrooms. magnesium salts, produced 15 pounds of tomatoes.
(E) Young Douglas fir seedlings die without the (C) Four different types of tomatoes were grown in
nutrients and water provided by chanterelle equal proportions in each of the plots.
filaments. (D) Some weeds that compete with tomatoes cannot
tolerate high amounts of magnesium salts in the
2. The reason much refrigerated food spoils is that it soil.
ends up out of sight at the back of the shelf. So why not
(E) The two experimental plots differed from each other
have round shelves that rotate? Because such rotating
with respect to soil texture and exposure to
shelves would have just the same sort of drawback, since
sunlight.
( 53 )
PaGaLGuYS .com Education
TM
5. Archaeologists have found wheeled ceramic toys extremely wide range of competing products.
made by the Toltec, twelfth-century inhabitants of what (B) Many consumers make decisions about which
is now Veracruz. Although there is no archaeological wines to purchase on the basis of reviews of wine
evidence that the Toltec used wheels for anything but published in books and periodicals.
toys, some anthropologists hypothesize that wheeled (C) Consumers selecting wine in a store often use the
utility vehicles were used to carry materials needed for price charged as their main guide to the wine’s
the monumental structures the Toltec produced. quality.
Which of the following, if true, would most help the (D) Wine retailers and producers can generally increase
anthropologists explain the lack of evidence noted the sales of a particular wine temporarily by
above? introducing a price discount.
(A) The Toltec sometimes incorporated into their toys (E) Consumers who purchase wine regularly generally
representations of utensils or other devices that have strong opinions about which wines they
served some practical purpose. prefer.
(B) Any wheeled utility vehicles used by the Toltec
could have been made entirely of wood, and unlike 8. The recent decline in land prices has hurt many
ceramic, wood decays rapidly in the humid climate institutions that had invested heavily in real estate. Last
of Veracruz. year, before the decline began, a local college added 2,000
acres to its holdings. The college, however, did not
(C) Carvings in monument walls suggest that the
purchase the land but received it as a gift. Therefore the
Toltec’s wheeled ceramic toys sometimes had ritual
price decline will probably not affect the college.
uses in addition to being used by both children and
adults as decorations and playthings. Which of the following, if true, casts most doubt on the
(D) Wheeled utility vehicles were used during the conclusion above?
twelfth century in many areas of the world, but (A) The 2,000 acres that the college was given last year
during this time wheeled toys were not very are located within the same community as the
common in areas outside Veracruz college itself.
(E) Some of the wheeled ceramic toys were found near (B) The college usually receives more contributions of
the remains of monumental structures. money than of real estate.
(C) Land prices in the region in which the college is
6. Demographers doing research for an international located are currently higher than the national
economics newsletter claim that the average per capita average.
income in the country of Kuptala is substantially lower
(D) Last year, the amount that the college allocated to
than that in the country of Bahlton. They also claim,
pay for renovations included money it expected to
however, that whereas poverty is relatively rare in
receive by selling some of its land this year.
Kuptala, over half the population of Bahlton lives in
extreme poverty. At least one of the demographers’ claims (E) Last year, the college paid no property taxes on land
must, therefore, be wrong. occupied by college buildings but instead paid fees
to compensate the local government for services
The argument above is most vulnerable to which of the provided.
following criticisms?
(A) It rejects an empirical claim about the average per 9. Civil trials often involve great complexities that are
capita incomes in the two countries without beyond the capacities of jurors to understand. As a result,
making any attempt to discredit that claim by jurors’ decisions in such trials are frequently incorrect.
offering additional economic evidence. Justice would therefore be better served if the more
(B) It treats the vague term “poverty” as though it had a complex trials were decided by judges rather than juries.
precise and universally accepted meaning. The argument above depends on which of the following
(C) It overlooks the possibility that the number of assumptions?
people in the two countries who live in poverty (A) A majority of civil trials involve complexities that
could be the same even though the percentages of jurors are not capable of understanding.
the two populations that live in poverty differ (B) The judges who would decide complex civil trials
markedly. would be better able to understand the
(D) It fails to show that wealth and poverty have the complexities of those trials than jurors are.
same social significance in Kuptala as in Bahlton. (C) The judges who would preside over civil trials
(E) It does not consider the possibility that incomes in would disallow the most complex sorts of evidence
Kuptala, unlike those in Bahlton, might all be very from being introduced into those trials.
close to the country’s average per capita income. (D) Jurors’ decisions are frequently incorrect even in
those civil trials that do not involve great
7. Normally, increases in the price of a product
complexities.
decrease its sales except when the price increase
accompanies an improvement in the product. Wine is (E) The sole reason in favor of having juries decide civil
unusual, however. Often increases in the price of a trials is the supposition that their decisions will
particular producer’s wine will result in increased sales, almost always be correct.
even when the wine itself is unchanged.
10. Some species of dolphins find their prey by
Which of the following, if true, does most to explain the echolocation; they emit clicking sounds and listen for
anomaly described above? echoes returning from distant objects in the water. Marine
(A) The retail wine market is characterized by an biologists have speculated that those same clicking

( 54 )
PaGaLGuYS .com Education
TM
sounds might have a second function: particularly loud coated skin.
clicks might be used by the dolphins to stun their prey at
close range through sensory overload. 13. A severe drought can actually lessen the total
amount of government aid that United States farmers
Which of the following, if discovered to be true, would receive as a group. The government pays farmers the
cast the most serious doubt on the correctness of the amount, if any, by which the market price at which crops
speculation described above? are actually sold falls short of a preset target price per
(A) Dolphins that use echolocation to locate distant bushel for the crops. The drought of 1983, for example,
prey also emit frequent clicks at intermediate caused farm-program payments to drop by $10 billion.
distances as they close in on their prey.
Given the information above, which of the following, if
(B) The usefulness of echolocation as a means of true, best explains why the drought of 1983 resulted in
locating prey depends on the clicking sounds being a reduction in farm-program payments?
of a type that the prey is incapable of perceiving,
(A) Prior to the drought of 1983, the government raised
regardless of volume.
the target price for crops in order to aid farmers in
(C) If dolphins stun their prey, the effect is bound to be reducing their debt loads.
so temporary that stunning from far away, even if
(B) Due to the drought of 1983, United States farmers
possible, would be ineffective.
exported less food in 1983 than in the preceding
(D) Echolocation appears to give dolphins that use it year.
information about the richness of a source of food
(C) Due to the drought of 1983, United States farmers
as well as about its direction.
had smaller harvests and thus received a higher
(E) The more distant a dolphin’s prey, the louder the market price for the 1983 crop than for the larger
echolocation clicks must be if they are to reveal the crop of the preceding year.
prey’s presence to the hunting dolphin.
(D) Due to the drought of 1983, United States farmers
11. Advertisement: The world’s best coffee beans come planned to plant smaller crops in 1984 than they
from Colombia. The more Colombian beans in a blend of had in 1983.
coffee, the better the blend, and no company purchases (E) Despite the drought of 1983, retail prices for food
more Colombian beans than Kreemo Coffee, Inc. So it did not increase significantly between 1982 and
only stands to reason that if you buy a can of Kreemo’s 1983.
coffee, you’re buying the best blended coffee available
today. 14. In order to increase revenues, an airport plans to
change the parking fees it charges at its hourly parking
The reasoning of the argument in the advertisement is lots. Rather than charging $2.00 for the first two-hour
flawed because it overlooks the possibility that period, or part thereof, and $1.00 for each hour
(A) the equipment used by Kreemo to blend and thereafter, the airport will charge $4.00 for the first four-
package its coffee is no different from that used by hour period, or part thereof, and $1.00 for each hour
most other coffee producers thereafter.
(B) not all of Kreemo’s competitors use Colombian Which of the following is a consideration that, if true,
coffee beans in the blends of coffee they sell suggests that the plan will be successful in increasing
(C) Kreemo sells more coffee than does any other revenues?
company (A) Very few people who park their cars at the hourly
(D) Kreemo’s coffee is the most expensive blended parking lot at the airport leave their cars for more
coffee available today than two hours at a time.
(E) the best unblended coffee is better than the best (B) Over the past several years, the cost to the airport
blended coffee of operating its hourly parking facilities has been
greater than the revenues it has received from them.
12. The only purpose for which a particular type of
tape is needed is to hold certain surgical wounds closed (C) People who leave their cars at the airport while on a
for ten days—the maximum time such wounds need tape. trip generally park their cars in lots that charge by
Newtape is a new brand of this type of tape. Newtape’s the day rather than by the hour.
salespeople claim that Newtape will improve healing (D) A significant portion of the money spent to operate
because Newtape adheres twice as long as the currently the airport parking lot is spent to maintain the
used tape does. facilities rather than to pay the salaries of the
personnel who collect the parking fees.
Which of the following statements, if true, would most
seriously call into question the claim made by (E) The hourly parking lots at the airport have recently
Newtape’s salespeople? been expanded and are therefore rarely filled to
capacity.
(A) Most surgical wounds take about ten days to heal.
(B) Most surgical tape is purchased by hospitals and 15. In the course of her researches, a historian recently
clinics rather than by individual surgeons. found two documents mentioning the same person, Erich
(C) The currently used tape’s adhesiveness is more than Schnitzler. One, dated May 3, 1739, is a record of
sufficient to hold wounds closed for ten days. Schnitzler’s arrest for peddling without a license. The
(D) Neither Newtape nor the currently used tape second, undated, is a statement by Schnitzler asserting
adheres well to skin that has not been cleaned. that he has been peddling off and on for 20 years.
(E) Newtape’s adhesion to skin that has been coated The facts above best support which of the following
with a special chemical preparation is only half as conclusions?
good as the currently used tape’s adhesion to such (A) Schnitzler started peddling around 1719.
( 55 )
PaGaLGuYS .com Education
TM
(B) Schnitzler was arrested repeatedly for peddling. the argument above?
(C) The undated document was written before 1765. (A) Last year’s best-selling airline seat was not the
(D) The arrest record was written after the undated safest airline seat on the market.
document. (B) No airline company has announced that it would be
(E) The arrest record provides better evidence that making safe seating a higher priority this year.
Schnitzler peddled than does the undated (C) The price of fuel was higher this year than it had
document. been in most of the years when the safest airline
seats sold poorly.
16. The recent upheaval in the office-equipment retail
(D) Because of increases in the cost of materials, all
business, in which many small firms have gone out of
airline seats were more expensive to manufacture
business, has been attributed to the advent of office
this year than in any previous year.
equipment “superstores” whose high sales volume keeps
their prices low. This analysis is flawed, however, since (E) Because of technological innovations, the safest
even today the superstores control a very small share of airline seat on the market this year weighed less
the retail market. than most other airline seats on the market.

Which of the following, if true, would most weaken the 3. A computer equipped with signature-recognition
argument that the analysis is flawed? software, which restricts access to a computer to those
(A) Most of the larger customers for office equipment people whose signatures are on file, identifies a person’s
purchase under contract directly from signature by analyzing not only the form of the signature
manufacturers and thus do not participate in the but also such characteristics as pen pressure and signing
retail market. speed. Even the most adept forgers cannot duplicate all of
(B) The superstores’ heavy advertising of their low the characteristics the program analyzes.
prices has forced prices down throughout the retail Which of the following can be logically concluded from
market for office supplies. the passage above?
(C) Some of the superstores that only recently opened (A) The time it takes to record and analyze a signature
have themselves gone out of business. makes the software impractical for everyday use.
(D) Most of the office equipment superstores are (B) Computers equipped with the software will soon be
owned by large retailing chains that also own installed in most banks.
stores selling other types of goods. (C) Nobody can gain access to a computer equipped
(E) The growing importance of computers in most with the software solely by virtue of skill at
offices has changed the kind of office equipment forging signatures.
retailers must stock. (D) Signature-recognition software has taken many
years to develop and perfect.
TEST 17
25 Minutes 16 Questions (E) In many cases even authorized users are denied
legitimate access to computers equipped with the
1. A report on acid rain concluded, “Most forests in software.
Canada are not being damaged by acid rain.” Critics of the
report insist the conclusion be changed to, “Most forests 4. Division manager: I want to replace the Microton
in Canada do not show visible symptoms of damage by computers in my division with Vitech computers.
acid rain, such as abnormal loss of leaves, slower rates of General manager: Why?
growth, or higher mortality.”
Division manager: It costs 28 percent less to train new
Which of the following, if true, provides the best logical staff on the Vitech.
justification for the critics’ insistence that the report’s
General manager: But that is not a good enough reason.
conclusion be changed?
We can simply hire only people who already know how
(A) Some forests in Canada are being damaged by acid to use the Microton computer.
rain.
Which of the following, if true, most seriously
(B) Acid rain could be causing damage for which
undermines the general manager’s objection to the
symptoms have not yet become visible.
replacement of Microton computers with Vitechs?
(C) The report does not compare acid rain damage to
(A) Currently all employees in the company are
Canadian forests with acid rain damage to forests
required to attend workshops on how to use
in other countries.
Microton computers in new applications.
(D) All forests in Canada have received acid rain during
(B) Once employees learn how to use a computer, they
the past fifteen years.
tend to change employers more readily than before.
(E) The severity of damage by acid rain differs from
(C) Experienced users of Microton computers command
forest to forest.
much higher salaries than do prospective
2. In the past most airline companies minimized employees who have no experience in the use of
aircraft weight to minimize fuel costs. The safest airline computers.
seats were heavy, and airlines equipped their planes with (D) The average productivity of employees in the
few of these seats. This year the seat that has sold best general manager’s company is below the average
to airlines has been the safest one—a clear indication that productivity of the employees of its competitors.
airlines are assigning a higher priority to safe seating than (E) The high costs of replacement parts make Vitech
to minimizing fuel costs. computers more expensive to maintain than
Which of the following, if true, most seriously weakens Microton computers.
( 56 )
PaGaLGuYS .com Education
TM
5. An airplane engine manufacturer developed a new individual workers’ relationships to the firm,
engine model with safety features lacking in the earlier negotiating the introduction of PRP contracts is
model, which was still being manufactured. During the complex and time consuming.
first year that both were sold, the earlier model far (D) Many firms in the metalworking industry have
outsold the new model; the manufacturer thus concluded modernized production equipment in the last five
that safety was not the customers’ primary consideration. years, and most of these introduced PRP contracts
Which of the following, if true, would most seriously at the same time.
weaken the manufacturer’s conclusion? (E) In firms in the metalworking industry where PRP
(A) Both private plane owners and commercial airlines contracts are in place, the average take-home pay is
buy engines from this airplane engine manufacturer. 15 percent higher than it is in those firms where
(B) Many customers consider earlier engine models workers have more traditional contracts.
better safety risks than new engine models, since 8. Crops can be traded on the futures market before
more is usually known about the safety of the they are harvested. If a poor corn harvest is predicted,
earlier models. prices of corn futures rise; if a bountiful corn harvest is
(C) Many customers of this airplane engine predicted, prices of corn futures fall. This morning
manufacturer also bought airplane engines from meteorologists are predicting much-needed rain for the
manufacturers who did not provide additional corn-growing region starting tomorrow. Therefore, since
safety features in their newer models. adequate moisture is essential for the current crop’s
(D) The newer engine model can be used in all planes in survival, prices of corn futures will fall sharply today.
which the earlier engine model can be used. Which of the following, if true, most weakens the
(E) There was no significant difference in price between argument above?
the newer engine model and the earlier engine (A) Corn that does not receive adequate moisture during
model. its critical pollination stage will not produce a
6. Between 1975 and 1985, nursing-home occupancy bountiful harvest.
rates averaged 87 percent of capacity, while admission (B) Futures prices for corn have been fluctuating more
rates remained constant, at an average of 95 admissions dramatically this season than last season.
per 1,000 beds per year. Between 1985 and 1988, (C) The rain that meteorologists predicted for tomorrow
however, occupancy rates rose to an average of 92 is expected to extend well beyond the corn-growing
percent of capacity, while admission rates declined to 81 region.
per 1,000 beds per year. (D) Agriculture experts announced today that a disease
If the statements above are true, which of the following that has devastated some of the corn crop will
conclusions can be most properly drawn? spread widely before the end of the growing
(A) The average length of time nursing-home residents season.
stayed in nursing homes increased between 1985 (E) Most people who trade in corn futures rarely take
and 1988. physical possession of the corn they trade.
(B) The proportion of older people living in nursing 9. A discount retailer of basic household necessities
homes was greater in 1988 than in 1975. employs thousands of people and pays most of them at
(C) Nursing home admission rates tend to decline the minimum wage rate. Yet following a federally
whenever occupancy rates rise. mandated increase of the minimum wage rate that
(D) Nursing homes built prior to 1985 generally had increased the retailer’s operating costs considerably, the
fewer beds than did nursing homes built between retailer’s profits increased markedly.
1985 and 1988. Which of the following, if true, most helps to resolve
(E) The more beds a nursing home has, the higher its the apparent paradox?
occupancy rate is likely to be. (A) Over half of the retailer’s operating costs consist of
7. Firms adopting “profit-related-pay” (PRP) payroll expenditures; yet only a small percentage
contracts pay wages at levels that vary with the firm’s of those expenditures go to pay management
profits. In the metalworking industry last year, firms salaries.
with PRP contracts in place showed productivity per (B) The retailer’s customer base is made up primarily of
worker on average 13 percent higher than that of their people who earn, or who depend on the earnings of
competitors who used more traditional contracts. others who earn, the minimum wage.
If, on the basis of the evidence above, it is argued that (C) The retailer’s operating costs, other than wages,
PRP contracts increase worker productivity, which of increased substantially after the increase in the
the following, if true, would most seriously weaken that minimum wage rate went into effect.
argument? (D) When the increase in the minimum wage rate went
(A) Results similar to those cited for the metalworking into effect, the retailer also raised the wage rate for
industry have been found in other industries where employees who had been earning just above
PRP contracts are used. minimum wage.
(B) Under PRP contracts costs other than labor costs, (E) The majority of the retailer’s employees work as
such as plant, machinery, and energy, make up an cashiers, and most cashiers are paid the minimum
increased proportion of the total cost of each unit wage.
of output. 10. The cotton farms of Country Q became so
(C) Because introducing PRP contracts greatly changes productive that the market could not absorb all that they
( 57 )
PaGaLGuYS .com Education
TM
produced. Consequently, cotton prices fell. The scientists assumes that
government tried to boost cotton prices by offering (A) serious scientific research is not a solitary activity,
farmers who took 25 percent of their cotton acreage out but relies on active cooperation among a group of
of production direct support payments up to a specified colleagues
maximum per farm. (B) research scientists tend not to regard as colleagues
The government’s program, if successful, will not be a those scientists whose renown they envy
net burden on the budget. Which of the following, if (C) a scientist can become a famous popularizer
true, is the best basis for an explanation of how this without having completed any important research
could be so? (D) research scientists believe that those who are well
(A) Depressed cotton prices meant operating losses for known as popularizers of science are not
cotton farms, and the government lost revenue motivated to do important new research
from taxes on farm profits. (E) no important new research can be accessible to or
(B) Cotton production in several counties other than Q accurately assessed by those who are not
declined slightly the year that the support- themselves scientists
payment program went into effect in Q.
(C) The first year that the support-payment program 13. Mouth cancer is a danger for people who rarely
was in effect, cotton acreage in Q was 5% below brush their teeth. In order to achieve early detection of
its level in the base year for the program. mouth cancer in these individuals, a town’s public health
officials sent a pamphlet to all town residents, describing
(D) The specified maximum per farm meant that for
how to perform weekly self-examinations of the mouth
very large cotton farms the support payments
for lumps.
were less per acre for those acres that were
withdrawn from production than they were for Which of the following, if true, is the best criticism of
smaller farms. the pamphlet as a method of achieving the public health
(E) Farmers who wished to qualify for support officials’ goal?
payments could not use the cotton acreage that (A) Many dental diseases produce symptoms that
was withdrawn from production to grow any other cannot be detected in a weekly self-examination.
crop. (B) Once mouth cancer has been detected, the
effectiveness of treatment can vary from person to
11. United States hospitals have traditionally relied person.
primarily on revenues from paying patients to offset
(C) The pamphlet was sent to all town residents,
losses from unreimbursed care. Almost all paying patients
including those individuals who brush their teeth
now rely on governmental or private health insurance to
regularly.
pay hospital bills. Recently, insurers have been strictly
limiting what they pay hospitals for the care of insured (D) Mouth cancer is much more common in adults than
patients to amounts at or below actual costs. in children.
(E) People who rarely brush their teeth are unlikely to
Which of the following conclusions is best supported
perform a weekly examination of their mouth.
by the information above?
(A) Although the advance of technology has made 14. Technological improvements and reduced
expensive medical procedures available to the equipment costs have made converting solar energy
wealthy, such procedures are out of the reach of directly into electricity far more cost-efficient in the last
low-income patients. decade. However, the threshold of economic viability for
(B) If hospitals do not find ways to raising additional solar power (that is, the price per barrel to which oil
income for unreimbursed care, they must either would have to rise in order for new solar power plants to
deny some of that care or suffer losses if they give be more economical than new oil-fired power plants) is
it. unchanged at thirty-five dollars.
(C) Some patients have incomes too high for eligibility Which of the following, if true, does most to help
for governmental health insurance but are unable to explain why the increased cost-efficiency of solar
afford private insurance for hospital care. power has not decreased its threshold of economic
(D) If the hospitals reduce their costs in providing care, viability?
insurance companies will maintain the current level (A) The cost of oil has fallen dramatically.
of reimbursement, thereby providing more funds (B) The reduction in the cost of solar-power equipment
for unreimbursed care. has occurred despite increased raw material costs
(E) Even though philanthropic donations have for that equipment.
traditionally provided some support for the (C) Technological changes have increased the efficiency
hospitals, such donations are at present declining. of oil-fired power plants.
12. Generally scientists enter their field with the goal of (D) Most electricity is generated by coal-fired or
doing important new research and accept as their nuclear, rather than oil-fired, power plants.
colleagues those with similar motivation. Therefore, when (E) When the price of oil increases, reserves of oil not
any scientist wins renown as an expounder of science to previously worth exploiting become economically
general audiences, most other scientists conclude that this viable.
popularizer should no longer be regarded as a true
colleague. 15. Start-up companies financed by venture capitalist
have a much lower failure rate than companies financed
The explanation offered above for the low esteem in by other means. Source of financing, therefore, must be a
which scientific popularizers are held by research
( 58 )
PaGaLGuYS .com Education
TM
more important causative factor in the success of a start- 2. Crimes are mainly committed by the young, and for
up company than are such factors as the personal this reason merely increasing the number of police
characteristics of the entrepreneur, the quality of strategic officers or expenditures on police services has little effect
planning, or the management structure of the company. on reducing the crime rate. In fact, the only factor
Which of the following, if true, most seriously weakens associated with a crime-rate drop is a decrease in the
the argument above? number of people in the community aged fourteen to
thirty.
(A) Venture capitalists tend to be more responsive than
other sources of financing to changes in a start-up The findings above can best serve as part of an
company’s financial needs. argument against
(B) The strategic planning of a start-up company is a (A) the likelihood that any law enforcement program
less important factor in the long-term success of will be effective in reducing the crime rate within a
the company than are the personal characteristics short time
of the entrepreneur. (B) increasing prison terms for young people found
(C) More than half of all new companies fall within five guilty of crimes
years. (C) introducing compulsory military conscription for
(D) The management structures of start-up companies people aged seventeen to nineteen
are generally less formal than the management (D) raising the age at which students are permitted to
structures of ongoing businesses. leave school
(E) Venture capitalists base their decisions to fund (E) a community’s plan to increase the number of
start-up companies on such factors as the recreational and educational activities in which
characteristics of the entrepreneur and quality of young adults can participate
strategic planning of the company.
3. A 20 percent decline in lobster catches in Maine
16. The proportion of women among students enrolled waters since 1980 can be justifiably blamed on legislation
in higher education programs has increased over the past passed in 1972 to protect harbor seals. Maine’s
decades. This is partly shown by the fact that in 1959, population of harbor seals is now double the level
only 11 percent of the women between twenty and existing before protection was initiated, and these seals
twenty-one were enrolled in college, while in 1981, 30 are known to eat both fish and lobsters.
percent of the women between twenty and twenty-one Which of the following, if true, would most seriously
were enrolled in college. weaken the argument above?
To evaluate the argument above, it would be most useful (A) Harbor seals usually eat more fish than lobsters, but
to compare 1959 and 1981 with regard to which of the the seals are natural predators of both.
following characteristics? (B) Although harbor seals are skillful predators of
(A) The percentage of women between twenty and lobsters, they rarely finish eating their catch.
twenty-one who were not enrolled in college (C) Harbor seals attract tourists to Maine’s coastal
(B) The percentage of women between twenty and areas, thus revitalizing the local economy.
twenty-five who graduated from college (D) Authors of the 1972 legislation protecting harbor
(C) The percentage of women who, after attending seals were convinced that an increase in that
college, entered highly paid professions animal’s numbers would not have a measurably
(D) The percentage of men between twenty and negative impact on the lobster catch.
twenty-one who were enrolled in college (E) The record lobster harvests of the late 1970’s
(E) The percentage of men who graduated from high removed large numbers of mature lobsters from the
school reproductive stock.

TEST 18 4. Politician: Fewer people are entering the labor


25 Minutes 16 Questions market now than previously. If the economy grows, the
demand for motivated and educated people will far
1. Since a rhinoceros that has no horn is worthless to
outstrip the supply. Some companies have already
poachers, the Wildlife Protection Committee plans to
started to respond to this labor-market situation by
protect selected rhinoceroses from being killed by
finding better ways to keep their current employees.
poachers by cutting off the rhinos’ horns.
Their concern is a sure indicator that the economy is
The Wildlife Protection Committee’s plan assumes that growing.
(A) poachers do not kill rhinos that are worthless to Which of the following is the best criticism of the
them politician’s reasoning?
(B) hornless rhinos pose less of a threat to humans, (A) The fact that companies are making prudent
including poachers, than do rhinos that have horns preparations for a possible future development
(C) rhinos are the only animals poachers kill for their does not mean that this development is already
horns taking place.
(D) hornless rhinos can successfully defend their young (B) The fact that some companies now try harder to
against nonhuman predators keep their employees does not mean that they
(E) imposing more stringent penalties on poachers will used to be indifferent to employee morale.
not decrease the number of rhinos killed by (C) The fact that demand will outstrip supply does not
poachers mean that there will be no supply at all.
(D) The fact that the number of new entrants into the
( 59 )
PaGaLGuYS .com Education
TM
labor market is declining does not mean that the than fresh potatoes bought in 10-pound bags.
number of new entrants is lower than it has ever (B) Since fresh potatoes are 80 percent water, one
been. pound of dehydrated potatoes is the equivalent of
(E) The fact that current employees have become more 5 pounds of fresh potatoes.
valuable to some companies does not mean that (C) Peeled potatoes in cans are also more expensive
those employees will do their jobs better than they than the less convenient fresh potatoes.
used to. (D) Retail prices of dehydrated potatoes have declined
5. Under current federal law, employers are allowed to by 20 percent since 1960 to the current level of
offer their employees free parking spaces as a tax-free about $3 a pound.
benefit, but they can offer employees only up to $180 (E) As a consequence of labor and processing costs, all
per year as a tax-free benefit for using mass transit. The convenience foods cost more than the basic foods
government could significantly increase mass transit from which they are derived.
ridership by raising the limit of this benefit to meet
8. Consumers in California seeking personal loans
commuters’ transportation costs.
have fewer banks to turn to than do consumers elsewhere
The proposal above to increase mass transit ridership in the United States. This shortage of competition among
assumes that banks explains why interest rates on personal loans in
(A) current mass transit systems are subject to California are higher than in any other region of the
unexpected route closings and delays United States.
(B) using mass transit creates less air pollution per Which of the following, if true, most substantially
person than using a private automobile weakens the conclusion above?
(C) the parking spaces offered by employers as tax-free (A) Because of the comparatively high wages they must
benefits can be worth as much as $2,500 per year pay to attract qualified workers, California banks
(D) many employees are deterred by financial charge depositors more than banks elsewhere do
considerations from using mass transit to commute for many of the services they offer.
to their places of employment (B) Personal loans are riskier than other types of loans,
(E) because of traffic congestion on major commuter such as home mortgage loans, that banks make.
routes, it is often faster to travel to one’s place of (C) Since bank deposits in California are covered by the
employment by means of mass transit than by same type of insurance that guarantees bank
private automobile deposits in other parts of the United States, they
are no less secure than deposits elsewhere.
6. Which of the following best completes the passage
below? (D) The proportion of consumers who default on their
personal loans is lower in California than in any
“Government” does not exist as an independent entity other region of the United States.
defining policy. Instead there exists a group of
(E) Interest rates paid by California banks to depositors
democratically elected pragmatists sensitive to the
are lower than those paid by banks in other parts
electorate, who establish policies that will result in their
of the United States because in California there is
own reelection. Therefore, if public policy is hostile to,
less competition to attract depositors.
say, environmental concerns, it is not because of
governmental perversity but because elected officials 9. Technically a given category of insurance policy is
believe that______ underpriced if, over time, claims against it plus expenses
(A) environmentalists would be extremely difficult to associated with it exceed total income from premiums.
satisfy with any policy, however environmentally But premium income can be invested and will then yield
sound returns of its own. Therefore, an underpriced policy does
(B) environmental concerns are being accommodated as not represent a net loss in every case.
well as public funds permit The argument above is based on which of the following
(C) the public is overly anxious about environmental assumptions?
deterioration (A) No insurance policies are deliberately underpriced
(D) the majority of voters vote for certain politicians in order to attract customers to the insurance
because of those politicians’ idiosyncratic company offering such policies.
positions on policy issues (B) A policy that represents a net loss to the insurance
(E) the majority of voters do not strongly wish for a company is not an underpriced policy in every
different policy case.
(C) There are policies for which the level of claims per
7. Fresh potatoes generally cost about $2 for a 10-
year can be predicted with great accuracy before
pound bag, whereas dehydrated instant potatoes cost, on
premiums are set.
average, about $3 per pound. It can be concluded that
some consumers will pay 15 times as much for (D) The income earned by investing premium income is
convenience, since sales of this convenience food continue the most important determinant of an insurance
to rise. company’s profits.
(E) The claims against at least some underpriced
Which of the following, if true, indicates that there is a
policies do not require paying out all of the
major flaw in the argument above?
premium income from those policies as soon as it
(A) Fresh potatoes bought in convenient 2-pound bags is earned.
are about $1 a bag, or 2 1/2 times more expensive

( 60 )
PaGaLGuYS .com Education
TM
10. Purebred cows native to Mongolia produce, on (D) It makes less profit, on average, for a given dollar
average, 400 liters of milk per year; if Mongolian cattle amount of sales during the first three quarters
are crossbred with European breeds, the crossbred cows combined than during the fourth quarter.
can produce, on average, 2,700 liters per year. An (E) The per-camera price it pays to wholesalers is
international agency plans to increase the profitability of higher, on average, during the fourth quarter than
Mongolia’s dairy sector by encouraging widespread during any of the other three quarters.
crossbreeding of native Mongolian cattle with European
breeds. 13. Canadians now increasingly engage in “out-
shopping,” which is shopping across the national border,
Which of the following, if true, casts the most serious
where prices are lower. Prices are lower outside of Canada
doubt on the viability of the agency’s plan?
in large part because the goods-and-services tax that pays
(A) Not all European breeds of cattle can be for Canadian social services is not applied.
successfully bred with native Mongolian cattle.
Which one of the following is best supported on the
(B) Many young Mongolians now regard cattle raising
basis of the information above?
as a low-status occupation because it is less
lucrative than other endeavors open to them. (A) If the upward trend in out-shopping continues at a
significant level and the amounts paid by the
(C) Mongolia’s terrain is suitable for grazing native
government for Canadian social services are
herds but not for growing the fodder needed to
maintained, the Canadian goods-and-services tax
keep crossbred animals healthy.
will be assessed at a higher rate.
(D) Cowhide and leather products, not milk, make up
(B) If Canada imposes a substantial tariff on the goods
the bulk of Mongolia’s animal product exports to
bought across the border, a reciprocal tariff on
Europe.
cross-border shopping in the other direction will be
(E) Many European breeds of cattle attain average milk imposed, thereby harming Canadian businesses.
production levels exceeding 2,700 liters.
(C) The amounts the Canadian government pays out to
11. Any combination of overwork and stress inevitably those who provide social services to Canadians are
leads of insomnia. Managers at HiCorp, Inc., all suffer increasing.
from stress. A majority of the managers—despite their (D) The same brands of goods are available to Canadian
doctors’ warnings—work well over 60 hours per week, shoppers across the border as are available in
whereas the other managers work no more than the Canada.
normal 40 hours per week. HiCorp gives regular bonuses (E) Out-shopping purchases are subject to Canadian
only to employees who work more than 40 hours per taxes when the purchaser crosses the border to
week. bring them into Canada.
Which of the following conclusions is most strongly
14. Surveys indicate that 52 percent of all women aged
supported by the statements above?
eighteen to sixty-five are in the labor force (employed
(A) Managers at HiCorp work under conditions that are outside the home) in any given month. On the basis of
more stressful than the conditions under which these surveys, a market researcher concluded that 48
managers at most other companies work. percent of all women aged eighteen to sixty-five are full-
(B) Most of the employee bonuses given by HiCorp are time homemakers year-round.
given to managers.
Which of the following, if true, would most seriously
(C) At HiCorp, insomnia is more widespread among weaken the researcher’s conclusion?
managers than among any other group of
(A) More women are in the labor force today than
employees.
during any other period since the Second World
(D) No manager at HiCorp who works only 40 hours War.
per week suffers from overwork.
(B) Many workers, both men and women, enter and exit
(E) Most of the managers at HiCorp who receive regular the labor force frequently.
bonuses have insomnia.
(C) Although only a small sample of the total
12. Holiday receipts—the total sales recorded in the population is surveyed each month, these samples
fourth quarter of the year—determine the economic have been found to be a reliable indicator of total
success or failure of many retail businesses. Camco, a monthly employment.
retailer selling just one camera model, is an excellent (D) Surveys show that more women than ever before
example. Camco’s holiday receipts, on average, account consider having a rewarding job an important
for a third of its yearly total receipts and about half of its priority.
yearly profits. (E) Women who are in the labor force have more
If the statements above are true, which of the following discretionary income available to them than do
must also be true about Camco on the basis of them? women who are not.
(A) Its fixed expenses per camera sold are higher during 15. Left-handed persons suffer more frequently than do
the fourth quarter than for any of the other three right-handed persons from certain immune disorders, such
quarters. as allergies. Left-handers tend to have an advantage over
(B) It makes more profit during the first and third the right-handed majority, however, on tasks controlled
quarters combined than during the fourth quarter. by the right hemisphere of the brain, and mathematical
(C) Its per-camera retail price is lower, on average, reasoning is strongly under the influence of the right
during the fourth quarter than during any one of hemisphere in most people.
the first three quarters. If the information above is true, it best supports which
( 61 )
PaGaLGuYS .com Education
TM
of the following hypotheses? sell it without increasing the markup the company
(A) Most people who suffer from allergies or other usually allows for profit: potential clients would simply
such immune disorders are left-handed rather than not believe that something so inexpensive would really
right-handed. work. Yet Mechanicorp’s reputation is built on fair prices
(B) Most left-handed mathematicians suffer from some incorporating only modest profit margins.
kind of allergy. The statements above, if true, most strongly support
(C) There are proportionally more left-handers among which of the following?
people whose ability to reason mathematically is (A) Mechanicorp will encounter difficulties in trying to
above average than there are among people with set a price for its newest product that will promote
poor mathematical reasoning ability. sales without threatening to compromise the
(D) If a left-handed person suffers from an allergy, that company’s reputation.
person will probably be good at mathematics. (B) Mechanicorp achieves large annual profits, despite
(E) There are proportionally more people who suffer small profits per unit sold, by means of a high
from immune disorders such as allergies than there volume of sales.
are people who are left-handed or people whose (C) Mechanicorp made a significant computational error
mathematical reasoning ability is unusually good. in calculating the production costs for its newest
product.
16. After observing the Earth’s weather patterns and
(D) Mechanicorp’s newest product is intended to
the 11-year sunspot cycle of the Sun for 36 years,
perform tasks that can be performed by other
scientists have found that high levels of sunspot activity
devices costing less to manufacture.
precede shifts in wind patterns that affect the Earth’s
weather. One can conclude that meteorologists will be (E) Mechanicorp’s production processes are designed
able to improve their weather forecasts based on this with the same ingenuity as are the products that
information. the company makes.

Which of the following, if true, most seriously weakens 3. Companies in the country of Kollontay can sell
the argument above? semiconductors in the country of Valdivia at a price that
(A) Weather forecasts are more detailed today than they is below the cost to Valdivian companies of producing
were 36 years ago. them. To help those Valdivian companies, the Valdivian
(B) Scientists can establish that sunspot activity legislature plans to set a minimum selling price in Valdivia
directly affects the Earth’s weather. for semiconductors manufactured in Kollontay that is ten
percent greater than the average production costs for
(C) Evidence other than sunspot activity has previously
companies in Valdivia.
enabled meteorologists to forecast the weather
conditions that are predictable on the basis of Which of the following, if true, most seriously threatens
sunspot activity. the success of the plan?
(D) Scientists have not determined why the sunspot (A) The annual rate of inflation in Kollontay is expected
activity on the Sun follows an 11-year cycle. to exceed ten percent within the next year.
(E) It has been established that predictable wind (B) Valdivia is not the only country where companies in
patterns yield predictable weather patterns. Kollontay currently sell semiconductors.
(C) Some Valdivian companies that sell semiconductors
TEST 19 have announced that they plan to decrease their
25 Minutes 16 Questions price for semiconductors.
1. A publisher is now providing university professors (D) The government of Kollontay will also set a
with the option of ordering custom textbooks for their minimum price for selling semiconductors in that
courses. The professors can edit out those chapters of a country.
book they are not interested in and add material of their (E) Emerging companies in countries other than
own choosing. Kollontay will still be able to sell semiconductors
The widespread use of the option mentioned above is in Valdivia at a price below the cost to Valdivian
LEAST likely to contribute to fulfilling which of the companies to manufacture them.
following educational objectives?
4. An experimental microwave clothes dryer heats
(A) Coverage of material relevant to a particular student neither air nor cloth. Rather, it heats water on clothes,
body’s specific needs thereby saving electricity and protecting delicate fibers
(B) Offering advanced elective courses that pursue in- by operating at a lower temperature. Microwaves are
depth investigation of selected topics in a field waves that usually heat metal objects, but developers of a
(C) Ensuring that students nationwide engaged in a microwave dryer are perfecting a process that will
specific course of study are uniformly exposed to a prevent thin metal objects such as hairpins from heating
basic set of readings up and burning clothes.
(D) Making the textbooks used in university courses Which of the following, if true, most strongly indicates
more satisfactory from the individual teacher’s that the process, when perfected, will be insufficient to
point of view make the dryer readily marketable?
(E) Keeping students’ interest in a course by offering (A) Metal snap fasteners on clothes that are commonly
lively, well-written reading assignments put into drying machines are about the same
thickness as most hairpins.
2. Mechanicorp’s newest product costs so little to
(B) Many clothes that are currently placed into
make that it appears doubtful the company will be able to
( 62 )
PaGaLGuYS .com Education
TM
mechanical dryers are not placed there along with surgical patients operated on at night need less
hairpins or other thin metal objects. anesthesia. Since larger amounts of anesthesia pose
(C) The experimental microwave dryer uses more greater risks for patients, the risks of surgery could be
electricity than future, improved models would be reduced if operations routinely took place at night.
expected to use. Which of the following, if true, argues most strongly
(D) Drying clothes with the process would not cause against the view that surgical risks could be reduced by
more shrinkage than the currently used mechanical scheduling operations at night?
drying process causes. (A) Energy costs in hospitals are generally lower at
(E) Many clothes that are frequently machine-dried by night than they are during the day.
prospective customers incorporate thick metal (B) More babies are born between midnight and seven
parts such as decorative brass studs or buttons. o’clock in the morning than at any other time.
5. Airplane manufacturer: I object to your (C) Over the course of a year, people’s biological
characterization of our X-387 jets as dangerous. No X- rhythms shift slightly in response to changes in the
387 in commercial use has ever crashed or even had a amounts of daylight to which the people are
serious malfunction. exposed.
(D) Nurses and medical technicians are generally paid
Airline regulator: The problem with the X-387 is not
more per hour when they work during the night
that it, itself, malfunctions, but that it creates a
than when they work during the day.
turbulence in its wake that can create hazardous
conditions for aircraft in its vicinity. (E) Manual dexterity and mental alertness are lower in
the late night than they are during the day, even in
The airline regulator responds to the manufacturer by people accustomed to working at night.
doing which of the following?
(A) Characterizing the manufacturer’s assertion as Questions 8-9
stemming from subjective interest rather than from
Walter: A copy of an artwork should be worth exactly what
objective evaluation of the facts
the original is worth if the two works are visually
(B) Drawing attention to the fact that the indistinguishable. After all, if the two works are visually
manufacturer’s interpretation of the word indistinguishable, they have all the same qualities, and if they
“dangerous” is too narrow have all the same qualities, their prices should be equal.
(C) Invoking evidence that the manufacturer has
explicitly dismissed as irrelevant to the point at Marissa: How little you understand art! Even if someone
issue could make a perfect copy that is visually indistinguishable
(D) Citing statistical evidence that refutes the from the original, the copy would have a different history and
manufacturer’s claim hence not have all the same qualities as the original.
(E) Casting doubt on the extent of the manufacturer’s 8. Which of the following is a point at issue between
knowledge of the number of recent airline disasters Walter and Marissa?
6. Damaged nerves in the spinal cord do not regenerate (A) Whether a copy of an artwork could ever be
themselves naturally, nor even under the spur of nerve- visually indistinguishable from the original
growth stimulants. The reason, recently discovered, is the (B) Whether the reproduction of a work of art is ever
presence of nerve-growth inhibitors in the spinal cord. worth more than the original is worth
Antibodies that deactivate those inhibitors have now been (C) Whether a copy of a work of art is ever mistaken
developed. Clearly, then, nerve repair will be a standard for the original
medical procedure in the foreseeable future. (D) Whether a copy of a work of art could have all the
Which of the following, if true, casts the most serious same qualities as the original
doubt on the accuracy of the prediction above? (E) Whether originality is the only valuable attribute
(A) Prevention of the regeneration of damaged nerves is that a work of art can possess
merely a by-product of the main function in the
human body of the substances inhibiting nerve 9. Marissa uses which of the following techniques in
growth. attempting to refute Walter’s argument?
(B) Certain nerve-growth stimulants have similar (A) Attacking his assumption that the price of an
chemical structures to those of the antibodies artwork indicates its worth
against nerve-growth inhibitors. (B) Raising a point that would undermine one of the
(C) Nerves in the brain are similar to nerves in the claims on which his conclusion is based
spinal cord in their inability to regenerate (C) Questioning his claim that a perfect copy of a work
themselves naturally. of art would be visually indistinguishable from the
(D) Researchers have been able to stimulate the growth original
of nerves not located in the spinal cord by using (D) Giving reason to believe that Walter is unable to
only nerve-growth stimulants. judge the quality of a work of art because of his
(E) Deactivating the substances inhibiting nerve growth inadequate understanding of the history of art
for an extended period would require a steady (E) Proposing alternative criteria for determining
supply of antibodies. whether two works of art are visually
indistinguishable
7. The human body secretes more pain-blocking
hormones late at night than during the day. Consequently, 10. Magnetic resonance imaging (MRI)—a noninvasive

( 63 )
PaGaLGuYS .com Education
TM
diagnostic procedure—can be used to identify blockages abilities that do not play an important role in
in the coronary arteries. In contrast to angiograms—the driving safely.
invasive procedure customarily used—MRI’s pose no (E) Awareness of the public-service advertisement is
risk to patients. Thus, to guarantee patient safety in the higher among the general population than it was
attempt to diagnose arterial blockages, MRI’s should among the subjects in the hospital study.
replace angiograms in all attempts at diagnosing coronary
blockages. 13. Investigator: XYZ Coins has misled its clients by
promoting some coins as “extremely rare” when in fact
Which of the following, if true, would most support the
those coins are relatively common and readily available.
recommendation above?
(A) Angiograms can be used to diagnose conditions XYZ agent: That is ridiculous. XYZ Coins is one of the
other than blockages in arteries. largest coin dealers in the world. We authenticate the
coins we sell through a nationally recognized firm and
(B) MRI’s were designed primarily in order to diagnose
operate a licensed coin dealership.
blockages in the coronary arteries.
(C) Angiograms reveal more information about the The XYZ agent’s reply is most vulnerable to the
nature of a blockage than an MRI can. criticism that it
(D) An MRI is just as likely as an angiogram to identify (A) exaggerates the investigator’s a claims in order to
an arterial blockage. make them appear absurd
(E) Some patients for whom an angiogram presents no (B) accuses the investigator of bias but presents no
risk are unwilling to undergo an MRI. evidence to support that accusation
(C) fails to establish that other coin dealers do not also
11. Naturally occurring chemicals cannot be newly authenticate the coins those dealers sell
patented once their structures have been published. (D) lists strengths of XYZ Coins while failing to
Before a naturally occurring chemical compound can be address the investigator’s charge
used as a drug, however, it must be put through the same
(E) provides no definition for the inherently vague
rigorous testing program as any synthetic compound,
phrase “extremely rare”
culminating in a published report detailing the chemical’s
structure and observed effects. 14. Both Writewell and Express provide round-the-
If the statements above are true, which of the following clock telephone assistance to any customer who uses
must also be true on the basis of them? their word-processing software. Since customers only call
(A) Any naturally occurring chemical can be reproduced the hot lines when they find the software difficult to use,
synthetically once its structure is known. and the Writewell hot line receives four times as many
calls as the Express hot line, Writewell’s word-processing
(B) Synthetically produced chemical compounds cannot
software must be more difficult to use than Express’s.
be patented unless their chemical structures are
made public. Which of the following, if true, most strengthens the
(C) If proven no less effective, naturally occurring argument above?
chemicals are to be preferred to synthetic (A) Calls to the Express hot line are almost twice as
compounds for use in drugs. long, on average, as are calls to the Writewell hot
(D) Once a naturally occurring compound has been line.
approved for use as a drug, it can no longer be (B) Express has three times the number of word-
newly patented. processing software customers that Writewell has.
(E) A naturally occurring chemical cannot be patented (C) Express receives twice as many letters of complaint
unless its effectiveness as a drug has been about its word-processing software as Writewell
rigorously established. receives about its word-processing software.
(D) The number of calls received by each of the two hot
12. A public-service advertisement advises that people lines has been gradually increasing.
who have consumed alcohol should not drive until they
(E) The Writewell hot-line number is more widely
can do so safely. In a hospital study, however, subjects
publicized than the Express hot-line number.
questioned immediately after they consumed alcohol
underestimated the time necessary to regain their driving 15. Over the last century, paleontologists have used
ability. This result indicates that many people who drink small differences between fossil specimens to classify
before driving will have difficulty following the triceratops into sixteen species. This classification is
advertisement’s advice. unjustified, however, since the specimens used to
Which of the following, if true, most strongly supports distinguish eleven of the species come from animals that
the argument above? lived in the same area at the same time.
(A) Many people, if they plan to drink alcohol, make Which of the following, if true, would enable the
arrangements beforehand for a nondrinker to drive conclusion of the argument to be properly drawn?
them home. (A) Not every species that lived in a given area is
(B) The subjects in the hospital study generally rated preserved as a fossil.
their abilities more conservatively than would (B) At least one individual of every true species of
people drinking alcohol outside a hospital setting. triceratops has been discovered as a fossil
(C) Some people refrain from drinking if they will have specimen.
to drive to get home afterward. (C) No geographical area ever supports more than three
(D) The subjects in the hospital study were also similar species at the same time.
questioned about the time necessary to regain
( 64 )
PaGaLGuYS .com Education
TM
(D) In many species, individuals display quite marked The statements above, if true, most strongly support
variation. which of the following hypotheses?
(E) Differences between fossil specimens of triceratops (A) The waters of the island lake are more severely
that came from the same area are no less distinctive polluted than those of Lake Superior.
than differences between specimens that came (B) The toxaphene was carried to the island in the
from different areas. atmosphere by winds.
16. Many consumers are concerned about the ecological (C) Banning chemicals such as toxaphene does not aid
effects of wasteful packaging. This concern probably the natural environment.
explains why stores have been quick to stock new (D) Toxaphene has adverse effects on human beings but
cleaning products that have been produced in a not on other organisms.
concentrated form. The concentrated form is packaged in (E) Concentrations of toxaphene in the soil of cotton-
smaller containers that use less plastic and require less growing regions are not sufficient of be measurable.
transportation space.
3. Last year in the United States, women who ran for
Which of the following, if true, most seriously state and national offices were about as likely to win as
undermines the explanation offered above? men. However, only about fifteen percent of the
(A) Few consumers believe that containers of candidates for these offices were women. Therefore, the
concentrated cleaning products are merely small reason there are so few women who win elections for
packages of regular cleaning products. these offices is not that women have difficulty winning
(B) The containers in which concentrated cleaning elections but that so few women want to run.
products are packaged are no harder to recycle than Which of the following, if true, most seriously
those in which regular cleaning products are undermines the conclusion given?
packaged.
(A) Last year the proportion of women incumbents
(C) Those concentrated cleaning products that are who won reelection was smaller than the
intended to be used diluted have clear instructions proportion of men incumbents who won
for dilution printed on their labels. reelection.
(D) The smaller containers of concentrated cleaning (B) Few women who run for state and national offices
products enable supermarkets and drugstores to run against other women.
increase their revenues from a given shelf space.
(C) Most women who have no strong desire to be
(E) Consumer pressure has led to the elimination of politicians never run for state and national offices.
wasteful cardboard packaging that was used for
(D) The proportion of people holding local offices who
compact discs.
are women is smaller than the proportion of
TEST 20 people holding state and national offices who are
25 Minutes 16 Questions women.
(E) Many more women than men who want to run for
1. In the first half of this year, from January to June,
state and national offices do not because they
about three million videocassette recorders were sold.
cannot get adequate funding for their campaigns.
This number is only 35 percent of the total number of
videocassette recorders sold last year. Therefore, total 4. Samples from a ceramic vase found at a tomb in
sales of videocassette recorders will almost certainly be Sicily prove that the vase was manufactured in Greece.
lower for this year than they were for last year. Since the occupant of the tomb died during the reign of a
Which of the following, if true, most seriously weakens Sicilian ruler who lived 2,700 years ago, the location of
the conclusion above? the vase indicates that there was trade between Sicily and
(A) The total number of videocassette recorders sold Greece 2,700 years ago.
last year was lower than the total number sold in Which of the following is an assumption on which the
the year before that. argument depends?
(B) Most people who are interested in owning a (A) Sicilian potters who lived during the reign of the
videocassette recorder have already purchased one. ruler did not produce work of the same level of
(C) Videocassette recorders are less expensive this year quality as did Greek potters.
than they were last year. (B) Sicilian clay that was used in the manufacture of
(D) Of the videocassette recorders sold last year, almost pottery during the ruler’s reign bore little
60 percent were sold in January. resemblance to Greek clay used to manufacture
(E) Typically, over 70 percent of the sales of pottery at that time.
videocassette recorders made in a year occur in the (C) At the time that the occupant of the tomb was alive,
months of November and December. there were ships capable of transporting large
quantities of manufactured goods between Sicily
2. Mud from a lake on an uninhabited wooded island and Greece.
in northern Lake Superior contains toxic chemicals, (D) The vase that was found at the Sicilian tomb was
including toxaphene, a banned pesticide for cotton that not placed there many generations later by
previously was manufactured and used, not in nearby descendants of the occupant of the tomb.
regions of Canada or the northern United States, but in
(E) The occupant of the tomb was not a member of the
the southern United States. No dumping has occurred on
royal family to which the Sicilian ruler belonged.
the island. The island lake is sufficiently elevated that
water from Lake Superior does not reach it. 5. In several cities, the government is going ahead with

( 65 )
PaGaLGuYS .com Education
TM
ambitious construction projects despite the high office landfills.
vacancy rates in those cities. The vacant offices, though (C) In many landfills, a significant proportion of space
available for leasing, unfortunately do not meet the is taken up by materials other than discarded
requirements for the facilities needed, such as court packaging materials.
houses and laboratories. The government, therefore, is not (D) It is impossible to avoid entirely the use of
guilty of any fiscal wastefulness. packaging materials that are not biodegradable in
Which of the following is an assumption on which the landfills.
argument above depends? (E) Sometimes, in packaging an item, plastics that are
(A) Adaptation of vacant office space to meet the not biodegradable in landfills are combined with
government’s requirements, if possible, would not cardboard.
make leasing such office space a more cost-
effective alternative to new construction. 8. Any serious policy discussion about acceptable
levels of risk in connection with explosions is not well
(B) The government prefers leasing facilities to owning
served if the participants fail to use the word “explosion”
them in cases where the two alternatives are
and use the phrase “energetic disassembly” instead. In
equally cost-effective.
fact, the word “explosion” elicits desirable reactions, such
(C) If facilities available for leasing come very close to as a heightened level of attention, whereas the substitute
meeting the government’s requirements for phrase does not. Therefore, of the two terms, “explosion”
facilities the government needs, the government can is the one that should be used throughout discussions of
relax its own requirements slightly and consider this sort.
those facilities in compliance.
Which of the following is an assumption on which the
(D) The government’s construction projects would not
argument above depends?
on being completed, add to the stock of facilities
available for leasing in the cities concerned. (A) In the kind of discussion at issue, the advantages of
desirable reactions to the term “explosion”
(E) Before embarking on any major construction
outweigh the drawbacks, if any, arising from
project, the government is required by law to
undesirable reactions to that term.
establish beyond any reasonable doubt that there
are no alternatives that are most cost-effective. (B) The phrase “energetic disassembly” has not so far
been used as a substitute for the word “explosion”
6. Potato cyst nematodes are a pest of potato crops. in the kind of discussion at issue.
The nematodes can lie dormant for several years in their (C) In any serious policy discussion, what is said by
cysts, which are protective capsules, and do not emerge the participants is more important than how it is
except in the presence of chemicals emitted by potato put into words.
roots. A company that has identified the relevant (D) The only reason that people would have for using
chemicals is planning to market them to potato farmers to “energetic disassembly” in place of “explosion” is
spread on their fields when no potatoes are planted; any to render impossible any serious policy discussion
nematodes that emerge will soon starve to death. concerning explosions.
Which of the following, if true, best supports the claim (E) The phrase “energetic disassembly” is not
that the company’s plan will be successful? necessarily out of place in describing a controlled
(A) Nematodes that have emerged from their cysts can rather than an accidental explosion.
be killed by ordinary pesticides.
9. Mannis Corporation’s archival records are stored in
(B) The only part of a potato plant that a nematode
an obsolete format that is accessible only by its current
eats is the roots.
computer system; thus they are inaccessible when that
(C) Some bacteria commonly present in the roots of system is not functioning properly. In order to avoid the
potatoes digest the chemicals that cause the possibility of losing access to their archival records in the
nematodes to emerge from their cysts. case of computer malfunction, Mannis plans to replace
(D) Trials have shown that spreading even minute its current computer system with a new system that
quantities of the chemicals on potato fields caused stores records in a format that is accessible to several
nine-tenths of the nematodes present to emerge different systems.
from their cysts.
The answer to which of the following questions would
(E) The chemicals that cause the nematodes to emerge be most helpful in evaluating the effectiveness of the
from their cysts are not emitted all the time the plan as a means of retaining access to the archival
potato plant is growing. records?
7. It is better for the environment if as much of all (A) Will the new computer system require fewer
packaging as possible is made from materials that are operators than the current system requires?
biodegradable in landfills. Therefore, it is always a change (B) Has Mannis Corporation always stored its archival
for the worse to replace packaging made from paper or records in a computerized format?
cardboard with packaging made from plastics that are not (C) Will the new computer system that Mannis plans
biodegradable in landfills. ensure greater security for the records stored than
Which of the following, if true, constitutes the strongest does Mannis’ current system?
objection to the argument above? (D) Will Mannis’ current collection of archival records
(A) The paper and cardboard used in packaging are be readily transferable to the new computer
usually not biodegradable in landfills. system?
(B) Some plastic used in packaging is biodegradable in (E) Will the new computer system be able to perform

( 66 )
PaGaLGuYS .com Education
TM
many more tasks than the current system is able to (B) The number of automobiles built by Deluxe each
perform? year has not increased sharply since 1970.
(C) Owners of Deluxe automobiles typically keep their
10. Last year the worldwide paper industry used over
cars well maintained.
twice as much fresh pulp (pulp made directly from raw
plant fibers) as recycled pulp (pulp made from (D) Since 1970, Deluxe has made fewer changes in the
wastepaper). A paper-industry analyst has projected that automobiles it manufactures than other car
by 2010 the industry will use at least as much recycled companies have made in their automobiles.
pulp annually as it does fresh pulp, while using a greater (E) Deluxe automobiles have been selling at relatively
quantity of fresh pulp than it did last year. stable prices in recent years.
If the information above is correct and the analyst’s 13. Many state legislatures are considering proposals to
projections prove to be accurate, which of the following the effect that certain policies should be determined not
projections must also be accurate? by the legislature itself but by public referenda in which
(A) In 2010 the paper industry will use at least twice as every voter can take part. Critics of the proposals argue
much recycled pulp as it did last years. that the outcomes of public referenda would be biased,
(B) In 2010 the paper industry will use at least twice as since wealthy special-interest groups are able to influence
much total pulp as it did last year. voters’ views by means of television advertisements.
(C) In 2010 the paper industry will produce more paper Which of the following, if true, most strengthens the
from a given amount of pulp than it did last year. critics’ argument?
(D) As compared with last year, in 2010 the paper (A) Many state legislators regard public referenda as a
industry will make more paper that contains only way of avoiding voting on issues on which their
recycled pulp. constituents are divided.
(E) As compared with last year, in 2010 the paper (B) During elections for members of the legislature, the
industry will make less paper that contains only number of people who vote is unaffected by
fresh pulp. whether the candidates run television
advertisements or not.
11. In malaria-infested areas, many children tend to
(C) Proponents of policies that are opposed by wealthy
suffer several bouts of malaria before becoming immune
special-interest groups are often unable to afford
to the disease. Clearly, what must be happening is that
advertising time on local television stations.
those children’s immune systems are only weakly
stimulated by any single exposure to the malaria parasite (D) Different special-interest groups often take
and need to be challenged several times to produce an opposing positions on questions of which policies
effective immune response. the state should adopt.
(E) Television stations are reluctant to become
Which of the following, if true, most seriously
associated with any one political opinion, for fear
undermines the explanatory hypothesis?
of losing viewers who do not share that opinion.
(A) Immediately after a child has suffered a bout of
malaria, the child’s caregivers tend to go to great 14. Advertisement: Of the many over-the-counter
lengths in taking precautions to prevent another medications marketed for the relief of sinus headache.
infection, but this level of attention is not SineEase costs the least per dose. And SineEase is as
sustained. effective per dose as the most effective of those other
(B) Malaria is spread from person to person by medications. So for relief from sinus headaches, SineEase
mosquitoes, and mosquitoes have become is the best buy.
increasingly resistant to the pesticides used to Which of the following, if true, most seriously weakens
control them. the argument above?
(C) A certain gene, if inherited by children from only (A) Most of the over-the-counter medications marketed
one of their parents, can render those children for the relief of sinus headache are equally effective
largely immune to infection with malaria. per dose in providing such relief.
(D) Antimalaria vaccines, of which several are in (B) Many of the over-the-counter medications marketed
development, are all designed to work by for the relief of sinus headache contain the same
stimulating the body’s immune system. active ingredient as SineEase.
(E) There are several distinct strains of malaria, and the (C) People who suffer from frequent sinus headaches
body’s immune response to any one of them does are strongly advised to consult a doctor before
not protect it against the others. taking any over-the-counter medication.
12. An advertisement designed to convince readers of (D) An over-the-counter medication that is marketed for
the great durability of automobiles manufactured by the the relief of symptoms of head cold is identical in
Deluxe Motor Car Company cites as evidence the fact composition to SineEase but costs less per dose.
that over half of all automobiles built by the company (E) The per dose price for any given over-the-counter
since 1970 are still on the road today, compared to no medication marketed for the relief of sinus
more than a third for any other manufacturer. headache is higher for smaller packages than it is
for larger packages.
Which of the following, if true, most strongly supports
the advertisement’s argument? 15. In the United States, vacationers account for more
(A) After taking inflation into account, a new Deluxe than half of all visitors to what are technically called
automobile costs only slightly more than a new “pure aquariums” but for fewer than one quarter of all
model did in 1970. visitors to zoos, which usually include a “zoo aquarium”
( 67 )
PaGaLGuYS .com Education
TM
of relatively modest scope.
Which of the following, if true, most helps to account
for the difference described above between visitors to
zoos and visitors to pure aquariums?
(A) In cities that have both a zoo and a pure aquarium,
local residents are twice as likely to visit the
aquarium as they are to visit the zoo.
(B) Virtually all large metropolitan areas have zoos,
whereas only a few large metropolitan areas have
pure aquariums.
(C) Over the last ten years, newly constructed pure
aquariums have outnumbered newly established
zoos by a factor of two to one.
(D) People who visit a zoo in a given year are two times
more likely to visit a pure aquarium that year than
are people who do not visit a zoo.
(E) The zoo aquariums of zoos that are in the same city
as a pure aquarium tend to be smaller than the
aquariums of zoos that have no pure aquarium
nearby.

16. Which of the following, if true, is the most logical


completion of the argument below?
The tax system of the Republic of Grootland
encourages borrowing by granting its taxpayers tax relief
for interest paid on loans. The system also discourages
saving by taxing any interest earned on savings.
Nevertheless, it is clear that Grootland’s tax system
does not consistently favor borrowing over saving, for if
it did, there would be no______
(A) tax relief in Grootland for those portions of a
taxpayer’s income, if any, that are set aside to
increase that taxpayer’s total savings
(B) tax relief in Grootland for the processing fees that
taxpayers pay to lending institutions when
obtaining certain kinds of loans
(C) tax relief in Grootland for interest that taxpayers are
charged on the unpaid balance in credit card
accounts
(D) taxes due in Grootland on the cash value of gifts
received by taxpayers from banks trying to
encourage people to open savings accounts
(E) taxes due in Grootland on the amount that a
taxpayer has invested in interest-bearing savings
accounts.

( 68 )
PaGaLGuYS .com Education
TM
TEST - 1
1. D 2. B 3. E 4. D 5. A
6. C 7. E 8. E 9. D 10. D
11. C 12. E 13. A 14. C 15. A
16. B 17. D 18. C 19. C 20. A

TEST-2
1. A 2. C 3. B 4. E 5. B
6. E 7. D 8. B 9. B 10. B
11. C 12. A 13. B 14. A 15. D
16. A 17. A 18. E 19. A 20. D

TEST - 3

1. B 2. B 3. C 4. E 5. E
6. A 7. C 8. C 9. C 10. E
11. D 12. B 13. B 14. D 15. A
16. C 17. C 18. D 19. A 20. E

TEST - 4

1. A 2. C 3. B 4. A 5. B
6. B 7. B 8. D 9. D 10. E
11. C 12. E 13. C 14. B 15. C
16. D 17. A 18. C 19. E 20. B

TEST - 5

1. D 2. E 3. E 4. D 5. C
6. B 7. B 8. D 9. C 10. E
11. D 12. E 13. C 14. A 15. A
16. C 17. C 18. E 19. A 20. C

TEST - 6

1. B 2. B 3. B 4. C 5. E
6. C 7. B 8. C 9. C 10. D
11. D 12. E 13. A 14. A 15. D
16. D 17. E 18. E 19. A 20. C

TEST - 7

1. C 2. C 3. E 4. C 5. A
6. D 7. D 8. C 9. A 10. A
11. D 12. A 13. D 14. B 15. E
16. E 17. D 18. B 19. E 20. B

TEST - 8
1. E 2. A 3. B 4. D 5. C
6. A 7. A 8. B 9. E 10. B
11. A 12. A 13. E 14. B 15. B
16. E 17. B 18. C 19. D 20. E

TEST - 9

1. A 2. B 3. D 4. D 5. B
6. E 7. C 8. D 9. A 10. E
11. B 12. D 13. A 14. C 15. E
16. B 17. B 18. A 19. C 20. A

TEST - 10

1. C 2. C 3. E 4. D 5. A
6. D 7. A 8. D 9. C 10. A
11. D 12. C 13. B 14. B 15. B
16. E 17. A 18. B 19. B 20. D
( 69 )
PaGaLGuYS .com Education
TM
TEST - 11

1. A 2. D 3. B 4. C 5. A
6. E 7. D 8. B 9. E 10. C
11. C 12. D 13. E 14. A 15. B
16. A 17. C 18. B 19. E 20. B

TEST - 12

1. D 2. B 3. D 4. C 5. E
6. C 7. E 8. C 9. A 10. C
11. C 12. E 13. C 14. A 15. B
16. E 17. E 18. D 19. 20.

TEST - 13
1. D 2. E 3. C 4. B 5. A
6. D 7. D 8. E 9. C 10. C
11. A 12. E 13. A 14. E 15. D

TEST - 14

1. E 2. C 3. A 4. B 5. A
6. B 7. E 8. B 9. A 10. E
11. C 12. B 13. D 14. D 15. A
16. B

TEST - 15

1. D 2. E 3. D 4. A 5. B
6. D 7. A 8. D 9. A 10. C
11. C 12. E 13. B 14. C 15. B
16. D

TEST - 16

1. D 2. A 3. B 4. E 5. B
6. E 7. C 8. D 9. B 10. B
11. C 12. C 13. C 14. A 15. C
16. B

TEST - 17

1. B 2. E 3. C 4. C 5. B
6. A 7. D 8. D 9. B 10. A
11. B 12. D 13. E 14. C 15. E
16. D

TEST - 18

1. A 2. A 3. E 4. A 5. D
6. E 7. B 8. A 9. E 10. C
11. E 12. D 13. A 14. B 15. C
16. C

TEST - 19

1. C 2. A 3. E 4. E 5. B
6. A 7. E 8. D 9. B 10. D
11. D 12. B 13. D 14. B 15. C
16. D

TEST - 20

1. E 2. B 3. E 4. D 5. A
6. D 7. A 8. A 9. D 10. A
11. E 12. B 13. C 14. D 15. B
16. A
( 70 )
PaGaLGuYS .com Education
TM

You might also like